Você está na página 1de 192

Arrasa no Exame: O teu Guia de Estudo de

Matemática A!
Ricardo Ferreira

Atualizado a 19/9/2021

Dica: Clica nos tı́tulos e subtı́tulos para ires automaticamente para essa
página.

Conteúdo
1 Geometria 1 — Produto escalar e comparação de planos e
retas 5
1.1 Colineares ou não colineares? . . . . . . . . . . . . . . . . . . 5
1.2 Vetor perpendicular a dois vetores não colineares . . . . . . . 6
1.3 Vetor perpendicular a outro . . . . . . . . . . . . . . . . . . . 7
1.4 Produto escalar e ângulos . . . . . . . . . . . . . . . . . . . . 9

2 Geometria 2 — Lugares Geométricos 11


2.1 Equações e interseções de retas e planos . . . . . . . . . . . . 11
2.2 Superfı́cie Esférica . . . . . . . . . . . . . . . . . . . . . . . . 13
2.3 Plano Tangente à Superfı́cie Esférica . . . . . . . . . . . . . . 13
2.4 Plano Mediador . . . . . . . . . . . . . . . . . . . . . . . . . . 14

3 Trigonometria 1 — Definições e Equações 16


3.1 Trigonometria em ]0, π/2[ . . . . . . . . . . . . . . . . . . . . 16
3.2 Trigonometria em R . . . . . . . . . . . . . . . . . . . . . . . 18
3.3 Redução ao primeiro quadrante . . . . . . . . . . . . . . . . . 21
3.4 Equações Trigonométricas . . . . . . . . . . . . . . . . . . . . 23

4 Trigonometria 2 – Tangente e Declive 25


4.1 Visualizando a tangente . . . . . . . . . . . . . . . . . . . . . 25
4.2 Inclinação e Declive . . . . . . . . . . . . . . . . . . . . . . . 27
4.3 Declive da reta perpendicular . . . . . . . . . . . . . . . . . . 29
Todos os direitos reservados.

1
Ricardo Ferreira

5 Complexos 1 – Forma Algébrica e Trigonométrica 30


5.1 Forma Algébrica e Trigonométrica . . . . . . . . . . . . . . . 30
5.2 Somar e Multiplicar nos complexos . . . . . . . . . . . . . . . 30
5.3 Simétrico: multiplicar por −1 (como subtrair) . . . . . . . . . 32
5.4 Potências de i: multiplicar por i . . . . . . . . . . . . . . . . 32
5.5 Conjugado (como dividir complexos) . . . . . . . . . . . . . . 33
5.6 Passar algébrica ↔ trigonométrica . . . . . . . . . . . . . . . 34
5.7 Complexos e Trigonometria . . . . . . . . . . . . . . . . . . . 35
5.8 Recapitulando . . . . . . . . . . . . . . . . . . . . . . . . . . . 36

6 Complexos 2 – Regiões e Equações 37


6.1 Regiões no Plano Complexo . . . . . . . . . . . . . . . . . . . 37
6.2 Equações . . . . . . . . . . . . . . . . . . . . . . . . . . . . . 42
6.3 Radiciação: a equação especial z n = w . . . . . . . . . . . . . 43
6.4 Recapitulando . . . . . . . . . . . . . . . . . . . . . . . . . . . 44

7 Sucessões — Progressões, Limites e Propriedades 46


7.1 Progressões Aritméticas . . . . . . . . . . . . . . . . . . . . . 46
7.2 Progressões Geométricas . . . . . . . . . . . . . . . . . . . . . 47
7.3 Limites de sucessões . . . . . . . . . . . . . . . . . . . . . . . 48
7.4 Propriedades das Sucessões . . . . . . . . . . . . . . . . . . . 50

8 Funções 1 — Funções Clássicas e Equações 54


8.1 As funções clássicas . . . . . . . . . . . . . . . . . . . . . . . . 54
8.2 Funções compostas . . . . . . . . . . . . . . . . . . . . . . . . 55
8.3 Calcular domı́nios . . . . . . . . . . . . . . . . . . . . . . . . 57
8.4 Funções Inversas I: logaritmos e equações . . . . . . . . . . . 57

9 Funções 2 — Transformações, Inequações, Sinal e Zeros 62


9.1 Transformações de funções I: como visualizar . . . . . . . . . 62
9.2 Transformações de funções II: funções pares e ı́mpares . . . . 65
9.3 Funções Inversas II: Como calcular . . . . . . . . . . . . . . . 66
9.4 Resolver inequações . . . . . . . . . . . . . . . . . . . . . . . 68
9.5 Zeros e Sinal . . . . . . . . . . . . . . . . . . . . . . . . . . . 69

10 Limites 1 — Como Calcular Limites 71


10.1 A estratégia geral . . . . . . . . . . . . . . . . . . . . . . . . . 71
10.2 Os limites clássicos . . . . . . . . . . . . . . . . . . . . . . . . 72
10.3 Truques I: Álgebra de Limites . . . . . . . . . . . . . . . . . . 72
10.4 Truques II: Inversão . . . . . . . . . . . . . . . . . . . . . . . 75
10.5 Truques III: Intermediário . . . . . . . . . . . . . . . . . . . . 76
10.6 Truques IV: Pôr em evidência . . . . . . . . . . . . . . . . . . 77
10.7 Truques V: Conjugado . . . . . . . . . . . . . . . . . . . . . . 79
10.8 Truques VI: Mudança de variável . . . . . . . . . . . . . . . . 80

Erro ou sugestão? Contacta-me: mailto:ricardoferreira.contactar@gmail.com 2


Vê no YouTube: https://www.youtube.com/channel/UCkRcdeyQ50TWFmk7vyuzf_g
Ricardo Ferreira

10.9 Truques VII: Criar constante . . . . . . . . . . . . . . . . . . 82


10.10Truques VIII: Um limite seminotável . . . . . . . . . . . . . . 82
10.11Truques IX: Continuidade . . . . . . . . . . . . . . . . . . . . 83
10.12Como calcular qualquer limite . . . . . . . . . . . . . . . . . . 84

11 Limites 2 — Continuidade e Assı́ntotas 85


11.1 Limites segundo Heine . . . . . . . . . . . . . . . . . . . . . . 85
11.2 Continuidade . . . . . . . . . . . . . . . . . . . . . . . . . . . 86
11.3 Assı́ntotas Verticais . . . . . . . . . . . . . . . . . . . . . . . . 89
11.4 Assı́ntotas Não Verticais . . . . . . . . . . . . . . . . . . . . . 92
11.5 Recapitulando . . . . . . . . . . . . . . . . . . . . . . . . . . . 93

12 Derivadas — Definição, Declive da Reta Tangente, Monoto-


nia e Concavidade 95
12.1 Taxa Média de Variação e Derivada . . . . . . . . . . . . . . 95
12.2 Como calcular derivadas . . . . . . . . . . . . . . . . . . . . . 96
12.3 Aplicações I: calcular limites . . . . . . . . . . . . . . . . . . . 101
12.4 Aplicações II: declive da tangente ao gráfico . . . . . . . . . . 102
12.5 Aplicações III: monotonia e extremos . . . . . . . . . . . . . . 104
12.6 Aplicações IV: concavidade e pontos de inflexão . . . . . . . . 106

13 Teorema de Bolzano–Cauchy 108

14 Combinatória 1 — Como Contar 111


14.1 Estratégia Geral . . . . . . . . . . . . . . . . . . . . . . . . . 111
14.2 Como contar I: “E (quantas opções)?” . . . . . . . . . . . . . 111
14.3 Como contar II: “Ou” . . . . . . . . . . . . . . . . . . . . . . 113
14.4 Como contar III: “Ordenar” . . . . . . . . . . . . . . . . . . . 114
14.5 Como contar IV: “Escolher” . . . . . . . . . . . . . . . . . . . 115
14.6 Estratégia Geral II e Truques Especiais . . . . . . . . . . . . 118

15 Combinatória 2 — Triângulo de Pascal, Binómio de Newton


e Equações 123
15.1 Triângulo de Pascal . . . . . . . . . . . . . . . . . . . . . . . . 123
15.2 Binómio de Newton . . . . . . . . . . . . . . . . . . . . . . . 127
15.3 Equações . . . . . . . . . . . . . . . . . . . . . . . . . . . . . 130

16 Probabilidades — Regra de Laplace e Fórmulas das Proba-


bilidades 134
16.1 Probabilidades I: Regra de Laplace . . . . . . . . . . . . . . . 134
16.2 Probabilidades II: Fórmulas e Tabela . . . . . . . . . . . . . . 135

Erro ou sugestão? Contacta-me: mailto:ricardoferreira.contactar@gmail.com 3


Vê no YouTube: https://www.youtube.com/channel/UCkRcdeyQ50TWFmk7vyuzf_g
Ricardo Ferreira

17 Soluções 141
17.1 Geometria 1 — Produto escalar e comparação de planos e retas141
17.2 Geometria 2 — Lugares Geométricos . . . . . . . . . . . . . . 144
17.3 Trigonometria 1 — Definições e Equações . . . . . . . . . . . 146
17.4 Trigonometria 2 – Tangente e Declive . . . . . . . . . . . . . 149
17.5 Complexos 1 – Forma Algébrica e Trigonométrica . . . . . . . 150
17.6 Complexos 2 – Regiões e Equações . . . . . . . . . . . . . . . 153
17.7 Sucessões — Progressões, Limites e Propriedades . . . . . . . 160
17.8 Funções 1 — Funções Clássicas e Equações . . . . . . . . . . 163
17.9 Funções 2 — Transformações, Inequações, Sinal e Zeros . . . 166
17.10Limites 1 — Como Calcular Limites . . . . . . . . . . . . . . 169
17.11Limites 2 — Continuidade e Assı́ntotas . . . . . . . . . . . . 173
17.12Derivadas — Definição, Declive da Reta Tangente, Monotonia
e Concavidade . . . . . . . . . . . . . . . . . . . . . . . . . . 176
17.13Teorema de Bolzano–Cauchy . . . . . . . . . . . . . . . . . . 182
17.14Combinatória 1 — Como Contar . . . . . . . . . . . . . . . . 183
17.15Combinatória 2 — Triângulo de Pascal, Binómio de Newton
e Equações . . . . . . . . . . . . . . . . . . . . . . . . . . . . 187
17.16Probabilidades — Regra de Laplace e Fórmulas das Proba-
bilidades . . . . . . . . . . . . . . . . . . . . . . . . . . . . . . 190

Erro ou sugestão? Contacta-me: mailto:ricardoferreira.contactar@gmail.com 4


Vê no YouTube: https://www.youtube.com/channel/UCkRcdeyQ50TWFmk7vyuzf_g
Ricardo Ferreira

1 Geometria 1 — Produto escalar e comparação


de planos e retas
O produto escalar está intimamente relacionado com ângulos (de facto, é
como se fosse o “ângulo a três dimensões”). Nesta ficha, utilizamos o con-
ceito de produto escalar e de vetores colineares para calcular ângulos e com-
parar retas e planos.

1.1 Colineares ou não colineares?


Definição. Dois vetores →

u e→

v são colineares se existir um número real
k tal que


u = k→ −
v.

Neste capı́tulo, aplicamos o conceito de vetores colineares em diferentes


situações.

(1) (a) Mostra que os vetores com coordenadas (2, 10, 6) e (1, 5, 3) são coli-
neares.
(b) Mostra que os vetores com coordenadas (6, 9, 5) e (2, 3, 1) são não
colineares.
(c) Os vetores com coordenadas (7, 9, 5) e (−7, −9, −5) são colineares?
Justifica.

(2) Sejam r, s e t, respetivamente, as retas de equação

r : (x, y, z) = (18, 4, 8) + k(−6, 18, 21),


s : (x, y, z) = (9, 4, 4) + k(−4, 12, 14)
t : (x, y, z) = (18, 4, 8) + k(−2, 3, 7).

(a) Mostra que as retas r e s são paralelas.


(b) As retas s e t são paralelas? Justifica.
(c) Escreve uma equação vetorial da reta que é paralela a t e que passa
pelo ponto com coordenadas (3, 6, 9).

(3) Sejam α, β e γ, respetivamente, os planos de equação

α : 2x + 5y − 7z = 0,
β : −4x − 10y + 14z + 29 = 0
γ : −2x − 5y + 7 = 0

(a) Mostra que os planos α e β são paralelos.


(b) Os planos β e γ são paralelos? Justifica.

Erro ou sugestão? Contacta-me: mailto:ricardoferreira.contactar@gmail.com 5


Vê no YouTube: https://www.youtube.com/channel/UCkRcdeyQ50TWFmk7vyuzf_g
Ricardo Ferreira

(c) Escreve a equação reduzida do plano paralelo a γ e que passa pelo


ponto com coordenadas (3, 2, 1).

(4) Considera as retas r e s e o plano α, cujas equações são, respetivamente,

r : (x, y, z) = (0, 0, 0) + k(−2, 8, −6),


s : (x, y, z) = (9, 4, 4) + k(−4, 16, −9)
α : 3x − 12y + 9z − 7 = 0

(a) Mostra que a reta r é perpendicular ao plano α.


(b) A reta s é perpendicular ao plano α? Justifica.
(c) Escreve a equação vetorial de uma reta perpendicular a α e que
passa pelo ponto com coordenadas (−1, 4, 3).

1.2 Vetor perpendicular a dois vetores não colineares


Por vezes, é necessário encontrar um vetor que é simultaneamente perpen-
dicular a dois vetores não colineares. Para isso, nós resolvemos um sistema
de equações


n ·→−
(
u =0

− →

n · v =0
em que os vetores → −
u e →
−v são os dois vetores não colineares e →

n o vetor
que queremos descobrir. O principal exemplo é quando temos dois vetores
paralelos ao plano e queremos descobrir o vetor normal ao plano. [Vê uma
breve explicação em https://youtu.be/QrorS3D8ehw?t=3881]

(1) Os vetores de coordenadas (4, 5, −5) e (2, 5, −7) são paralelos ao plano
α (e são não colineares). Determina as coordenadas de um vetor normal
ao plano α.

(2) Os pontos de coordenadas (0, 2, 1), (2, 7, −6) e (2, 3, −8) pertencem ao
plano β (e são não colineares). Determina as coordenadas de um vetor
normal ao plano β.

Erro ou sugestão? Contacta-me: mailto:ricardoferreira.contactar@gmail.com 6


Vê no YouTube: https://www.youtube.com/channel/UCkRcdeyQ50TWFmk7vyuzf_g
Ricardo Ferreira

(3) Considera um paralelepı́pedo retângulo [ABCDEF GH].

−−→ −→
(a) Supõe que os vetores AB e AC têm coordenadas (1, −5, −7) e
(1, 3, 5), respetivamente e que o ponto A tem coordenadas (0, 0, 1).
Determina a equação reduzida do plano ABC.
−→ −−→
(b) Supõe agora que os vetores AE e AD têm coordenadas (1, −5, −7) e
(1, 3, 5), respetivamente e que o ponto B tem coordenadas (−1, 2, 1).
Determina a equação reduzida do plano BCF .
−→
(c) Supõe agora que o vetor AC tem coordenadas (7, 3, 4), e que os
pontos C e D têm coordenadas (3, 6, 7) e (2, 3, 5), respetivamente.
Determina a equação vetorial da reta DH.

1.3 Vetor perpendicular a outro


Dois vetores →

u e→−v são perpendiculares se e só se →−
u ·→−
v = 0. Por isso, se
tivermos um vetor de coordenadas (a, b, c), então as coordenadas (x, y, z) do
vetor perpendicular têm de satisfazer a equação

ax + by + cz = 0.

Neste caso, se a e b fossem diferentes de zero, então (b, −a, 0) seria um vetor
perpendicular a (a, b, c). [Vê a estratégia geral a duas e três dimensões em
https://youtu.be/QrorS3D8ehw?t=3330].

Erro ou sugestão? Contacta-me: mailto:ricardoferreira.contactar@gmail.com 7


Vê no YouTube: https://www.youtube.com/channel/UCkRcdeyQ50TWFmk7vyuzf_g
Ricardo Ferreira

(1) Determina as coordenadas de um vetor que seja perpendicular ao vetor


de coordenadas:

(a) (2, 4)
(b) (−3, 0)
(c) (1, 4, 7)
(d) (2, 0, −5)
(e) (0, 0, 4)

(2) Consider o vetor →


−u de coordenadas (3, −3, 0). Determina as coorde-
nadas de dois vetores →
−v e→

w tal que os três vetores →

u,→

v e→
−w sejam
perpendiculares entre si.

(3) Considera o plano α, cuja equação reduzida é

3x − 5y + 7z + 9 = 0.

(a) Mostra que a reta cujo vetor diretor tem coordenadas (4, 1, −1) é
paralela ao plano α.
(b) Escreve a equação vetorial de duas retas s e t que não sejam para-
lelas entre si e que sejam paralelas ao plano α.
(c) Mostra que o plano de equação 5x + 3y − 9 = 0 é perpendicular ao
plano α.
(d) Determina a equação reduzida de dois planos β e γ que não são
paralelos entre si e que são perpendiculares ao plano α.

(4) Considera a reta r, cuja equação vetorial é

(x, y, z) = (4, 3, 7) + k(3, −3, 0).

(a) Determina as coordenadas de dois vetores não colineares →



u e →

v
que sejam perpendiculares ao vetor diretor de r.
(b) Determina a equação reduzida de dois planos paralelos à reta r e
que não sejam paralelos entre si.
(c) Determina a equação reduzida de dois planos que contêm a reta r
e que não sejam paralelos entre si.
(d) Sejam →−
u e→ −v dois vetores paralelos a um plano β, e B um ponto
desse plano. Com base no exercı́cio anterior, justifica porque é
que se os vetores →

u e→ −v forem colineares, então não conseguimos
determinar a equação reduzida do plano β.

Erro ou sugestão? Contacta-me: mailto:ricardoferreira.contactar@gmail.com 8


Vê no YouTube: https://www.youtube.com/channel/UCkRcdeyQ50TWFmk7vyuzf_g
Ricardo Ferreira

1.4 Produto escalar e ângulos


A grande importância do produto escalar é que para encontrar ângulos no
espaço (difı́cil!), só precisas de saber as coordenadas dos vetores (fácil!)
[aprende esta matéria no capı́tulo “Produto Escalar”em https://youtu.
be/QrorS3D8ehw?t=2628]

(1) Sejam →

u e→ −
v os vetores de coordenadas (u1 , u2 , u3 ) e (v1 , v2 , v3 ), respe-
tivamente.

(a) Escreve as duas fórmulas do produto escalar →



u ·→

v (uma usando as
coordenadas dos vetores, outra usando o ângulo entre eles →

u,→
\ −
v ).
(b) Usando as duas fórmulas anteriores, escreve uma fórmula para o
ângulo →

u,→
\ −
v em função das coordenadas e das normas dos vetores.

(2) Considere a pirâmide quadrangular [ABCDE].

−−→ −→
(a) Supõe que os vetores AB e AC têm coordenadas (1, 4, −3) e (5, 1, 0),
respetivamente. Calcula B ÂC, apresentando o resultado em radia-
nos.
(b) Supõe agora que os pontos A, B e C têm, respetivamente, coorde-
nadas (3, 1, 7), (4, 7, 0) e (2, −5, 0). Calcula AB̂C, apresentando o
resultado em radianos.
−−→ −→
(c) Supõe agora que os vetores AB e AC têm coordenadas (1, 4, −3) e
−−→ −→
(5, 1, 0), respetivamente. Calcula o produto escalar BC · AC e, com
esse resultado, calcula B ÂC, apresentando o resultado em radianos.
(Verifica a tua resposta comparando com o (a)).
−−→ −→
(d) Porque é que o produto escalar AB · AC não pode ser um número
negativo?

Erro ou sugestão? Contacta-me: mailto:ricardoferreira.contactar@gmail.com 9


Vê no YouTube: https://www.youtube.com/channel/UCkRcdeyQ50TWFmk7vyuzf_g
Ricardo Ferreira

(3) O ângulo α entre duas retas r e s é, por definição, o menor ângulo entre
as duas retas.

Sendo assim, α será sempre um ângulo agudo ou reto, ou seja, sabemos


que α ∈ [0, π/2]

(a) Seja r e s as retas cujos vetores diretores têm coordenadas (6, 7, −3)
e (2, 5, 7), respetivamente. Seja α o ângulo entre as retas r e s.
Determina, em radianos, a amplitude de α.
(b) Agora, sejam r e s as retas de equação

r : (x, y, z) = (9, 0, 0) + k(6, 7, −3),


s : (x, y, z) = (9, 4, 4) + k(4, −5, 4),

e seja α o ângulo entre elas. Determina, em radianos, a amplitude


de α.
(c) Duas retas r e s intersetam-se num único ponto — o ponto A, de
coordenadas (6, 3, 2). Sabe-se ainda que o ponto B, de coordena-
das (0, −4, 5), pertence à reta r e que o ponto C, de coordenadas
(4, 7, −5), pertence à reta s. Seja α o ângulo entre as retas r e s.
Determina, em radianos, a amplitude de α.

Erro ou sugestão? Contacta-me: mailto:ricardoferreira.contactar@gmail.com 10


Vê no YouTube: https://www.youtube.com/channel/UCkRcdeyQ50TWFmk7vyuzf_g
Ricardo Ferreira

2 Geometria 2 — Lugares Geométricos


2.1 Equações e interseções de retas e planos
Por um lado, os pontos (x, y, z) da reta são os que satisfazem a equação

(x, y, z) = (a, b, c) + k(d, e, f ),

o que na prática é resolver o sistema



x = a + kd

y = b + ke

z = c + kf

em que a incógnita é k. Efetivamente, os pontos (x, y, z) da reta são os


pontos de coordenadas gerais

(a + kd, b + ke, c + kf ),

em que k é um número real qualquer.


Por outro lado, os pontos (x, y, z) do plano são as soluções da equação
ax + by + cz + d = 0. Para saberes de onde vêm estas equações, vê:

ˆ https://youtu.be/QrorS3D8ehw?t=2107 (Equação vetorial da reta)

ˆ https://youtu.be/QrorS3D8ehw?t=3491 (Equação do plano)

(1) (a) Escreve a equação vetorial da reta que contém o ponto de coorde-
nadas (1, 2, 3) e cujo vetor diretor é (4, 5, 6).
(b) Escreve a equação vetorial da reta que contém os pontos de coorde-
nadas (1, 2, 3) e (4, 5, 6).
(c) Seja r a reta de equação (x, y, z) = (−1, 7, 5) + k(0, 1, 0). Determina
as coordenadas de três pontos da reta e de dois vetores diretores da
reta.
(d) Determina a equação vetorial da reta s que é a interseção dos planos
x = −1 e z = 5. Compara o vetor diretor que obtiveste com os
vetores normais dos dois planos — o que concluis? Justifica esse
facto.

(2) Considera a reta r, cuja equação vetorial é

r : (x, y, z) = (2, −5, 7) + k(−2, 8, 0).

(a) Mostra que o ponto de coordenadas (−1, 7, 7) pertence à reta r.


(b) Mostra que o ponto de coordenadas (−2, 11, 9) não pertence à reta
r.

Erro ou sugestão? Contacta-me: mailto:ricardoferreira.contactar@gmail.com 11


Vê no YouTube: https://www.youtube.com/channel/UCkRcdeyQ50TWFmk7vyuzf_g
Ricardo Ferreira

(c) Sem fazer os cálculos, justifica porque é que o ponto (1, 2, 3) não
pertence à reta r.
(d) Escreve as coordenadas de três pontos colineares que não pertencem
à reta r.
(e) Escreve as coordenadas de dois pontos que pertencem à reta r.

(3) Considera o plano α, cuja equação reduzida é

6x + 3y − 7z + 2 = 0.

(a) Mostra que o ponto de coordenadas (−1, −1, −1) pertence ao plano
α.
(b) Mostra que o ponto de coordenadas (4, 5, −1) não pertence ao plano
α.
(c) Determina a equação reduzida de um plano β, paralelo ao plano α,
tal que o ponto (4, 5, −1) pertença ao plano β.

(4) Considera as retas r, s, t e o plano α, cujas equações são, respetivamente,

r : (x, y, z) = (2, −5, 7) + k(−6, 4, 8),


s : (x, y, z) = (−1, 0, −1) + k(−7, 1, 19),
t : (x, y, z) = (0, 0, 0) + k(−7, 1, 19),
α : 3x + 2y + z = 0.

(a) A reta r interseta o plano α num único ponto. Determina as coor-


denadas desse ponto.
(b) Pelo mesmo processo de substituir as coordenadas gerais de um
ponto da reta na equação do plano, mostra que a reta s não inter-
seta o plano α.
(c) Comparando o vetor diretor de s com o vetor normal a α, e con-
siderando os pontos da reta s, mostra novamente que a reta s não
interseta o plano α.
(d) Pelo mesmo processo de substituir as coordenadas gerais de um
ponto da reta na equação do plano, conclui que a reta t está contida
no plano α.
(e) Comparando o vetor diretor de t com o vetor normal a α, e con-
siderando os pontos da reta t, mostra novamente que a reta t está
contida no plano α.

Erro ou sugestão? Contacta-me: mailto:ricardoferreira.contactar@gmail.com 12


Vê no YouTube: https://www.youtube.com/channel/UCkRcdeyQ50TWFmk7vyuzf_g
Ricardo Ferreira

2.2 Superfı́cie Esférica


A seguir às retas e aos planos, a superfı́cie esférica é o lugar geométrico mais
importante. A sua equação reduzida é

(x − a)2 + (y − b)2 + (z − c)2 = r2 ,

em que (a, b, c) são as coordenadas do centro e r o raio da superfı́cie esférica.


Vê as duas maneiras de definir circunferências/superfı́cies esféricas em

ˆ https://youtu.be/QrorS3D8ehw?t=596 (distância)

ˆ https://youtu.be/QrorS3D8ehw?t=3457 (produto escalar)

(1) Determina a equação reduzida da superfı́cie esférica com centro em


(0, −6, 4) e raio 4.

(2) Determina a equação reduzida da superfı́cie esférica com centro em


(−4, 7, 2) e que contém o ponto (−2, 6, 3).

(3) Determina a equação reduzida da superfı́cie esférica com centro em


(3, 2, 1) e cujo volume da respetiva esfera é 36π.

(4) Determina a equação reduzida da superfı́cie esférica com centro em


(13, −9, 3) e cuja área é 36π.

(5) Considere os pontos A, B de coordenadas (3, 7, 1) e (−1, 3, 5). Determine


a equação reduzida da superfı́cie esférica que contém os pontos A e B,
sabendo que o segmento [AB] é um diâmetro dessa superfı́cie. Resolve
isto das duas maneiras:

(a) primeiro, usando o produto escalar [vê um exemplo em https://


youtu.be/mW9tClygTGs?t=748]
(b) depois, determinando as coordenadas do centro e o raio. Compara
esta com a tua primeira resposta.

2.3 Plano Tangente à Superfı́cie Esférica


O plano tangente à superfı́cie esférica em A é o plano que interseta a su-
perfı́cie esférica num só ponto — o ponto A
Este plano só se define através do produto escalar — vê a definição em

ˆ https://youtu.be/QrorS3D8ehw?t=3424

Erro ou sugestão? Contacta-me: mailto:ricardoferreira.contactar@gmail.com 13


Vê no YouTube: https://www.youtube.com/channel/UCkRcdeyQ50TWFmk7vyuzf_g
Ricardo Ferreira

(1) Considere a superfı́cie esférica de equação

x2 + y 2 + (z − 1)2 = 1

Primeiro, sem fazer cálculos e pensando apenas geometricamente, de-


termina a equação reduzida do plano tangente à superfı́cie esférica no
ponto A, de coordenadas:

(a) (0, 0, 0)
(b) (0, 0, 2)
(c) (0, 1, 1)

Agora, chamemos de C o centro da superfı́cie esférica. Para cada res-


posta acima, compara o vetor normal a esse plano com o respetivo vetor
−→
CA — o que concluis?

(2) Sejam A e B os pontos de coordenadas (4, 7, 1) e (−8, 5, −3). Considere


a superfı́cie esférica que contém os pontos A e B de modo que o seg-
mento [AB] seja um diâmetro. Determina a equação reduzida do plano
tangente a essa superfı́cie esférica em A.

(3) Considere uma certa superfı́cie esférica com centro no ponto C, de co-
ordenadas (1, 4, 3). Sabe-se que A é um ponto dessa superfı́cie esférica e
que pertence ao semieixo positivo Ox. Sabe-se ainda que o ponto P de
coordenadas (2, 0, −2) pertence ao plano tangente à superfı́cie esférica
em A. Determina as coordenadas de A.

2.4 Plano Mediador


O plano mediador de dois pontos A e B pode ser definido de duas maneiras
— uma através de distância, outra através do produto escalar. Para uma
explicação a duas dimensões (mediatriz), vê
ˆ https://youtu.be/QrorS3D8ehw?t=549 (distância)

ˆ https://youtu.be/QrorS3D8ehw?t=3393 (produto escalar)

(1) Primeiro, sem fazer cálculos e pensando apenas geometricamente, de-


termina a equação reduzida do plano mediador de A e B, em que as
coordenadas de A e B são respetivamente:

(a) (0, 0, −1) e (0, 0, 1),


(b) (4, 0, 0) e (2, 0, 0),
(c) (1, −1, 0) e (−1, 1, 0).

Agora, para cada resposta acima, compara o vetor normal a esse plano
−−→
com o respetivo vetor AB — o que concluis?

Erro ou sugestão? Contacta-me: mailto:ricardoferreira.contactar@gmail.com 14


Vê no YouTube: https://www.youtube.com/channel/UCkRcdeyQ50TWFmk7vyuzf_g
Ricardo Ferreira

(2) Sejam A e B os pontos de coordenadas (0, 5, −4) e (−2, −7, 6), respeti-
vamente. Determina a equação reduzida do plano mediador de A e B.
Resolve este exercı́cio das duas maneiras:

(a) primeiro, usando a distância


(b) depois, determinando as coordenadas do ponto médio e usando o
produto escalar. Compara esta com a tua primeira resposta.

(3) Na figura estão duas pirâmides [ABCDE] e [BCDEF ], que têm a


mesma base e a mesma altura.

(a) Supõe que os pontos A e F têm coordenadas (7, 2 − 5) e (3, −6, 9).
Determina a equação reduzida do plano BCD.
−→
(b) Supõe agora que o vetor AF tem coordenadas (2, −4, 6) e que o
ponto B tem coordenadas (0, 5, 5). Determina a equação reduzida
do plano BCD.

Erro ou sugestão? Contacta-me: mailto:ricardoferreira.contactar@gmail.com 15


Vê no YouTube: https://www.youtube.com/channel/UCkRcdeyQ50TWFmk7vyuzf_g
Ricardo Ferreira

3 Trigonometria 1 — Definições e Equações


3.1 Trigonometria em ]0, π/2[
Inicialmente, definimos as razões trigonométricas para triângulos retângulos.
Sejam o, a, h os lados oposto, adjacente e hipotenusa como na figura,

nós definimos
o a o
sen α = , cos α = , tg α = .
h h a
Com isto vemos imediatamente duas propriedades importantı́ssimas. Pri-
meiro,
sen α
tg α = .
cos α
E depois, pelo teorema de Pitágoras, temos o2 + a2 = h2 , ou dividindo tudo
por h2 , obtemos
sen2 α + cos2 α = 1.

(1) Para trabalhar com a trigonometria, é essencial sabermos as razões tri-


gonométricas dos ângulos π/6, π/4 e π/3. Neste exercı́cio, percebemos
a origem destes valores.

π/6 π/4 π/3


√ √
sen 1/2 2/2 3/2
√ √
cos 3/2 2/2 1/2
√ √
tg 1/ 3 1 3

Erro ou sugestão? Contacta-me: mailto:ricardoferreira.contactar@gmail.com 16


Vê no YouTube: https://www.youtube.com/channel/UCkRcdeyQ50TWFmk7vyuzf_g
Ricardo Ferreira

(a) Considera o seguinte triângulo retângulo, cuja hipotenusa é 1 e um


dos ângulos tem amplitude π/4 radianos.

Considerando os ângulos do triângulo, mostra que os catetos têm


igual comprimento. Através do Teorema de Pitágoras, conclui que
π   π  √2
sen = cos = .
2 2 2

(b) Considera o triângulo retângulo à esquerda da figura, cuja hipote-


nusa é 1 e um dos ângulos tem amplitude π/3 radianos.

Colocando outro triângulo igual ao primeiro à direita deste, como


na figura, nós obtemos um triângulo maior, em que dois dos seus
lados são iguais a 1 (as hipotenusas dos triângulos mais pequenos).
Considerando os ângulos do triângulo maior, mostra que o triângulo
maior é equilátero. Deduz que cos(π/3) = √ 1/2. E através do Teo-
rema de Pitágoras, conclui que sen(π/3) = 3/2.

Erro ou sugestão? Contacta-me: mailto:ricardoferreira.contactar@gmail.com 17


Vê no YouTube: https://www.youtube.com/channel/UCkRcdeyQ50TWFmk7vyuzf_g
Ricardo Ferreira

(c) Considera o seguinte triângulo retângulo, cuja hipotenusa é 1 e um


dos ângulos tem amplitude π/6 radianos.

Seguindo
√ o mesmo processo que em (b), mostra que cos(π/6) =
3/2. Com isto, conclui que sen(π/6) = 1/2.
(d) Com base nos resultados obtidos em (a), (b) e (c), calcula
π  π  π 
tg , tg , tg .
6 4 3

CONCLUSÃO: Podemos saber todas as razões trigonométricas de


π/6, π/4, π/3 sem memorizar a tabela — basta pensar no que há de tão
especial sobre os triângulos retângulos com ângulos de π/6, π/4 ou π/3
radianos.

3.2 Trigonometria em R
Infelizmente, a definição anterior de sen α, cos α, tg α é muito limitada, pois
só está definida para ângulos agudos α ∈]0, π/2[.
Mas é aqui que nos lembramos da propriedade fundamental:

sen2 α + cos2 α = 1.

Por outras palavras, o ponto (cos α, sen α) é uma solução da equação

x2 + y 2 = 1

para qualquer ângulo agudo α ∈]0, π/2[.

Erro ou sugestão? Contacta-me: mailto:ricardoferreira.contactar@gmail.com 18


Vê no YouTube: https://www.youtube.com/channel/UCkRcdeyQ50TWFmk7vyuzf_g
Ricardo Ferreira

Assim, as nossas soluções (cos α, sen α) formam um quarto de circun-


ferência. Mas ora, as soluções reais (x, y) da equação x2 + y 2 = 1 formam
uma circunferência inteira!
Isto leva-nos a pensar: e se nós considerássemos qualquer ponto (x, y)
dessa circunferência e chamássemos as suas coordenadas de (cos α, sen α),
para o respetivo ângulo α?

É assim que definimos cos α e sen α para todos os ângulos reais α ∈ R —


são, respetivamente, a abcissa e a ordenada do ponto da circunferência onde
o ângulo é α. No fundo, saber cos α e sen α é saber as abcissas e ordenadas
dos pontos da circunferência - daı́ o nome “circunferência trigonométrica”,
porque toda a trigonometria está nesta circunferência.
E agora que já temos sen α e cos α para todos os ângulos reais, podemos,
a partir destas duas funções, definir tg α para todos os reais:
sen α
tg α = .
cos α
Ou seja, aquilo que antes era uma propriedade da tg α (para ângulos α
agudos), torna-se agora na própria definição de tg α (para ângulos α reais).
Curiosamente, com esta definição, tg α é o declive da reta correspondente
a α.

Erro ou sugestão? Contacta-me: mailto:ricardoferreira.contactar@gmail.com 19


Vê no YouTube: https://www.youtube.com/channel/UCkRcdeyQ50TWFmk7vyuzf_g
Ricardo Ferreira

Assim, também podemos visualizar a tangente na circunferência trigo-


nométrica.

Para ângulos agudos, basta fazer uma semelhança de triângulos; ou para


ângulos reais, basta pensar no significado de declive — “Quantos passos
ando na direção y se andar 1 passo na direção x?” Isto é o mesmo que
perguntar onde é que a reta respetiva ao ângulo α interseta a reta x = 1. E
por isso, tg α será a ordenada dessa interseção.

(1) Considera a circunferência de raio r > 0 centrada na origem.

(a) Escreve as coordenadas de um ponto qualquer desta circunferência


em função de sen α e cos α, em que α é o ângulo respetivo a esse
ponto.
(b) Considera o triângulo retângulo da figura seguinte. Através da se-
melhança de triângulos, confirma a tua resposta em (a).

Erro ou sugestão? Contacta-me: mailto:ricardoferreira.contactar@gmail.com 20


Vê no YouTube: https://www.youtube.com/channel/UCkRcdeyQ50TWFmk7vyuzf_g
Ricardo Ferreira

(c) Os pontos desta circunferência de raio r são as soluções (x, y) de


uma certa equação — qual? Com isto, convence-te da tua resposta
em (a).

(2) Considera a circunferência de raio r > 0 centrada na origem. Escreve


a ordenada do ponto de interseção da reta respetiva a α com a reta
x = 1, em função de tg α. Demonstra o teu resultado através de uma
semelhança de triângulos.

CONCLUSÃO: Não esquecer o raio!!! (A semelhança de triângulos


dá-nos uma demonstração completa. Ver a equação da circunferência não é
uma demonstração completa mas ajuda-nos a lembrar).

3.3 Redução ao primeiro quadrante


Com esta nova definição de sen α, cos α, tg α, nós podemos pegar nas razões
trigonométricas dos ângulos π/6, π/4, π/3 e ficar a saber outros ângulos
para os quais sen α, cos α, tg α têm valores especiais — basta abusarmos da
simetria da circunferência.
A esta técnica de deduzir seno, cosseno e tangente de ângulos não agu-
dos a partir do seno, cosseno e tangente de ângulos agudos chamamos de
“redução ao primeiro quadrante”. Mas, no fundo, é só olhar para a cirfun-
ferência trigonométrica.
Vê todas as técnicas em https://youtu.be/7FGwHqXpmBg?t=1287.

Erro ou sugestão? Contacta-me: mailto:ricardoferreira.contactar@gmail.com 21


Vê no YouTube: https://www.youtube.com/channel/UCkRcdeyQ50TWFmk7vyuzf_g
Ricardo Ferreira

(1) Olhando para a circunferência trigonométrica e, sabendo o seno, cosseno


e tangente dos ângulos π/6, π/4, π/3, calcula:

(a) sen(3π/4) (i) cos(2π/3)


(b) sen(5π/4) (j) cos(3π/6)
(c) sen(−π/4) (k) tg(3π/4)
(d) sen(11π/4) (l) tg(5π/4)
(e) cos(5π/6) (m) tg(4π/3)
(f) cos(7π/6) (n) tg(7π/6)
(g) sen(11π/6) (o) tg(5π/6)
(h) sen(9π/6) (p) tg(−13π/3)

(2) Seja α um ângulo real qualquer. Com base no exercı́cio anterior e


olhando para a circunferência trigonométrica, escreve o seno, cosseno
e tangente de π − α, π + α e −α em função de sen α, cos α ou tg α.

(3) Seja α um ângulo real qualquer.

(a) Olhando para a circunferência trigonométrica, escreve o seno, cos-


seno e tangente de π/2 − α em função de sen α, cos α ou tg α [Este
é o resultado mais importante, porque o usamos nas equações tri-
gonométricas].
(b) Considera um triângulo retângulo tal que um dos seus ângulos é
α. Considerando os dois ângulos agudos desse triângulo, demonstra
mais uma vez a tua resposta em (a) [Esta interpretação só funciona
para ângulos α agudos, mas pode ajudar a lembrar].
(c) Olhando para circunferência trigonométrica, escreve cos(π/2 + α),
sen(−π/2−α) e tg(−π/2+α) em função de sen α, cos α ou tg α. Ve-
rifica a tua resposta combinando os resultados obtidos no exercı́cio
2 e 3a.

CONCLUSÃO: Não precisamos memorizar nada — é só olhar para a


circunferência trigonométrica. O ângulo mais especial é π/2 − α porque nos
ajuda a passar senos a cossenos e vice-versa, o que dará muito jeito para
resolvermos equações.

Erro ou sugestão? Contacta-me: mailto:ricardoferreira.contactar@gmail.com 22


Vê no YouTube: https://www.youtube.com/channel/UCkRcdeyQ50TWFmk7vyuzf_g
Ricardo Ferreira

3.4 Equações Trigonométricas


Para resolver equações trigonométricas:

1. Escrevemos tudo na mesma lı́ngua, ou seja, sen(· · · ) = sen(· · · ) ou


cos(· · · ) = cos(· · · ) ou tg(· · · ) = tg(· · · ) (aqui usamos principalmente
o truque de trocar entre sen e cos através do ângulo π/2 − α)

2. Olhando para a circuferência trigonométrica, escrevemos as soluções


gerais [Vê https://youtu.be/7FGwHqXpmBg?t=3512]

Se a questão só pedir soluções num determinado domı́nio, então depois temos
de experimentar os vários valores de k ∈ Z que funcionam.
Vê mais exemplos de equações (zeros e máximos/mı́nimos) em https:
//youtu.be/7FGwHqXpmBg?t=1637.

(1) Olhando para a circunferência trigonométrica, determina os números


reais que são soluções das equações:

(a) sen x = 0 (j) cos x = − 3/2
(b) cos x = 0 (k) cos x = cos(−π/521)
(c) tg x = 0 (l) tg x = 1

(d) sen x = 1 (m) tg x = 1/ 3

(e) cos x = −1 (n) tg x = − 3
(f) sen x = 1/2 (o) tg x = tg(π/33)

(g) sen x = − 3/2 (p) sen x = cos(π/7)
(h) sen x = sen(5π/12) (q) sen x = cos(4π/7)
(i) cos x = 1/2 (r) cos x = sen (5π/6)

(2) Determina os números reais que são soluções da equação sen(2x) = cos x.
Resolve este exercı́cio de três maneiras:

ˆ Primeiro, colocando a expressão na forma sen(· · · ) = sen(· · · );


ˆ Segundo, colocando a expressão na forma cos(· · · ) = cos(· · · );
ˆ Por fim, utilizando a fórmula sen (2x) = 2 sen x cos x.

(3) Determina os números reais que pertencem ao intervalo − π2 , π2 e que


 

são soluções das equações:

(a) sen x = cos(2x)


(b) sen (x + π/4) = cos(3x)
(c) sen (2x − π/2) = cos(3x − π/3)

Erro ou sugestão? Contacta-me: mailto:ricardoferreira.contactar@gmail.com 23


Vê no YouTube: https://www.youtube.com/channel/UCkRcdeyQ50TWFmk7vyuzf_g
Ricardo Ferreira


(4) Seja f (x) = sen x e g(x) = 3 cos x. Determina os valores reais de x
onde os gráficos das funções f e g se intersetam.

CONCLUSÃO: Mais uma vez, não precisamos memorizar nada — é


só olhar para a circunferência trigonométrica (e se for preciso, trocar senos
e cossenos através do π/2 − α).

Erro ou sugestão? Contacta-me: mailto:ricardoferreira.contactar@gmail.com 24


Vê no YouTube: https://www.youtube.com/channel/UCkRcdeyQ50TWFmk7vyuzf_g
Ricardo Ferreira

4 Trigonometria 2 – Tangente e Declive


4.1 Visualizando a tangente
Considera o ponto de interseção da reta de inclinação α com a reta x = 1.
A tangente de α é a ordenada desse ponto.

(1) Considera a equação tg x = sen x.

(a) Pensando geometricamente, esta equação tem soluções reais?


(b) Resolve a equação e confirma a tua resposta em (a).

Erro ou sugestão? Contacta-me: mailto:ricardoferreira.contactar@gmail.com 25


Vê no YouTube: https://www.youtube.com/channel/UCkRcdeyQ50TWFmk7vyuzf_g
Ricardo Ferreira

(2) Considera a circunferência de raio 4 com centro na origem, como na


figura. Seja A um ponto da circunferência situado no terceiro quadrante
tal que a reta OA tenha inclinação π/3 radianos. Sabe-se ainda que B
é o ponto de interseção da reta OA com a reta de equação x = 4 e que
C é o ponto de coordenadas (4, 0).

(a) Escreve as coordenadas do ponto A.


(b) Escreve as coordenadas do ponto B. Qual é o declive da reta OA?

(c) Mostra que a área do triângulo [ABC] é 12 3.

(3) Seja A um ponto da circunferência trigonométrica que está no primeiro


quadrante, como mostra a figura. Sabe-se ainda que o ponto B é a
interseção entre a reta OA e a reta x = 1, tendo coordenadas (1, t).

(a) Escreve uma expressão para a abcissa do ponto A em função de t.


(b) Usando o teu resultado em (a), escreve uma expressão para a orde-
nada do ponto A em função de t.

Erro ou sugestão? Contacta-me: mailto:ricardoferreira.contactar@gmail.com 26


Vê no YouTube: https://www.youtube.com/channel/UCkRcdeyQ50TWFmk7vyuzf_g
Ricardo Ferreira

(c) Seja α um número real qualquer. A equação cos2 α + sen2 α = 1


relaciona os valores de cos α e sen α. Com base nos teus resultados
em (a) e (b), escreve outras duas equações: uma relacionando cos α
e tg α e outra relacionando sen α e tg α.

4.2 Inclinação e Declive


Seja α o ângulo que parte do eixo Ox e que termina numa certa reta r, como
na figura.

A este ângulo α chamamos de inclinação da reta. E o declive desta


reta é igual a tg α. Ou seja, “o declive da reta é igual à tangente da
inclinação da reta”.

(1) Considera a reta que contém o ponto de coordenadas (−1, 0) e cujo


ângulo que parte da reta e termina no eixo Ox tem amplitude π/6
radianos, como mostra a figura.

Determina a equação reduzida dessa reta.

Erro ou sugestão? Contacta-me: mailto:ricardoferreira.contactar@gmail.com 27


Vê no YouTube: https://www.youtube.com/channel/UCkRcdeyQ50TWFmk7vyuzf_g
Ricardo Ferreira

(2) Considera a circunferência de raio 2 com centro na origem, como na


figura. Sejam A e B os pontos do primeiro e terceiro quadrante, respe-
tivamente, tal que o segmento de reta [AB] seja um diâmetro da circun-
ferência. Sabe-se que r é a reta tangente à circunferência no ponto A e
que interseta o eixo Oy no ponto C. Sabe-se ainda que o ângulo AÔC
tem amplitude π/3 radianos.

(a) Calcula o declive da reta r.


(b) Determina a equação reduzida da reta r.

(c) Mostra que a área do triângulo [ABC] é 4 3.

Erro ou sugestão? Contacta-me: mailto:ricardoferreira.contactar@gmail.com 28


Vê no YouTube: https://www.youtube.com/channel/UCkRcdeyQ50TWFmk7vyuzf_g
Ricardo Ferreira

4.3 Declive da reta perpendicular


Se tivermos uma reta cujo declive é m, então o declive da reta perpendicular
a esta será −1/m.
Podemos demonstrar isto usando produto escalar ou trigonometria: vê
as duas demonstrações em https://youtu.be/QrorS3D8ehw?t=3216.

(1) Considera a circunferência de raio 2 com centro na origem O.

(a) Seja A o ponto da circunferência pertencente ao segundo quadrante


tal que a reta OA tenha inclinação 2π/3 radianos. Calcula a equação
reduzida da reta tangente à circunferência no ponto A.
(b) Seja B e C pontos da circunferência tal que o segmento [BC] é um
diâmetro da circunferência. Sabendo que a reta BC tem declive 2,
calcula a equação reduzida da mediatriz entre B e C.

(2) Seja [ABCD] o quadrado inscrito numa circunferência, como na figura.


Sabe-se que A e C têm coordenadas (10, 0) e (2, 6) respetivamente. Sabe-
se ainda que a abcissa de D é menor que a abcissa de B.

(a) Determina a equação reduzida da circunferência.


(b) Determina a equação reduzida da reta BD.
(c) Usando os resultados em (a) e (b), determina as coordenadas do
ponto D.

Erro ou sugestão? Contacta-me: mailto:ricardoferreira.contactar@gmail.com 29


Vê no YouTube: https://www.youtube.com/channel/UCkRcdeyQ50TWFmk7vyuzf_g
Ricardo Ferreira

5 Complexos 1 – Forma Algébrica e Trigonométrica


5.1 Forma Algébrica e Trigonométrica

Nós queremos um sistema de números em que −1 exista. Por outras
palavras, nós queremos um número que elevado ao quadrado dê −1. A esse
número chamamos i. E já sabemos a sua grande propriedade: i2 = −1.
Um número complexo está na forma algébrica se for x + iy, em que x e
y são números reais. Por outro lado,um número complexo está na forma
trigonométrica se for r cos θ + i sen θ , em que r > 0 é um real positivo e
θ é um real qualquer. Mas muitas vezes, abreviamos isto para reiθ .
Para perceberes a origem dos números complexos, bem como as de-
finições da forma algébrica e da trigonométrica, vê https://youtu.be/
dw0GLhVS7hE?t=30

(1) Os números complexos seguintes estão na forma algébrica?

(a) 1 (f) (1 + i)/2


(b) −2i + 5 (g) cos(π/3) + i sen(π/3)
(c) i2
(d) 1/i (h) sen(π/9) + i tg(π/6)

(e) 1/(2 + i) (i) 3 2 + iπ 2

(2) Os números complexos seguintes estão na forma trigonométrica?

(a) 1 (f) −2eiπ/7


(b) i (g) −5eiπ/7
(c) −1 (h) cos(π/3) + i sen(π/3)
(d) 5eiπ/7 (i) e−iπ
(e) −eiπ/7 (j) ei(π+2i)

5.2 Somar e Multiplicar nos complexos


Somar números complexos é como somar vetores no plano:

(a, b) + (c, d) = (a + c, b + d),

ou seja, (a + bi) + (c + di) = (a + c) + (b + d)i.


Mas o que torna os números complexos especiais é que também dão para
multiplicar:
(a, b) × (c, d) = (ac − bd, ad + bc).
Na forma algébrica, isto é uma simples propriedade distributiva:

(a + bi)(c + di) = (ac − bd) + (ad + bc)i.

Erro ou sugestão? Contacta-me: mailto:ricardoferreira.contactar@gmail.com 30


Vê no YouTube: https://www.youtube.com/channel/UCkRcdeyQ50TWFmk7vyuzf_g
Ricardo Ferreira

Mas na forma trigonométrica, temos

(reiθ )(seiφ ) = (rs)ei(θ+φ)

Ou seja, multiplicar por um número complexo é fazer uma rotação por esse
ângulo e multiplicar os módulos. Afinal esta multiplicação aparentemente
estranha tem um significado geométrico muito intuitivo! São estes signifi-
cados geométricos da soma e da multiplicação que fazem com que a forma
algébrica seja melhor para somar, e a trigonométrica para multiplicar.
Para perceberes melhor o significado geométrico que distingue estas duas
formas, vê https://youtu.be/dw0GLhVS7hE?t=411

(1) Escreve os seguintes números complexos na forma algébrica.

(a) (2 − i)(4 + 3i)


(b) (5 − i)(4 − 2i)
(c) (3 − 2i)2
(d) i(2 + 3i)(2 − 3i)

(2) Escreve os seguintes números complexos na forma trigonométrica.

(a) (eiπ/2 )(eiπ/3 )


(b) (eiπ/4 )(e−iπ/6 )
(c) (e−iπ/3 )(2eiπ/7 )
(d) (5e−iπ/5 )(6e−iπ/7 )
(e) (2e−iπ/3 )(3eiπ/3 )
√ √
(3) Seja z = 2 + 2i um número complexo na forma algébrica. A sua
forma trigonométrica é z = 2eiπ/4 .

(a) Considerando a forma trigonométrica de z, em que quadrante está


z5?
(b) Calcula z 3 . Faz isto de duas maneiras:
ˆ na primeira, usa a forma algébrica de z e escreve z 3 na forma
algébrica
ˆ na segunda, usa a forma trigonométrica de z e escreve z 3 na
forma trigonométrica.

Podemos multiplicar números complexos na forma algébrica, mas se fo-


rem potências muito grandes, é muito mais fácil usar a forma trigo-
nométrica.

Erro ou sugestão? Contacta-me: mailto:ricardoferreira.contactar@gmail.com 31


Vê no YouTube: https://www.youtube.com/channel/UCkRcdeyQ50TWFmk7vyuzf_g
Ricardo Ferreira

5.3 Simétrico: multiplicar por −1 (como subtrair)


Já vimos que números como −2eiπ/6 não estão na forma trigonométrica —
o módulo tem de ser positivo. Mas então, sabendo z, como escrevemos −z?
Para descobrir, é só pensar no desenho: −z é o número simétrico a z em
relação à origem.
Vê como escrever −z na forma algébrica e trigonométrica em https:
//youtu.be/dw0GLhVS7hE?t=1919
√ √
(1) Seja z = (1/ 2)+(1/ 2)i um número complexos na forma algébrica. A
sua forma trigonométrica é z = eiπ/4 . Escreve o simétrico −z na forma
algébrica. Faz isto de duas maneiras:

ˆ na primeira, usa a forma algébrica de z


ˆ na segunda, coloca o número −1 na forma trigonométrica e, usando
a forma trigonométrica de z, calcula (−1)z.

Ou seja, na forma algébrica, multiplicar por −1 é aplicar a propriedade


distributiva. Por outro lado, na forma trigonométrica, multiplicar por
−1 é uma rotação de π.

(2) Coloca os seguintes números complexos na forma trigonométrica.

(a) −eiπ/5
(b) −2e−i(4π/7)
(c) −3e−iπ/2

5.4 Potências de i: multiplicar por i


Multiplicar por i é fazer uma rotação de π/2. Percebe como calcular potências
de i em https://youtu.be/dw0GLhVS7hE?t=782

(1) Escreve os números complexos i2 , i3 , i4 na forma algébrica. Faz isto de


duas maneiras:

ˆ na primeira, lembrando que i2 = −1


ˆ na segunda, colocando i na forma trigonométrica, calculando as
potências e colocando esses números na forma algébrica.

No fundo, multiplicar por i é fazer uma rotação de π/2.

Erro ou sugestão? Contacta-me: mailto:ricardoferreira.contactar@gmail.com 32


Vê no YouTube: https://www.youtube.com/channel/UCkRcdeyQ50TWFmk7vyuzf_g
Ricardo Ferreira

(2) Escreve o número complexo i521 na forma algébrica. Faz isto de duas
maneiras:

ˆ escreve i521 na forma i4k+r (com k ∈ Z, r = 0, 1, 2 ou 3) e conclui


usando os resultados em (1);
ˆ escreve i na forma trigonométrica, calcula a potência, e coloca esse
número na forma algébrica.

Ou seja, fazer 4 rotações de π/2 é o mesmo que não fazer nada, por
isso ignoramos essa parte.

(3) Coloca os seguintes números complexos na forma algébrica.

(a) i33
(b) i57
(c) i4042
(d) i−1

5.5 Conjugado (como dividir complexos)


O conjugado é o primeiro truque fundamental para resolver equações, porque
é graças a ele que conseguimos tirar números complexos do denomi-
nador.
Aprende mais sobre o conjugado em:

ˆ https://youtu.be/dw0GLhVS7hE?t=1789 (definição de conjugado e


significado geométrico)

ˆ https://youtu.be/dw0GLhVS7hE?t=2077 (como tirar complexos do


denominador)

(1) Seja z um número complexo.

(a) Mostra que zz = |z|2 . Faz isto de duas maneiras: uma colocando z
na forma algébrica e outra colocando z na forma trigonométrica.
Ou seja, multiplicar um complexo pelo seu conjugado transforma-o
num número real.
(b) Usando o resultado em (a), mostra que
1 z
= .
z |z|2

Ou seja, multiplicar pelo conjugado do denominador em cima e em


baixo transforma o denominador num número real.

Erro ou sugestão? Contacta-me: mailto:ricardoferreira.contactar@gmail.com 33


Vê no YouTube: https://www.youtube.com/channel/UCkRcdeyQ50TWFmk7vyuzf_g
Ricardo Ferreira

(2) Calcula o conjugado dos números complexos seguintes. Se o número


estiver na forma algébrica, dá a tua resposta na forma algébrica. E
se estiver na forma trigonométrica, dá o teu resultado na forma trigo-
nométrica.

(a) 1+i (g) −i


(b) 7 − 2i (h) −1
(c) −6 + 7i
(i) 2ei(π/4)
(d) −3i − 2
(e) sen(π/16) + i cos(π/6) (j) 5e−i(3π/7)
(f) i (k) 3ei(5π/3)

(3) Coloca os seguintes números complexos na forma algébrica.

1 1 1
(a) (b) (c)
1 + 2i 4 − 3i −7 − 2i

4 + 3i 2 − 6i 1
(d) (e) (f)
−1 − i 5−i i5

5.6 Passar algébrica ↔ trigonométrica


Mudar entre forma algébrica e forma trigonométrica é o segundo truque
fundamental para resolver equações, porque permite-nos escolher a forma
mais conveniente para fazer os nossos cálculos. Mudar entre forma algébrica
e trigonométrica é um simples exercı́cio de trigonometria — é só olhar para
o desenho.

Vê a técnica completa em https://youtu.be/dw0GLhVS7hE?t=1635.

Erro ou sugestão? Contacta-me: mailto:ricardoferreira.contactar@gmail.com 34


Vê no YouTube: https://www.youtube.com/channel/UCkRcdeyQ50TWFmk7vyuzf_g
Ricardo Ferreira

(1) Coloca os seguintes números complexos na forma algébrica.

(a) eiπ/2 (e) e−i5π/6


(b) e−iπ/2 (f) ei2π/3
(c) eiπ (g) 2eiπ/7
(d) e−iπ (h) −3eiπ/5

(2) Coloca os números complexos −1 + i e 1 − i na forma trigonométrica.


Resolver tg θ = y/x tem sempre duas soluções — para as distinguir,
temos de ver em que quadrante está o nosso número complexo.

(3) Coloca os seguintes números complexos na forma trigonométrica.



(a) 7 + 7i (d) 2 + 2 3i

(b) −5 − 5i (e) 3i − 3
√ √
(c) −2 − 2 3i (f) −3 3 − 3i

5.7 Complexos e Trigonometria


Por vezes, para colocar na forma trigonométrica, é necessário fazer reduções
ao primeiro quadrante.

(1) Coloca o número complexo cos(8π/7)+i sen(π/7) na forma trigonométrica.


Para isso, resolve o sistema de equações
(
cos x = cos(8π/7)
sen x = sen (π/7)

Ou seja, colocar números complexos do tipo ± cos(· · · ) ± i sen(· · · ) ou


±sen(· · · ) ± i cos(· · · ) na forma trigonométrica é resolver um sistema de
equações trigonométricas.

(2) Coloca os seguintes números complexos na forma trigonométrica. Não


precisas de resolver o sistema explicitamente — basta resolveres o sis-
tema geometricamente, olhando para a circunferência trigonométrica.

(a) cos(π/12) + i sen(π/12)


(b) 4 cos(π/12) + i 4 sen(π/12)
(c) 2 cos(23π/12) + i 2 sen(−π/12)
(d) cos(π/12) + i sen(11π/12)
(e) 3 cos(7π/12) + i 3 sen(5π/12)
(f) cos(−5π/12) + i sen(7π/12)

Erro ou sugestão? Contacta-me: mailto:ricardoferreira.contactar@gmail.com 35


Vê no YouTube: https://www.youtube.com/channel/UCkRcdeyQ50TWFmk7vyuzf_g
Ricardo Ferreira

(g) sen(5π/12) + i sen(π/12)


(h) cos(−π/8) + i cos(5π/8)
(i) cos(π/12) − i sen(−π/12)
(j) − cos(6π/7) + i sen(6π/7)
(k) − cos(5π/13) − i sen(7π/13)
(l) cos(8π/7) − i sen(−π/7)

Ou seja, para resolver o sistema, basta olhar para a circunferência tri-


gonométrica

(3) Coloca o número complexo

−sen(3π/5) + i cos(−2π/5)

na forma trigonométrica. Faz isto de duas maneiras:

ˆ iguala os ângulos primeiro e depois coloca na forma cos(· · · ) +


i sen(· · · )
ˆ reverte a ordem: primeiro coloca na forma ± cos(· · · ) ± i sen(· · · )
e depois iguala os ângulos.

5.8 Recapitulando
(1) Escreve os seguintes números complexos na forma trigonométrica. Faz
isto de duas maneiras:

ˆ uma, fazendo todos os cálculos na forma algébrica e, no fim, pas-


sando para forma trigonométrica
ˆ outra, colocando primeiro todos os números na forma trigonométrica
e depois fazendo os cálculos

(a) 1/(4 + 4i)


√ √
(b) (1 + 3)2 /( 3 − i)i13

Dividir na forma trigonométrica é mais imediato — não precisamos de


conjugados, mas só das propriedades das potências

(2) Sejam z1 = sen(−π/6) + i cos(7π/6), z2 = 3 + i, z3 = 1 − 2i. Escreve
o número complexo
z1 − z26
i7 z3
na forma algébrica.

Nos complexos, o mais importante é saber quando usar a forma algébrica


e quando usar a forma trigonométrica.

Erro ou sugestão? Contacta-me: mailto:ricardoferreira.contactar@gmail.com 36


Vê no YouTube: https://www.youtube.com/channel/UCkRcdeyQ50TWFmk7vyuzf_g
Ricardo Ferreira

6 Complexos 2 – Regiões e Equações


6.1 Regiões no Plano Complexo
Para definir regiões no plano complexo, é preciso inequações. E para definir
inequações, só podemos comparar números reais.
Felizmente, há quatro números reais associados a um número complexo:
a parte real Re(z) e a parte imaginária Im(z) (associados à forma
algébrica) bem como o módulo |z| e o argumento Arg(z), (associados
à forma trigonométrica).
De facto, na forma algébrica, z = Re(z) + i Im(z) e na forma trigo-
nométrica, z = |z| eiArg(z)
Se quisermos usar números complexos diretamente para definir regiões,
podemos, mas nesse caso só através de equações (e não de inequações).
Vê muitos exemplos de regiões no plano complexo em https://youtu.
be/dw0GLhVS7hE?t=917.

6.1.1 Regiões definidas por Re(z) e Im(z)


(1) Sejam a e b números reais tal que a < b, e considera a condição

x ≥ a ∧ x ≤ b.

Reescreve esta condição de três maneiras:

ˆ na forma de “inequação dupla”: · · · ≤ x ≤ · · · ,


ˆ na forma de intervalo: [· · · , · · · ],
ˆ na forma de “distância/cı́rculo nos reais”: |x − · · ·| ≤ · · · .
(Cuidado! Este |· · ·| é o módulo para números reais e não o módulo
para números complexos!)

Podemos reescrever uma “interseção”de inequações de quatro maneiras


diferentes — podemos usar qualquer uma destas para definir regiões no
plano.

Erro ou sugestão? Contacta-me: mailto:ricardoferreira.contactar@gmail.com 37


Vê no YouTube: https://www.youtube.com/channel/UCkRcdeyQ50TWFmk7vyuzf_g
Ricardo Ferreira

(2) Representa, no plano complexo, o conjunto de pontos que satisfazem as


seguintes condições.

(a) Re(z) = 0
(b) Re(z) = 2
(c) Re(z) = −3
(d) Re(z) ≥ −3
(e) Re(z) < −3
(f) Re(z) ≥ 3 ∨ Re(z) ≤ 1
(g) Re(z) ≥ 3 ∧ Re(z) ≤ 1
(h) Re(z) ≥ −3 ∧ Re(z) ≤ 1
(i) Im(z) = 0
(j) Im(z) = −5
(k) Im(z) = 2
(l) Im(z) > 2
(m) Im(z) < 2
(n) |Im(z)| ≤ 2
(o) |Im(z)| ≥ 2
(p) |Im(z) − 1| ≤ 2
(q) |Re(z) + 2| ≤ 3 ∧ Im(z) ∈ [−1, 1]
(r) |Re(z) − 1| ≥ 3 ∧ Im(z) ≥ 2

Re(z) = k é uma reta vertical e Im(z) = k é uma reta horizontal. Na


geometria do plano, acontece o mesmo: x = k é uma reta vertical, y = k
é uma reta horizontal.

(3) Escreve as condições que definem as seguintes regiões no plano complexo.

(a) Terceiro quadrante


(b) Semieixo real positivo
(c) Quadrado com vértices em 1 + 2i, 3 + 2i e 3 + 4i
(d) Retângulo com vértices em 0, 6 e 6 − 4i.

(4) Representa, no plano complexo, o conjunto de pontos que satisfazem as


seguintes condições.

(a) Im(z) = Re(z)


(b) Im(z) = −Re(z)
(c) Im(z) = 2Re(z)

Erro ou sugestão? Contacta-me: mailto:ricardoferreira.contactar@gmail.com 38


Vê no YouTube: https://www.youtube.com/channel/UCkRcdeyQ50TWFmk7vyuzf_g
Ricardo Ferreira

(d) Im(z) = 4Re(z)


(e) Im(z) = 4Re(z) + 2
(f) Im(z) = 4Re(z) − 1
(g) Re(z) = 3Im(z)
(h) (Re(z) − 1)/Im(z) = −2
(i) Im(z) ≥ Re(z) + 2
(j) Im(z) ≤ Re(z) + 2
(k) Re(z) ≥ (1/2)Im(z) + 1
(l) Re(z) ≤ (1/4)Im(z) ∧ Re(z) ≤ −1

Tal como na geometria do plano, podemos definir não só retas verticais
e horizontais, mas também retas com qualquer declive e ordenada na
origem.

(5) Escreve as condições que definem as seguintes regiões no plano complexo.

(a) Triângulo com vértices em 0, 3 e 3 + 3i


(b) Triângulo com vértices em −4 − 3i, 2 − 3i e i.
(c) Paralelogramo com vértices em i, 3i e −5 + 2i.
(d) Pentágono com vértices em 4, 4i, 4i − 4, −8i e −4i + 4.

Cada inequação corresponde a uma linha do polı́gono.

(6) Representa, no plano complexo, o conjunto de pontos que satisfazem as


seguintes condições.

(a) Im(z) = (Re(z))2


(b) Im(z) = (Re(z))3
(c) Im(z) = 1/Re(z)
(d) Im(z) × Re(z) = 1
(e) Im(z) = sen (Re(z))

Tal como na geometria no plano, podemos representar não só qualquer


reta, mas o gráfico de qualquer função — é só substituir x e y por Re(z)
e Im(z).

Erro ou sugestão? Contacta-me: mailto:ricardoferreira.contactar@gmail.com 39


Vê no YouTube: https://www.youtube.com/channel/UCkRcdeyQ50TWFmk7vyuzf_g
Ricardo Ferreira

6.1.2 Regiões definidas por |z| e Arg(z)

(7) Representa, no plano complexo, o conjunto de pontos que satisfazem as


seguintes condições.

(a) |z| = 2 (h) |z − 1 − i| = 1


(b) |z| = 4 (i) |z − 1 + 2i| = 1
(c) |z| = 0
(j) |z| ≤ 1
(d) |z| = −1
(k) |z| > 1
(e) |z − i| = 1
(f) |z − i| = 2 (l) |z + 1 + i| < 2
(g) |z + i| = 1 (m) 1 ≤ |z + 2i| ≤ 2

|z − a| = k é uma circunferência de centro em a e raio k. Mas na


geometria, uma circunferência de centro em A e raio k é definida por
d(P, A) = k. Ou seja, o módulo nos complexos corresponde à distância
na geometria.

(8) Representa, no plano complexo, o conjunto de pontos que satisfazem as


seguintes condições.

(a) Arg(z) = 0
(b) Arg(z) = π
(c) Arg(z) = 5π/4
(d) Arg(z) ≥ 0
(e) Arg(z) < π/2
(f) 7π/4 ≤ Arg(z) ≤ 2π
(g) −π/4 ≤ Arg(z) ≤ 0
(h) −π/4 ≤ Arg(z) ≤ 2π
(i) Arg(3z) ∈ [2π/3, 4π/3]
(j) Arg(z) ∈ [0, π/4] ∧ |z − 1 − i| ≤ 1

Arg(z) = k é uma semirreta que parte da origem e que tem inclinação


k radianos.

Erro ou sugestão? Contacta-me: mailto:ricardoferreira.contactar@gmail.com 40


Vê no YouTube: https://www.youtube.com/channel/UCkRcdeyQ50TWFmk7vyuzf_g
Ricardo Ferreira

6.1.3 Regiões definidas por Re(z), Im(z), |z| e Arg(z)

(9) Escreve as condições que definem os seguintes lugares geométricos. Faz


isto de duas maneiras: uma usando só Re(z) e Im(z), outra usando só
|z| e Arg(z).

(a) Primeiro quadrante


(b) Quarto quadrante
(c) Eixo real
(d) Semieixo imaginário positivo
(e) Bissetriz dos quadrantes ı́mpares
(f) Reta de inclinação α que passa pela origem
(g)

(h)

Os lugares geométricos definidos através de retas que passam pela origem


podem-se escrever tanto através de Re(z) e Im(z), como de |z| e Arg(z).

Erro ou sugestão? Contacta-me: mailto:ricardoferreira.contactar@gmail.com 41


Vê no YouTube: https://www.youtube.com/channel/UCkRcdeyQ50TWFmk7vyuzf_g
Ricardo Ferreira

(10) Seja w = −1 + 3i e z = 2ei(4π/7) .

(a) Mostra que w satisfaz a condição



π 3π
ww = 10 ∧ Arg(w) ∈ , .
2 4

(b) Usando a tua resposta em (a), em que quadrante está o ponto wz?
(c) Mostra que z satisfaz a condição

Im(z) > − 3Re(z) ∧ Re(z) < 0.

Por vezes, a maneira mais fácil de ver se um número complexo sa-


tisfaz certas condições, é substituir essas condições por condições
equivalentes, que se ajustam mais à forma do nosso número com-
plexo.

6.2 Equações
Para resolver equações nos complexos, primeiro simplificamos a equação de
forma a ter um número complexo igual a outro número complexo.
Se os dois complexos estiverem na forma algébrica, ou seja, a+bi = c+di,
então resolvemos o sistema (
a=c
b=d
Por outras palavras, dois números complexos na forma algébrica são iguais
se e só se tiverem a mesma parte real e a mesma parte imaginária.
Se os dois complexos estiverem na forma trigonométrica, ou seja, reiθ =

se , então resolvemos o sistema
(
r=s
θ = φ + 2kπ
Por outra palavras, dois números complexos na forma trigonométrica são
iguais se e só se tiverem o mesmo módulo e o “mesmo”argumento
(incluindo múltiplos de 2π).
Vê exemplos e aprende a estratégia para resolver equações em https:
//youtu.be/dw0GLhVS7hE?t=2316.

(1) Considera a equação z 2 = −iz.


(a) Escreve, na forma algébrica, os números complexos que são soluções
desta equação. Faz isto de duas maneiras: uma colocando z na
forma algébrica, outra colocando z na forma trigonométrica.
(b) Os afixos das soluções não nulas desta equação formam um polı́gono
regular. Determina a área desse polı́gono.

Erro ou sugestão? Contacta-me: mailto:ricardoferreira.contactar@gmail.com 42


Vê no YouTube: https://www.youtube.com/channel/UCkRcdeyQ50TWFmk7vyuzf_g
Ricardo Ferreira

(2) Determina os números complexos que são soluções das seguintes equações.
Dá a tua resposta na forma algébrica ou na forma trigonométrica.

(a) z + iz = 5 (g) z 3 = 4z 2
(b) z 2 = 3iz √ √
(h) ( 2 + i 2)z 5 = z 3
(c) z 2 + iz + 2 = 0
(i) z 4 = 8ei(π/7) z
(d) z 3 = 2iz
(e) z 2 + 2z = 0 (j) Im(z)Re(z) = 0
(f) (z − 2i)2 = −4iz (k) |z|3 = 4z

6.3 Radiciação: a equação especial z n = w


Às soluções da equação z n = w, nós chamamos de raı́zes n-ésimas de w.
Isto porque são os números que elevados a n dão w.
Por exemplo, as raı́zes quadradas de −1 são os números que elevados ao
quadrado dão −1. Ou seja, são as soluções da equação z 2 = −1. Neste caso,
até já sabemos a resposta: as raı́zes quadradas são i e −i.
O que torna as raı́zes n-ésimas tão especiais é que juntas formam os
vértices de um polı́gono regular (com n lados).
Vê como resolver estas equações em https://youtu.be/dw0GLhVS7hE?
t=2511

(1) Determina os números complexos que são soluções das equações seguin-
tes e representa os respetivos afixos no plano complexo.

(a) z 3 = 1
(b) z 3 = −1
(c) z 3 = eiπ/7
(d) z 5 = −1
(e) z 4 = −1
(f) z 4 = 3
(g) z 4 = 7eiπ/4
(h) Observando as tuas respostas e considerando a equação z n = reiθ ,
explica como é que n, r e θ afetam o número de soluções, o módulo
das soluções, e o argumento das soluções.

Resolver as equações z n = w é o mesmo que resolver qualquer outra


equação nos complexos.
(2) Sejam z1 , z2 , z3 e z4 as quatro soluções da equação z 4 = ei(π/3) . Repre-
senta estas soluções no plano complexo e mostra que z1 +z2 +z3 +z4 = 0.
Este é um padrão geral: se somarmos todas as soluções de uma equação
z n = w, então o resultado é sempre 0!

Erro ou sugestão? Contacta-me: mailto:ricardoferreira.contactar@gmail.com 43


Vê no YouTube: https://www.youtube.com/channel/UCkRcdeyQ50TWFmk7vyuzf_g
Ricardo Ferreira

(3) Escreve as equações z n = reiθ cujas soluções são os vértices dos seguintes
polı́gonos regulares centrados na origem.

(a) Triângulo com vértice em 1


(b) Triângulo com vértice em 5
(c) Triângulo com vértice em 3eiπ/4
(d) Quadrado com vértice em 1 + i
(e) Pentágono com vértice em ei(4π/7)
(f) Hexágono com vértice em i

Nas equações z n = reiθ , todas as soluções têm módulo r1/n . A primeira


solução terá argumento θ/n e para chegar às outras soluções, vamos
adicionando múltiplos de 2π/n. Temos assim n soluções, ou seja, um
polı́gono regular com n lados e em que um dos vértices é r1/n eiθ/n .

(4) Seja w uma raı́z quarta de 2eiπ/7 e z uma raı́z cúbica de i. Escreve o
número complexo w4 z 6 na forma trigonométrica.

(5) Considera o número complexo z = k + i, em que k é um número real.

(a) Supondo que z é uma raı́z quadrada de 8 − 6i, determina k.


(b) Supondo que z é uma raı́z quadrada de um número real, determina
k.
(c) Supondo que z é uma raı́z quadrada de um imaginário puro cuja
parte imaginária é positiva, determina k.
(d) Seja w o número complexo que satisfaz a condição
 
−π
Re(w) = 3 ∧ Arg(w) ∈ ,0 .
2
Supondo que z é uma raı́z quadrada de w, determina k.

6.4 Recapitulando
(1) Seja z um número complexo.

(a) Determina o lugar geométrico formado pelas soluções da equação


z = −z. Resolve isto de duas maneiras: uma colocando z na forma
algébrica, outra colocando z na forma trigonométrica.
(b) Mostra que
z+z
= Re(z).
2
(c) Supondo que z pertence ao primeiro quadrante e pensando em z e
z como vetores, representa graficamente a soma (z + z)/2. Verifica
que essa soma correponde a Re(z).

Erro ou sugestão? Contacta-me: mailto:ricardoferreira.contactar@gmail.com 44


Vê no YouTube: https://www.youtube.com/channel/UCkRcdeyQ50TWFmk7vyuzf_g
Ricardo Ferreira

(d) Usando (b), mostra novamente que a equação z = −z define o eixo


imaginário.
(e) Supondo que z é um imaginário puro, verifica geometricamente a
tua resposta em (a) e (d).
(f) Mostra que
z−z
= Im(z).
2i
(g) Supondo que z pertence ao primeiro quadrante e pensando em z e
z como vetores, representa graficamente a soma (z − z)/2i. Verifica
que essa soma correponde a Im(z).
(h) Usando (f), mostra que a equação z = z define o eixo real.
(i) Resolvendo a equação z = z, confirma a tua resposta em (h).
(j) Supondo que z é um número real, verifica geometricamente a tua
resposta em (h) e (i).

Há outra maneira mais matreira de escrever Re(z) e Im(z), através das
fórmulas (z + z)/2 e (z − z)/2i. Com isto, um número complexo é um
imaginário puro se e só se z = −z, e é real se e só se z = z.

(2) Considera a seguinte região no plano complexo, cujos pontos são defini-
dos através de circunferências de centro em 0, em −2 + 2i e 2 + 2i.

(a) Escreve a condição que define esta região.


(b) Mostra que as raı́zes quartas de −64 pertencem a esta região
(c) Considera a equação z 4 = 16eiθ . Determina θ de modo que exata-
mente três soluções desta equação pertençam à região.

Erro ou sugestão? Contacta-me: mailto:ricardoferreira.contactar@gmail.com 45


Vê no YouTube: https://www.youtube.com/channel/UCkRcdeyQ50TWFmk7vyuzf_g
Ricardo Ferreira

7 Sucessões — Progressões, Limites e Proprieda-


des
7.1 Progressões Aritméticas
Uma progressão aritmética é aquela que para do termo anterior, obter o
termo seguinte, é preciso somar sempre pelo mesmo valor.
Aprende como calcular o termo geral e a soma dos N primeiros termos
de uma progressão aritmética em https://youtu.be/DBDuf-mPrQk?t=27

(1) Considera uma progressão aritmética (un ). Sabe-se que u3 = 1 e que


u6 = 10.

(a) Calcula o termo de geral de (un ).


(b) Calcula o valor de u12 .
(c) Sabendo que 43 é termo da sucessão, determina a ordem desse
termo.
(d) Calcula a soma dos dez primeiros termos da sucessão, ou seja, u1 +
u2 + · · · + u10 .
(e) Calcula a soma u5 + u6 + · · · + u10 . Faz isto de duas maneiras:
uma aplicando diretamente a fórmula dos N primeiros termos, outra
calculando a soma u1 + u2 + u3 + u4 e usando o teu resultado em
(d).

(2) Seja (vn ) uma progressão aritmética. Sabe-se que 2v7 = v4 e que a
soma dos seis primeiros termos é igual a 13. Sabendo que 0 é termo da
sucessão, determina a ordem desse termo.

(3) Seja (wn ) uma progressão aritmética. Sabe-se que w1 +w2 +· · ·+w5 = 5
e que w2 + w3 + · · · + w6 = 25. Calcula o termo geral de (wn ). Apresenta
o teu resultado na forma a + bn, em que a e b são números reais.

Erro ou sugestão? Contacta-me: mailto:ricardoferreira.contactar@gmail.com 46


Vê no YouTube: https://www.youtube.com/channel/UCkRcdeyQ50TWFmk7vyuzf_g
Ricardo Ferreira

7.2 Progressões Geométricas


Uma progressão aritmética é aquela que para do termo anterior, obter o
termo seguinte, é preciso multiplicar sempre pelo mesmo valor.
Aprende como calcular o termo geral e a soma dos N primeiros termos
de uma progressão geométrica em https://youtu.be/DBDuf-mPrQk?t=474

(1) Considera uma progressão geométrica (un ). Sabe-se que u3 = 5 e que


u6 = 40.

(a) Calcula o termo de geral de (un ).


(b) Calcula o valor de u11 .
(c) Sabendo que 320 é termo da sucessão, determina a ordem desse
termo.
(d) Calcula a soma dos oito primeiros termos da sucessão, ou seja, u1 +
u2 + · · · + u8 .
(e) Calcula a soma u3 + u4 + · · · + u8 . Faz isto de duas maneiras:
uma aplicando diretamente a fórmula dos N primeiros termos, outra
calculando a soma u1 + u2 e usando o teu resultado em (d).

(2) Seja (vn ) uma progressão geométrica. Sabe-se que 2v1 = v2 + v3 e que
v5 = −48. Sabendo que a sucessão (vn ) não é constante, determina o
termo geral de vn . Apresenta a tua resposta na forma a × bn , em que a
e b são números reais.

(3) Seja (wn ) uma progressão geométrica. Sabe-se que w1 × w2 × w3 = 8


e que w2 × w3 × w4 = 8/27. Sabendo que 0 não é termo da sucessão,
calcula a soma dos três primeiros termos de (wn ).

(4) Sejam a, b números reais diferentes de zero. Sabe-se que a, b, 2 são termos
consecutivos de uma progressão aritmética e que 3, b, a + 2 são termos
consecutivos de uma progressão geométrica. Determina os valores de a
e b.

Erro ou sugestão? Contacta-me: mailto:ricardoferreira.contactar@gmail.com 47


Vê no YouTube: https://www.youtube.com/channel/UCkRcdeyQ50TWFmk7vyuzf_g
Ricardo Ferreira

7.3 Limites de sucessões


Quando uma sucessão tem um limite a, que é um número real, dizemos que
a sucessão converge para a. Se uma sucessão não converge, então dizemos
que ela diverge. E se ela diverge, então há duas opções: ou diverge para
±∞ ou o limite não existe.
Aprende tudo sobre limites de sucessões em

ˆ https://youtu.be/DBDuf-mPrQk?t=1396 (Definição de limite)

ˆ https://youtu.be/DBDuf-mPrQk?t=1840 (Propriedades de limites)

ˆ https://youtu.be/DBDuf-mPrQk?t=2805 (Definição de limite para


±∞)

ˆ https://youtu.be/DBDuf-mPrQk?t=3391 (Truques para calcular li-


mites)

ˆ https://youtu.be/sG0CRKLQjd8?t=1257 (O limite (1 + 1/n)n )

(1) Calcula o limite das sucessões seguintes.

(a) 1/n (l) (−1)n


(b) 1/n3 (m) n3 − n
(c) 3/n3 (n) −2n4 + n
(d) 1/n2 − 2/n3 (o) −n3 + 3n2 − n + 1
(e) (2n + 1)/n (p) (3n − 2)/(6n2 + 1)
(f) n (q) (5n3 + 1)/(3n + 2)
(g) n2 (r) 4n4 /(3n4 − 2n2 )
√ √ √
(h) n (s) n + 1 − n
√ √
(i) 2n (t) n + 1 − n − 1

(j) (3/2)n (u) n2 + 1 − n
√ √
(k) (1/3)n (v) 3n + 1 − 2n − 1

Erro ou sugestão? Contacta-me: mailto:ricardoferreira.contactar@gmail.com 48


Vê no YouTube: https://www.youtube.com/channel/UCkRcdeyQ50TWFmk7vyuzf_g
Ricardo Ferreira

(2) Sabendo que lim(1 + 1/n)n = e, calcula o limite das sucessões seguintes.

1 2n 1 −2n 1 n/5
     
(a) 1 + (b) 1 + (c) 1 +
n n n

1 n+1 1 2n−3
     n
1
(d) 1 + (e) 1 + (f) 1 +
n n 2n

1 n
2n+1
2 n
    
1
(g) 1 − (h) 1 − (i) 1 +
2n 3n n

n
3 2n−1 2 n
    
2
(j) 1 − (k) 1 − (l) 2 +
n n n

 n  n  n
1 1 1 1
(m) − (n) 1 + (o) 1 +
2 2n n+1 2n + 1

 n  n  n
4 n+3 3n + 1
(p) 2 − (q) (r)
3n + 1 n−2 2n − 5

n −2n+1
1 n
   
n−3 −3n + 1
(s) (t) (u) 1 − 2
2n + 1 3n + 1 n

Erro ou sugestão? Contacta-me: mailto:ricardoferreira.contactar@gmail.com 49


Vê no YouTube: https://www.youtube.com/channel/UCkRcdeyQ50TWFmk7vyuzf_g
Ricardo Ferreira

7.4 Propriedades das Sucessões


Há três propriedades importantı́ssimas das sucessões: ser monótona, ser
limitada e ser convergente.
Se uma sucessão é convergente, então ela é limitada. Por outro lado, se
uma sucessão é limitada e monótona, então é convergente.

Convergente ⇒ Limitada
Convergente ⇐ Limitada + Monótona

Ser limitada não é suficiente para garantir convergência, tanto que (−1)n é
limitada, mas não é convergente. De facto, pensando no contrarrecı́proco,
vemos que se uma sucessão é divergente, então ela ou não é limitada (como
no caso de n) ou não é monótona (como no caso de (−1)n ).
Aprende as definições de monotonia, limitação e convergência, bem como
as suas relações em
ˆ https://youtu.be/DBDuf-mPrQk?t=1144 (Sucessão monótona e limi-
tada)
ˆ https://youtu.be/DBDuf-mPrQk?t=1396 (Sucessão convergente)

ˆ https://youtu.be/DBDuf-mPrQk?t=1840 (Relação entre monotonia,


limitação e convergência)

(1) As sucessões (un ) seguintes são monótonas. Verificando se un+1 − un é


positivo ou negativo, determina se as sucessões seguintes são crescentes
ou decrescentes.

(a) n (f) 2n/(n + 1)


(b) −n (g) (−2n + 1)/(2n + 2)
(c) n2
(d) 1/n (h) (n − 2)/(−3n + 1)
√ √
(e) −1/n (i) n + 1 − n

(2) Considera as sucessões (a)–(i) do exercı́cio anterior. Queremos determi-


nar se estas são limitadas.
(a) Calcula o limite de cada sucessão.
Se a sucessão for convergente, sabemos logo que é limitada! Se for
divergente, não sabemos — lembra-te do (−1)n !)
(b) Combinando as tuas respostas em (1) e (2)(a), determina o intervalo
mais pequeno que contém todos os termos de cada sucessão.
Uma sucessão crescente é minorada, e uma sucessão decrescente é
majorada. E se essa sucessão convergir, então o limite será, respe-
tivamente, o majorante ou o minorante.

Erro ou sugestão? Contacta-me: mailto:ricardoferreira.contactar@gmail.com 50


Vê no YouTube: https://www.youtube.com/channel/UCkRcdeyQ50TWFmk7vyuzf_g
Ricardo Ferreira

As sucessões monótonas são muito simpáticas — têm sempre limite.


Isto porque ou são limitadas e assim convergem, ou não são limitadas
e assim divergem para ±∞ (ou seja, não há divergências manhosas do
estilo (−1)n !)

(3) Mostra que as sucessões (un ) seguintes são não monótonas. Faz isto de
duas maneiras:

ˆ uma, mostrando que un+1 − un às vezes é positivo e outras vezes é


negativo
ˆ outra, escolhendo três números r < s < t tal que ur < us mas que
us > ut (ou vice-versa: ur > us mas us < ut )

(a) (−1)n
(b) (−1/2)n
(c) (n − 2)2

As sucessões não monótonas podem ser muita variadas! A primeira é li-


mitada mas divergente, a segunda é convergente e a terceira é divergente
para +∞.

(4) Considera a seguinte sucessão (un ) definida por ramos.


(
3 − 2n, n ≤ 5
un =
(1/2)n , n > 5

Desenha a sucessão (un ) e estuda-a quanto à monotonia, limitação e


convergência. Caso existam, indica um majorante e um minorante da
sucessão, bem como o seu limite.
Ser convergente é ser “eventualmente limitada” para qualquer majorante
ou minorante que escolhamos. Por isso os primeiros termos de uma
sucessão não interessam para saber se a sucessão é convergente.

(5) Considera a seguinte sucessão (un ) definida por recorrência.


(
u1 = 1/2
un+1 = 1/un
Desenha a sucessão (un ) e estuda-a quanto à monotonia, limitação e
convergência. Caso existam, indica um majorante e um minorante da
sucessão, bem como o seu limite.
Nada melhor para perceber uma sucessão do que fazer um desenho!

Erro ou sugestão? Contacta-me: mailto:ricardoferreira.contactar@gmail.com 51


Vê no YouTube: https://www.youtube.com/channel/UCkRcdeyQ50TWFmk7vyuzf_g
Ricardo Ferreira

(6) Seja (un ) uma sucessão limitada e crescente.

(a) A sucessão (un ) é convergente ou divergente? Justifica.


(b) Seja (vn ) a seguinte sucessão definida por ramos.
(
u1 + 1, n = 1
vn =
un−1 , n≥2

Indica se a sucessão (vn ) é monótona, se é limitada e se é conver-


gente.

(7) Considera as duas sucessões mais importantes:

ˆ 1/n é decrescente, limitada (por [0, 1]) e convergente para 0+ (ou


seja, convergente para 0 por valores superiores a 0)
ˆ (−1)n é não monótona, limitada (por [−1, 1]) e divergente.

Dá um exemplo de uma sucessão

(a) limitada mas divergente


(b) estritamente decrescente, mas que não diverge para −∞
(c) estritamente crescente, mas que não diverge para +∞
(d) convergente para 0+
(e) convergente para 0−
(f) convergente para 2+
(g) convergente para 2−
(h) convergente para 0 mas não monótona

Primeiro, uma sucessão pode ser estritamente crescente/decrescente e


não divergir para ±∞ — basta que convirja, ou seja, que se seja limi-
tada. Segundo, é fácil criar uma sucessão convergente para a+ ou a−
usando a ± 1/n. Terceiro, uma sucessão pode ser convergente e não
monótona — basta que o limite seja tanto por valores superiores como
inferiores.

(8) Considera a sucessão (un ) definida por un = (−2)n .

(a) Desenha a sucessão (un ) e determina o número de termos que estão


no intervalo [−32, 64]
(b) Resolvendo a inequação 4 ≤ 2n ≤ 32, determina o número de termos
(un ) de ordem par que estão no intervalo [4, 32].

Erro ou sugestão? Contacta-me: mailto:ricardoferreira.contactar@gmail.com 52


Vê no YouTube: https://www.youtube.com/channel/UCkRcdeyQ50TWFmk7vyuzf_g
Ricardo Ferreira

(9) Considera a sucessão (vn ) definida por

(−1)n+1
vn = −1 + .
n
Determina o número de termos de ordem ı́mpar que estão no intervalo
[−13/14, −7/8].

Erro ou sugestão? Contacta-me: mailto:ricardoferreira.contactar@gmail.com 53


Vê no YouTube: https://www.youtube.com/channel/UCkRcdeyQ50TWFmk7vyuzf_g
Ricardo Ferreira

8 Funções 1 — Funções Clássicas e Equações


8.1 As funções clássicas
As funções mais importantes são:

ˆ xk (e a sua inversa k x)

ˆ ex (e a sua inversa ln x)

ˆ 1/x

ˆ sin x, cos x, tan x

ˆ |x|
Isto porque todas as funções que estudamos são uma combinação destas
funções.
Aprende sobre estas funções em
ˆ https://youtu.be/sG0CRKLQjd8?t=665 (função exponencial)

ˆ https://youtu.be/sG0CRKLQjd8?t=1861 (logaritmo)

ˆ https://youtu.be/7FGwHqXpmBg?t=1637 (funções trigonométricas)

ˆ https://youtu.be/bUU9zb5TVcc?t=1031 (função módulo)

(1) Desenha o gráfico das funções x, x3 , x5 e x7 . E sabendo que o gráfico da


inversa é uma reflexão em relação à reta y = x, desenha o gráfico das
√ √ √
funções 3 x, 5 x e 7 x.
À exceção de x, o gráfico de qualquer função x2n+1 (ou seja, de expoente
ı́mpar) é parecido com o da função x3 — a única diferença é que quanto
maior for o expoente, mais “próximo”do eixo dos yy está o gráfico (isto
porque a função “varia mais rapidamente”). No fundo, se soubermos x
e x3 , então sabemos tudo. Por isso, só precisamos de saber a inversa
√ 3
x.
(2) Desenha o gráfico das funções x2 , x4 e x6 . E com isto, desenha o gráfico
√ √ √
das respetivas funções inversas x, 4 x e 6 x (lembra-te que só podes re-
fletir funções bijetivas, ou seja, em que a cada valor de y corresponde um
e só um valor de x. Por isso, deves “encolher”o domı́nio para [0, +∞[ e
só refletir essa parte do gráfico. Ou para mais informações, lê o Capı́tulo
8.4 “Funções Inversas”).
O gráfico de qualquer função x2n (ou seja, de expoente par) é parecido
com o da função x2 — a única diferença é que quanto maior for o
expoente, mais “próximo”do eixo dos yy”está o gráfico (isto porque a
função “varia mais rapidamente”). No fundo, se soubermos x2 , sabemos

tudo. Por isso, também só precisamos de saber a inversa x.

Erro ou sugestão? Contacta-me: mailto:ricardoferreira.contactar@gmail.com 54


Vê no YouTube: https://www.youtube.com/channel/UCkRcdeyQ50TWFmk7vyuzf_g
Ricardo Ferreira

(3) Desenha o gráfico das funções 2x , ex e 3x . E com isto, desenha o gráfico


das respetivas funções inversas log2 (x), ln x e log3 (x).
O gráfico de qualquer função exponencial ax , com a > 1, tem sempre
a mesma forma — a única diferença é que quanto maior for a base
a, mais “próximo”do eixo dos yy está o gráfico (isto porque a função
“varia mais rapidamente”). No fundo, se soubermos ex , sabemos tudo.
Por isso, só precisamos de saber a inversa ln x.

(4) Considera as funções seguintes.

(a) x (g) ln x
(b) x2 (h) 1/x

(c) x (i) sin x
(d) x3 (j) cos x

(e) 3 x (k) tan x
(f) ex (l) |x|

Para cada uma destas funções, desenha o seu gráfico. E com esse gráfico,
escreve o domı́nio e contradomı́nio, estuda o sinal e zeros, bem como
a sua monotonia e extremos relativos. Para o sinal e monotonia, in-
dica os respetivos intervalos onde a função é positiva/negativa e cres-
cente/descrescente.

8.2 Funções compostas


Além de somar, multiplicar e dividir, há também outra maneira de criar
novas funções a partir das “velhas”, que é aplicar uma função a seguir à
outra — a esta nova função chamamos de função composta. Por exemplo,
g ◦ f lê-se “g após f ”, porque primeiro fazemos a função f e depois fazemos
a função g.

f g
x f (x) g ◦ f (x)
Pela imagem, vemos que domı́nio da composta g ◦ f vai ser os valores de
x tais que x pertence ao domı́nio de f e tais que f (x) pertence ao domı́nio
de g, ou seja, os valores de x tais que

x ∈ Df ∧ f (x) ∈ Dg

E agora, com os nossos valores de x do domı́nio de g ◦ f , sabemos que f (x)


será o “novo”domı́nio de g. Por isso, o contradomı́nio de g ◦ f será igual
ao contradomı́nio de g, mas em vez de considerarmos todos os valores do
domı́nio de g, consideramos só os tais valores f (x).

Erro ou sugestão? Contacta-me: mailto:ricardoferreira.contactar@gmail.com 55


Vê no YouTube: https://www.youtube.com/channel/UCkRcdeyQ50TWFmk7vyuzf_g
Ricardo Ferreira

Aprende mais sobre a função composta em

ˆ https://youtu.be/bUU9zb5TVcc?t=1806 (Definição)

ˆ https://youtu.be/bUU9zb5TVcc?t=2075 (Domı́nio da composta)

ˆ https://youtu.be/bUU9zb5TVcc?t=1149 (Composição com a função


módulo)

(1) Para cada função f e g seguinte, escreve as funções compostas g ◦ f e


f ◦ g.

(a) f (x) = x2 , g(x) = −2x


(b) f (x) = 1/x, g(x) = x3 − 3x
(c) f (x) = ex , g(x) = x2 + 2x
(d) f (x) = tan x, g(x) = sen x
(e) f (x) = tan x, g(x) = ex
(f) f (x) = x2 , g(x) = |x|

Escrever g ◦f (x) é pegar na função g(x) e substituir todos os x por f (x).

(2) Seja f (x) = ln x e g(x) = ex . Repara que g ◦ f (x) = x e que a função


x tem domı́nio R. Calcula o domı́nio da composta g ◦ f pela definição
Dg◦f = {x ∈ R : x ∈ Df ∧ f (x) ∈ Dg } e compara-o com o domı́nio de x.
Para calcular o domı́nio da composta, não podemos calcular g ◦ f e ver
o domı́nio diretamente. Temos é de ver os valores de x que satisfazem
x ∈ Df ∧ f (x) ∈ Dg .

(3) Calcula o domı́nio da função composta g ◦ f , para as seguintes funções


f e g. Com isso, escreve a expressão de g ◦ f (x), desenha o seu gráfico
e determina o seu contradomı́nio.

(a) f (x) = ln x, g(x) = ex (e) f (x) = sen x, g(x) = |x|


(b) f (x) = ex , g(x) = ln x (f) f (x) = 1/x, g(x) = |x|
(c) f (x) = x + 2, g(x) = |x| (g) f (x) = 1/x, g(x) = x2

(d) f (x) = ln x, g(x) = |x| (h) f (x) = x, g(x) = 1/x

A função módulo |· · ·| torna tudo positivo — basta refletir a parte ne-


gativa do gráfico em relação aos eixo dos xx (ou seja, qualquer valor
negativo y < 0 passa a positivo −y). A função x2 também torna tudo
positivo mas ainda “aumenta”os valores — os valores menores que 1
ficam mais perto de 0 e os valores maiores que 1 ficam mais perto de
+∞. A raı́z quadrada, sendo a inversa, tem o efeito contrário.

Erro ou sugestão? Contacta-me: mailto:ricardoferreira.contactar@gmail.com 56


Vê no YouTube: https://www.youtube.com/channel/UCkRcdeyQ50TWFmk7vyuzf_g
Ricardo Ferreira

8.3 Calcular domı́nios


Se o enunciado não disser nada, então qualquer função f (x) tem domı́nio R.
Isto a não ser que tenha:
ˆ frações 1/ · · · ou (· · · )−1 (temos denominador · · · =
6 0)
ˆ tangentes tan(· · · ) (temos ângulo · · · 6= π/2 + kπ)
√ 1
ˆ raı́zes de ı́ndice par 2n · · · ou (· · · ) 2n (temos radicando · · · ≥ 0)
ˆ logaritmos ln(· · · ) (temos · · · > 0)
Para cada fração, tangente raı́z de ı́ndice par ou logaritmo que a função tem,
nós escrevemos a respetiva condição. Intersetando todas estas condições dá-
nos o domı́nio da função.
Tanto as frações como as tangentes têm um domı́nio especial pela própria
maneira como estão definidas. Já as raı́zes de base par e o logaritmo têm
domı́nio especial porque são as inversas de x2n e ex . Assim, o contradomı́nio
das funções originais, que é [0, +∞[ e ]0, +∞[ torna-se no domı́nio das res-
petivas funções inversas.
Para representar o domı́nio de uma função f , nós escrevemos Df .
Descobre como calcular domı́nios, incluindo os da função inversa e da
função composta em https://youtu.be/bUU9zb5TVcc?t=1870.

(1) Calcula o domı́nio das funções seguintes.


1 1
(a) (g)
2x − 4 tan x

(b) (2x − 4)−1 (h) 2x − 6
1 3
(c) 5 + (i) (2x − 6) 4
(x − 1)(x − 2)
(j) ln(3x + 6)
3x3 − 1 4x + 7
(d) + 
1

sin x cos x (k) ln
(e) tan(3x) 3x + 6
tan(πx) −1
   
1
(f) tan (l)
x ln x

8.4 Funções Inversas I: logaritmos e equações


Há duas maneiras de pensar na função inversa.
A primeira maneira é

f ◦ f −1 (x) = x ∧ f −1 ◦ f (x) = x

ou seja, a função inversa f −1 é aquela que corta com a função f (e vice-


versa).

Erro ou sugestão? Contacta-me: mailto:ricardoferreira.contactar@gmail.com 57


Vê no YouTube: https://www.youtube.com/channel/UCkRcdeyQ50TWFmk7vyuzf_g
Ricardo Ferreira

f f −1
x f (x) x f −1 (x)
f −1 f

Esta primeira maneira é mais útil para verificarmos se calculámos bem


a função inversa e também para resolver equações e inequações.

8.4.1 Propriedades dos logaritmos


Há três propriedades fundamentais dos logaritmos:

ln(xy) = ln x + ln y
ln(xy ) = y ln x
ln x
loga (x) =
ln a
Aqui, em vez de escrevermos ln poderı́amos escrever loga para qualquer
a positivo e diferente de 1.
Pelas duas primeiras propriedades, vemos também que ln(1/y) = − ln y
e que ln(x/y) = ln x − ln y.
Para demonstrar estas propriedades, o segredo é transformá-las numa
expressão mais simples — para isso, fazemos a inversa dos dois lados. Por
exemplo, se queremos demonstrar

ln(xy) = ln x + ln y,

então fazendo e elevado ao lado esquerdo e e elevado ao lado direito, ficamos


com
eln(xy) = eln x+ln y .
Mas a inversa corta com a original e então temos

xy = eln x+ln y .

E agora que escrevemos a nossa propriedade de uma maneira mais simples,


é fácil demonstrá-la.
Outro truque muito importante é

ax = ex ln a

porque nos permite transformar qualquer exponencial de base a numa ex-


ponencial de base e — basta usar o facto de que a = eln a .
Aprende as propriedades algébricas dos logaritmos e como usá-las em
demonstrações em https://youtu.be/sG0CRKLQjd8?t=2070.

Erro ou sugestão? Contacta-me: mailto:ricardoferreira.contactar@gmail.com 58


Vê no YouTube: https://www.youtube.com/channel/UCkRcdeyQ50TWFmk7vyuzf_g
Ricardo Ferreira

(1) Sem recorrer às propriedade dos logaritmos, demonstra as seguintes


igualdades.

(a) ln(xy) = ln x + ln y
(b) ln(1/y) = − ln y
(c) ln(x/y) = ln x − ln y
(d) ln(xy ) = y ln x
(e) loga (x) = ln x/ ln a

(2) Utilizando o truque · · · = eln(··· ) , mostra que aln b = bln a .

(3) Calcula log2 (2) e log2 (4). Com isto, mostra que

log2 (x + y) 6= log2 (x) log2 (y).

A propriedade que sabemos é ln(xy) = ln(x) + ln(y) e não ln(x + y) =


ln(x) ln(y)!!!

(4) Mostra que (ln x)2 6= 2 ln x.


A propriedade que sabemos é ln(xy ) = y ln(x) e não (ln x)y = y ln(x)!!!

8.4.2 Resolver equações


Já vimos que para resolver a equação ln x = · · · , fazemos a inversa dos
dois lados e ficamos com x = ln(· · · ). Do mesmo modo, podemos também

usar outras inversas: temos ln(ex ) = x, ( x)2 = x (para x ≥ 0) e temos um
caso especial:

x2 = |x|
Isto acontece porque elevar um número ao quadrado “aumenta”esse número
e torna-o positivo, e depois a raı́z quadrada √ cancela esse aumento mas não
cancela esse “tornar positivo”. Por isso, x2 é a função que torna o número
positivo, ou seja, é a função módulo |x|. É por esta razão que se nós temos
x2 = · · · , então, fazendo a raı́z quadrada dos dois lados, ficamos com |x| =
· · · (e daı́ as soluções ± · · · ).
2
√ Como a raı́z quadrada é a inversa de x , então poderı́amos esperar que
x2 = x. O problema é que quando falamos em inversas, só podemos con-
siderar funções bijetivas. Neste 2
√ caso, para tornar x bijetiva, o seu domı́nio
tem de ser [0, +∞[. 2
Ou seja, x = x é verdade, mas só em [0, +∞[. Mas

como a função 2
x está definida em R, então podemos alargar o domı́nio e

dizer que x2 = |x|.
É importante ainda relembrar que quando resolvemos equações e ine-
quações, temos de verificar se as soluções pertencem ao domı́nio.

Erro ou sugestão? Contacta-me: mailto:ricardoferreira.contactar@gmail.com 59


Vê no YouTube: https://www.youtube.com/channel/UCkRcdeyQ50TWFmk7vyuzf_g
Ricardo Ferreira

Aprende tudo em

ˆ https://youtu.be/bUU9zb5TVcc?t=151 (Funções Bijetivas)

ˆ https://youtu.be/sG0CRKLQjd8?t=2780 (Equações exponenciais e


logarı́tmicas)

(5) Resolve a equação ln(x − 1) = ln(2x). Esta equação não tem soluções.
Porquê?
Quando resolves equações, verifica se as soluções estão no domı́nio!
√ √
(6) Resolve as equações ln(ex + 1) = ln(2ex ) e x2 + 1 = 2x2 . Qual é a
semelhança entre as duas resoluções?
Resolver equações com ln x e ex é seguir exatamente a mesma lógica

que com x e x2 . A única diferença é que ln(ex ) = x, enquanto que

x2 = |x|.

(7) Considera a equação


   
1 2
ln √ − ln =0
x x

. Resolve esta equação de duas maneiras:

ˆ uma, colocando a equação na forma ln(· · · ) = ln(· · · ) e fazendo


inversa nos dois lados;
ˆ outra, usando as propriedades dos logaritmos de modo a colocar a
equação na forma ln(· · · ) = 0.

Quando resolvemos equações com logaritmos, às vezes precisamos de


usar as propriedades dos logaritmos.

(8) Resolve as seguintes equações.



(a) e2x = 5 (k) x2 + 2 − 2x = 0
(b) e2x = 5ex (l) ln(4ex − 4) = 2x
(c) ln x + 1 = 0
p
(m) ln( ln(x + 2) + e2x ) − x = 0
(d) ex (ln x + 1) = 0
(n) (ex − 1)2
(e) ex (x + 1)(ln x + 1) = 0
(o) e2x − 3ex − 4 = 0
(x − 1)(ln x + 1)
(f) =0 (p) 52x − 3 × 5x − 4 = 0
x+1
(g) ln(ex + 2) + ln(ex + 1) = 2x (q) 32x − 3x+1 − 4 = 0
(h) ln(ex + 2) + ln(ex − 1) = 2x (r) 32(x+1) − 3x+1 − 2 = 0
(i) x2 = 4 √
ln x
(j) x2 = −4 (s) =1
ln x − 2

Erro ou sugestão? Contacta-me: mailto:ricardoferreira.contactar@gmail.com 60


Vê no YouTube: https://www.youtube.com/channel/UCkRcdeyQ50TWFmk7vyuzf_g
Ricardo Ferreira

A estratégia geral é: isolar funções f (x) para um lado, x e constantes


para o outro, para depois fazermos inversas f −1 dos dois lados. Ou
então será resolver uma quadrática.

(9) Resolve a equação 35−x = 2x . Faz isto de duas maneiras:

ˆ uma, fazendo log3 dos dois lados e usando as regras dos logaritmos
ˆ outra, aplicando o truque · · · = eln(··· ) ao 2 e ao 3 e fazendo ln dos
dois lados.

(10) Calcula o domı́nio da função


1
f (x) =  .
ln(x − 3) − ln(1/x) − ln(4) (x2 − 3)ex

Erro ou sugestão? Contacta-me: mailto:ricardoferreira.contactar@gmail.com 61


Vê no YouTube: https://www.youtube.com/channel/UCkRcdeyQ50TWFmk7vyuzf_g
Ricardo Ferreira

9 Funções 2 — Transformações, Inequações, Sinal


e Zeros
9.1 Transformações de funções I: como visualizar
Sabendo nós os gráficos das funções clássicas, nós podemos perceber os
gráficos de muitas mais — basta interpretarmos as transformações de funções
af (x), f (bx), f (x + c) e f (x) + d.
As transformações af (x) e f (x) + d, como alteram os valores de y, são
mais óbvias: se antes o nosso valor era y, agora será ay e y + d.
Por outro lado, as transformações f (bx) e f (x + c) são menos intuitivas,
porque, neste caso, não é o gráfico da função que transformamos, mas sim
o plano que está por detrás da função.
Por exemplo, temos f (x + 2). Ora, x + 2 faz avançar o plano duas
unidades para a direita. Assim, se fixarmos o gráfico e mexermos o plano
por detrás deste duas unidades para a direita, então é como se o gráfico
tivesse andado duas unidades para a esquerda!

Erro ou sugestão? Contacta-me: mailto:ricardoferreira.contactar@gmail.com 62


Vê no YouTube: https://www.youtube.com/channel/UCkRcdeyQ50TWFmk7vyuzf_g
Ricardo Ferreira

O mesmo acontece, por exemplo, com f (3x). Ora, 3x dilata horizontal-


mente o plano e por isso é como se contraı́sse horizontalmente o gráfico!

Vê uma explicação detalhada destas https://youtu.be/bUU9zb5TVcc?


t=2674.

(1) Desenha o gráfico das funções 3x, −3x, −3x + 5. Faz isto de duas ma-
neiras:

ˆ uma, descobrindo dois pontos do gráfico e desenhando a reta que


os contém
ˆ outra, transformando a função f (x) = x

(2) Desenha as circunferências de equação x2 + y 2 = 1, (x − 2)2 + y 2 =


1, (x − 2)2 + (y + 3)2 = 1 Faz isto de duas maneiras:

ˆ uma, descobrindo o centro e o raio


ˆ outra, transformando a circunferência de equação x2 + y 2 = 1.

Todas as equações de circunferências (x − a)2 + (y − b)2 = r2 são trans-


formações da equação x2 + y 2 = r2 . Curiosamente, também todas as
equações de planos a(x−x0 )+b(y−y0 )+c(z−z0 ) = 0 são transformações
da equação ax + by + cz = 0.

Erro ou sugestão? Contacta-me: mailto:ricardoferreira.contactar@gmail.com 63


Vê no YouTube: https://www.youtube.com/channel/UCkRcdeyQ50TWFmk7vyuzf_g
Ricardo Ferreira

(3) Desenha o gráfico das funções sin(x − π/2), cos(x − π/2) e sin(π − x).
Faz isto de duas maneiras:

ˆ uma, transformando as funções sin x e cos x;


ˆ outra, aplicando reduções ao primeiro quadrante.

Podemos pensar em reduções ao primeiro quadrante como translações


das funções trigonométricas.

(4) Chamamos Perı́odo Positivo Mı́nimo de uma função f ao menor valor


positivo p tal que f (x) = f (x+p) (ou seja, é o menor valor para a função
se repetir). Por exemplo, sin x e cos x têm perı́odo positivo mı́nimo 2π
enquanto que tan x tem perı́odo positivo mı́nimo π.
Sabendo os perı́odos positivos mı́nimos de sin x, cos x, tan x e pensando
geometricamente em transformações, determina os perı́odos positivos
mı́nimos das seguintes funções.

(a) sin(2x) (e) tan(x/2)


(b) tan(2x)
(f) sin(3x + 5)
(c) cos(πx)
(d) cos(x/2) (g) 4 sin(3x + 5) + 9

Se o perı́odo positivo mı́nimo de f (x) é p, então o perı́odo positivo


mı́nimo de af (bx + c) + d é p/b. Ou seja, a contração/dilatação ho-
rizontal f (bx) é a única transformação que muda o perı́odo positivo
mı́nimo.

(5) Considera a função f (x) = sin(πx) no intervalo [−1, 1]. Desenha o


gráfico das seguintes funções. E com esse gráfico, determina o respetivo
domı́nio e contradomı́nio.

(a) f (x) (e) 4f (x) − 5 (i) f (3x + 5)


(b) f (x) + 3 (f) f (x − 3)
(j) 4f (3x + 5) − 5
(c) 2f (x) (g) f (2x)
(d) 2f (x) + 3 (h) f (2x − 3) (k) 4f (3x + 2) + 1

As transformações f (bx) e f (x + c) afetam os xx e por isso mudam o


domı́nio. Por outro lado, as transformações af (x) e f (x) + d afetam os
yy e por isso mudam o contradomı́nio.

(6) Sabendo o gráfico de 2x e que (1/2) = 2−1 , desenha o gráfico de (1/2)x .


Sabendo os gráficos ax para a > 1, também sabemos os gráficos de
ax para 0 < a < 1 — basta considerar 1/a. Vê https://youtu.be/
sG0CRKLQjd8?t=1078

Erro ou sugestão? Contacta-me: mailto:ricardoferreira.contactar@gmail.com 64


Vê no YouTube: https://www.youtube.com/channel/UCkRcdeyQ50TWFmk7vyuzf_g
Ricardo Ferreira

9.2 Transformações de funções II: funções pares e ı́mpares


Uma função par é aquela que é simétrica em relação ao eixo dos yy. Por
isso, se refletirmos a função em relação ao eixo dos yy, então ficamos com a
mesma função. Ou seja, f (−x) = f (x).
Por outro lado, uma função ı́mpar é aquela que é simétrica em relação
à origem. Por isso, se refletirmos a função primeiro em relação ao eixo dos
yy e depois em relação ao eixo dos xx, então ficamos com a mesma função.
Ou seja, −f (−x) = f (x).
Na prática, a maneira mais fácil de verificar se uma função é par ou
ı́mpar é calculando f (−x) e ver se é igual a f (x) ou −f (x).

Par : f (−x) = f (x)


Ímpar : f (−x) = −f (x)
Aprende como demonstrar que uma função é par ou ı́mpar em https:
//youtu.be/bUU9zb5TVcc?t=3179

(1) Calculando f (−x), determina se as seguintes funções f são pares ou


ı́mpares.

(a) x (e) 1/x (i) ex


(b) x2 (f) sin x
(j) 3
(c) x3 (g) cos x
(d) |x| (h) tan x (k) 0

(2) Considera a função par f (x) = x2 e a função ı́mpar g(x) = x3 . Verifica


se as funções f ◦ f, f ◦ g, g ◦ f e g ◦ g são pares ou ı́mpares e demonstra-o.
Compor funções pares e ı́mpares é como multiplicar números pares e
ı́mpares: dois pares fica par, um par e um ı́mpar fica par, só dois ı́mpares
é que fica ı́mpar.

(3) Mostra que a função (ex + 1)/(ex − 1) é ı́mpar.

(4) Seja f uma função. Sabe-se que, no intervalo [0, +∞[, f tem um único
zero em x = 1 e é crescente.

(a) Supõe agora que f é uma função ı́mpar. Pensando geometricamente,


estuda a monotonia de f em ] − ∞, 0].
(b) Supõe que f é uma função par. Pensando geometricamente, estuda
os zeros e a monotonia de f em ] − ∞, 0].

Erro ou sugestão? Contacta-me: mailto:ricardoferreira.contactar@gmail.com 65


Vê no YouTube: https://www.youtube.com/channel/UCkRcdeyQ50TWFmk7vyuzf_g
Ricardo Ferreira

9.3 Funções Inversas II: Como calcular


A segunda maneira de definir função inversa é

f (x) = y ⇔ x = f −1 (y)

ou seja, se a função original f leva o x para o y, então a função inversa f −1


tem de levar esse y de volta ao x (e vice-versa).

f
x y
f −1

Para podermos andar assim para trás e para a frente, é preciso que a
cada valor de y da função f corresponda um e só um valor de x, ou seja,
que a função f seja bijetiva. Por isso, se for preciso, “encolhemos”o domı́nio
de f para a tornar bijetiva (é o que acontece, por exemplo, com x2 , que só
é bijetiva em [0, +∞[).
Esta segunda maneira é mais útil para calcular a função inversa f −1 —
basta começar com a equação f (x) = y e resolvermos em ordem a x até ficar
x = · · · . E depois, pela definição, já sabemos que · · · será a nossa inversa
f −1 (y)!
No fundo, a função inversa é “trocar”os valores de x e os valores de y.
Por isso,

ˆ graficamente, é uma reflexão em relação à reta y = x, ou seja, à


bissetriz dos quadrantes ı́mpares.

ˆ o domı́nio e contradomı́nio trocam, ou seja, o domı́nio de f é o con-


tradomı́nio de f −1 e o contradomı́nio de f é o domı́nio de f −1 .

Lembramos mais uma vez que só podemos fazer a inversa de funções bijeti-
vas, por isso se tivermos de encolher o domı́nio de f para a tornar bijetiva,
então é esse o domı́nio que consideramos.
Vê como calcular a expressão analı́tica da inversa f −1 em https://
youtu.be/bUU9zb5TVcc?t=1596.

(1) Considera a função bijetiva f (x) = x − 2.

(a) Desenha o gráfico de f . E sabendo que o gráfico de f −1 é uma


reflexão em relação à reta y = x, desenha também o gráfico de f −1 .
(b) Resolvendo a equação y = f (x) em ordem a x, determina a ex-
pressão analı́tica de f −1 . Verifica que o gráfico dessa função f −1 é
o gráfico que desenhaste em (a).
(c) Verifica a tua resposta em (b), mostrando que f ◦ f −1 (x) = x e que
f −1 ◦ f (x) = x.

Erro ou sugestão? Contacta-me: mailto:ricardoferreira.contactar@gmail.com 66


Vê no YouTube: https://www.youtube.com/channel/UCkRcdeyQ50TWFmk7vyuzf_g
Ricardo Ferreira

(2) Considera a função bijetiva g(x) = 2x − 4.

(a) Desenha o gráfico de g. E com isso, desenha também o gráfico de


g −1 .
(b) Resolvendo a equação y = g(x) em ordem a x, determina a expressão
analı́tica de g −1 . Verifica que o gráfico dessa função g −1 é o gráfico
que desenhaste em (a).
(c) Verifica a tua resposta em (b), mostrando que g ◦ g −1 (x) = x e que
g −1 ◦ g(x) = x.

(3) Considera a função bijetiva


(
1/x se x < 0
h(x) = √
x se x ≥ 0.

(a) Desenha o gráfico de h. E com isso, desenha também o gráfico de


h−1 .
(b) Resolvendo a equação y = h(x) em ordem a x, escreve h−1 como
uma função definida por ramos. Verifica que o gráfico dessa função
h−1 é o gráfico que desenhaste em (a).
(c) Verifica a tua resposta em (b), mostrando que h ◦ h−1 (x) = x e que
h−1 ◦ h(x) = x.

(4) Considera as seguintes funções bijetivas.


1 (d) f (x) = ln(ex + 2)
(a) 4 + , x ∈ R\{0}
2x
3x − 5 √
(b) , x ∈ R\{3} (e) f (x) = ln x + 1, x ∈]0, +∞[
2x − 6
3ex − 5
(c) , x ∈ R\{ln 3} (f) f (x) = (x + 2)2 , x ∈] − ∞, −2[
2ex − 6

Para cada uma destas funções f , determina a expressão analı́tica da res-


petiva inversa f −1 . Verifica a tua resposta, mostrando que f ◦f −1 (x) = x
e que f −1 ◦ f (x) = x.

(5) Seja f uma função bijetiva cujo domı́nio é ]− ∞, 4] e cujo contradomı́nio


é [−1, 1]. Determina do domı́nio e o contradomı́nio de f −1 .

Erro ou sugestão? Contacta-me: mailto:ricardoferreira.contactar@gmail.com 67


Vê no YouTube: https://www.youtube.com/channel/UCkRcdeyQ50TWFmk7vyuzf_g
Ricardo Ferreira

9.4 Resolver inequações


Tal como nas equações, resolver inequações é “fazer a mesma coisa”do lado
esquerdo e do lado direito (até chegarmos a x ≥ · · · ou x ≤ · · · ). Ou
dizendo de outra maneira, resolver inequações é aplicar a mesma função do
lado esquerdo e do lado direito.
Por exemplo, se temos x − 5 ≥ 0, então somamos 5 dos dois lados e fica
x ≥ 5. Mas somar 5 é aplicar a função f (x) = x + 5 dos dois lados: se
tı́nhamos x − 5 ≥ 0, agora temos f (x − 5) ≥ f (0), ou seja, x ≥ 5.
Agora, a grande questão é: quando é que temos de trocar os sinais ≥ ou
≤ numa inequação? Ou seja, porque é que −x ≥ 0 fica x ≤ 0 mas x − 5 ≥ 0
fica x ≥ 5?
A grande diferença é que multiplicar por −1 é aplicar a função f (x) = −x
e essa função é decrescente. Por outro lado, f (x) = x + 5 é uma função
crescente.
Se nos lembrarmos das definições, uma função é crescente se para x ≥ y
tivermos f (x) ≥ f (y). Pelo contrário, uma função é decrescente se para x ≥
y, tivermos f (x) ≤ f (y). No fundo, as definições de função crescente
e decrescente ensinam-nos a resolver inequações: elas dizem-nos se
temos de trocar os sinais ≤ ou ≥ quando fazemos essa função nos dois lados.
Assim, se fizermos dos dois lados uma função crescente, então mante-
mos o sinal. Mas se fazemos uma função decrescente, então trocamos o
sinal.
Crescente: mantém o sinal
Decrescente: troca o sinal
Por fim, tal como nas equações, nas inequações o conjunto de solução
tem de pertencer ao domı́nio. Na prática, isto é fazer a interseção destes
dois conjuntos.

(1) Determina o conjunto-solução em R das seguintes inequações.

(a) 2x + 1 ≤ 0 (e) ln x ≥ 3
(b) 3x + 5 < 2x − 2 (f) ln x ≤ 3
(c) 3x + 5 < 4x − 2 (g) ln(2ex − 1) − x ≤ 0
1 1 ex − 1
(d) < , (h) x ≤ ex − 1
3x + 5 4x − 2 e +1

Olha o domı́nio!

(2) Considera a função


1
f (x) = 4 − .
ln(e2x + 1)
Determina os valores de x para os quais f (x) ∈ [0, 4].

Erro ou sugestão? Contacta-me: mailto:ricardoferreira.contactar@gmail.com 68


Vê no YouTube: https://www.youtube.com/channel/UCkRcdeyQ50TWFmk7vyuzf_g
Ricardo Ferreira

(3) Olhando para a cı́rcunferência trigonométrica ou para os gráficos, de-


termina o conjunto-solução em [0, 2π[ das seguintes inequações.

(a) sin x ≥ 0 (c) cos x > 3/2
(b) tan x ≤ 0 (d) tan x ≥ 1

(4) Determina em R o conjunto solução da inequação



24xex (x2 + 1)( x + 1)(|x| + 1) ≤ 0.

Se estamos a resolver ab < 0 e a é sempre positivo, então o que vai


determinar o sinal de ab é b. Ou seja, ab será negativo se e só b for
negativo: ab < 0 ⇔ b < 0.

9.5 Zeros e Sinal


Nas equações, quando temos ab = 0, já sabemos que fica

a = 0 ∨ b = 0.

Mas se agora tivermos ab > 0 ou ab < 0?


Ora, ab é positivo se a e b tiverem o mesmo sinal, ou seja,

(a > 0 ∧ b > 0) ∨ (a < 0 ∧ b < 0).

E ab é negativo se a e b tiverem sinais contrários, ou seja,

(a > 0 ∧ b < 0) ∨ (a < 0 ∧ b > 0).

Mas quando a e b começam a ser as mais variadas funções, torna-se difı́cil


saber em quais intervalos estão quais sinais. Para nos ajudar a fazer isso,
construı́mos um quadro de sinal.
Imaginemos que já temos algo da forma f (x) > 0. Antes de fazer o
quadro de sinal, precisamos de 3 passos.

(1) Organizar a função


(2) Calcular o Domı́nio
(3) Calcular os Zeros
Organizar a função significa fatorizar ao máximo os termos da função.
Fazendo isto, a nossa vida fica fácil — estudar o sinal e os zeros de f (x)
reduz-se ao problema de estudar o sinal e os zeros de cada um dos seus
fatores. Por vezes, até é mais fácil organizar a função em último lugar (por
exemplo, para fatorizar uma quadrática, é mais fácil se já soubermos os
zeros).

Erro ou sugestão? Contacta-me: mailto:ricardoferreira.contactar@gmail.com 69


Vê no YouTube: https://www.youtube.com/channel/UCkRcdeyQ50TWFmk7vyuzf_g
Ricardo Ferreira

Depois, é só saber onde estudar o sinal, calculando o domı́nio e os zeros.


E por fim, estamos prontos para fazer o quadro.
Aprende como fazer o quadro de sinal de uma função em https://
youtu.be/bUU9zb5TVcc?t=3383.

(1) Estuda o sinal e os zeros das seguintes funções. Indica os intervalos


em que cada função é positiva/negativa, bem como os seus zeros (se
existirem).

(a) f (x) = x3 + 5x2 + 6x


x2 − 3x − 4
(b) f (x) =
ex ln x

(2 ln x − 2) 2x − 1
(c) f (x) =
(1 − x2 )

(2) Calcula o domı́nio da função f , definida por


 x
e + xex+1

f (x) = ln .
x2 − 4

(3) Fazendo o quadro de sinal de uma função apropriada, determina o


conjunto-solução em R da seguinte inequação.
2x − 3 x+1
≤ .
x+3 x−1

Para resolver qualquer inequação, o mais fácil é meter tudo para um


lado para ficar na forma f (x) ≥ 0 ou ≤ 0. Depois, é só estudar o sinal
de f .

Erro ou sugestão? Contacta-me: mailto:ricardoferreira.contactar@gmail.com 70


Vê no YouTube: https://www.youtube.com/channel/UCkRcdeyQ50TWFmk7vyuzf_g
Ricardo Ferreira

10 Limites 1 — Como Calcular Limites


10.1 A estratégia geral
Para calcular limites limx→a f (x), em que a pode ser ±∞, só precisamos de
dois passos.

(1) Substituir — direto ou indeterminação?


(2) Fazer limites notáveis
Primeiro, substituimos x por a e fazemos as contas. Se tivermos sorte,
o limite é imediato e nem precisamos passar ao segundo passo.
Se não for imediato, é porque nos dá uma de quatro indeterminações:

∞ 0
∞−∞ 0×∞
∞ 0
Nestes casos, não conseguimos calcular o limite imediatamente porque
um dos fatores quer ficar muito grande e o outro muito pequeno
(por exemplo +∞ vs −∞, ou ∞ vs 1/∞). Nestes casos, nós precisamos de
desempatar, ou seja, perceber se o fator que está a querer ficar muito grande
é mais “rápido”ou mais “lento”do que o fator que quer ficar muito pequeno.
Para desempatar, nós usamos os quatro limites notáveis

ln x ex
lim =0 lim = +∞ (p ∈ R)
x→+∞ x x→+∞ xp

ex − 1 sin x
lim =1 lim =1
x→0 x x→0 x

Os dois limites ln x/x e ex /xp são uma indeterminação ∞/∞, por isso
para desempatar, vemos qual é o termo que vai para infinito mais rapida-
mente. O que esses dois limites notáveis nos dizem é que ln x tende para
infinito mais lentamente do que qualquer função polinomial (por exemplo,
x) e que ex tende para infinito mais rapidamente do que qualquer função
polinomial. Vemos isto porque o limite ln x/x é 0, ou seja, o x “ganha”,
enquanto que o limite ex /xp é +∞, ou seja, o ex “ganha”.
Por outro lado, os limites (ex −1)/x e sin x/x, sendo iguais a 1, dizem-nos
que o termo de cima e o termo de baixo são igualmente rápidos. De certo
modo, o limite diz-nos que a função ex − 1 “comporta-se”como a função x
perto de 0. E do mesmo modo, a função sin x “comporta-se”como a função
x perto de 0.
No fundo, os limites notáveis apenas ajudam-nos a comparar as dife-
rentes “velocidades” a que cada termo tende para o seu limite.
Se queres uma explicação mais detalhada, vê https://youtu.be/2o_
yyxSq6aw?t=14.

Erro ou sugestão? Contacta-me: mailto:ricardoferreira.contactar@gmail.com 71


Vê no YouTube: https://www.youtube.com/channel/UCkRcdeyQ50TWFmk7vyuzf_g
Ricardo Ferreira

10.2 Os limites clássicos


Há certos limites clássicos que vamos usar a toda a hora. E a maneira mais
fácil de os sabermos é desenhando o gráfico.

(1) Desenhando os respetivos gráficos, determina os seguintes limites.

(a) lim x2 (g) lim 1/x (m) lim ln x


x→+∞ x→+∞ x→+∞
(b) lim x2 (h) lim 1/x
x→−∞ x→−∞
√ (n) lim ln x
(c) lim x (i) lim 1/x x→0+
x→+∞ x→0+

(d) lim x (j) lim 1/x (o) lim tan x
x→0+ x→0− x→ π2 −
(e) lim −2 (k) lim ex
x→525 x→+∞
(f) lim 4 (l) lim ex (p) lim tan x
x→−625 x→−∞ x→ π2 +

Limites clássicos? Faz o desenho!

10.3 Truques I: Álgebra de Limites


Como os limites de funções definem-se a partir dos limites de sucessões, então
os limites de funções herdam todas as propriedades simpáticas dos
limites de sucessões — principalmente, a álgebra de limites. Ou seja,

lim (f (x) + g(x)) = lim f (x) + lim g(x)


x→a x→a x→a
lim f (x) × g(x) = lim f (x) × lim g(x)
x→a x→a x→a
lim f (x)/g(x) = lim f (x)/ lim g(x)
x→a x→a x→a

Na prática, sabendo nós os limites clássicos e os limites notáveis, con-


seguimos calcular qualquer limite que envolva somar, multiplicar ou dividir
esses limites.
Vê alguns exemplos de álgebra de limites em https://youtu.be/2o_
yyxSq6aw?t=359.

(1) Dos limites clássicos de 1/x, sabemos que


1 1 1
+
= +∞, − = −∞, = 0.
0 0 ±∞
Usando isto e a álgebra de limites, calcula os seguintes limites.
1 −2 1
(a) lim (b) lim (c) lim
x→+∞ 3x x→+∞ 3x x→+∞ ex

Erro ou sugestão? Contacta-me: mailto:ricardoferreira.contactar@gmail.com 72


Vê no YouTube: https://www.youtube.com/channel/UCkRcdeyQ50TWFmk7vyuzf_g
Ricardo Ferreira

1 1 1
(d) lim (h) lim (l) lim
ln x
x→+∞ x→+∞ xex ln x x→0+− ln x
1 7 12
(e) lim 2 (i) lim (m) lim
x→+∞ x xex ln x
x→+∞ x→0 − ln x
+

1 1 1
(f) lim √ (j) lim √ (n) lim
π + tan x
x→+∞ x x→0+ x x→ 2

1 1 −3
(g) lim (k) lim (o) lim
x→+∞ x3/2 x→0+ ln x x→ π2 − tan x

Sabendo os limites clássicos de 1/x, podemos calcular qualquer limite


1/ · · · .

(2) Usando apenas limites clássicos e álgebra de limites, calcula os seguintes


limites.
x
(a) lim (x + ex ) (f) lim
x→+∞ x→−∞ ex

(b) lim (−x − x) ex
x→+∞ (g) lim 2
x→−∞ x
(c) lim xex
x→+∞ ln x
(h) lim
(d) lim −x ln x x→0+ x
x→+∞
ln(x + 1)
(e) lim (−x)(− ln x) (i) lim
x→+∞ x→0+ ln x

Há muitos casos que parecem indeterminação mas não são!!! Por exem-
plo, ∞ + ∞, ∞ × ∞, 0/∞ e ∞/0 não são indeterminações porque ambos
os termos evoluem na mesma direção — por exemplo, os dois ∞ e ∞
querem ficar muito grandes ou os dois 0 e 1/∞ querem ficar muito pe-
quenos. Conclusão: primeiro de tudo, tenta substituir! E não inventes
indeterminações onde elas não existem!

(3) Usando apenas limites clássicos e álgebra de limites, calcula os seguintes


limites.
tan x tan x
(a) lim (c) lim
x→ π2 − π − 2x x→ π2 + (π − 2x)2
tan x tan x
(b) lim (d) lim
x→ π2 + π − 2x x→ π2 + 3 + π − 2x

Temos que ±∞/0+ = ±∞ e que ±∞/0− = ∓∞. Isto porque 0+ sig-


nifica “tender para 0 por valores maiores que 0 — ou seja, 0+ vai ser
sempre um número positivo e por isso mantém o sinal. Por outro lado,
0− significa “tender para 0 por valores menores que 0 — ou seja, 0− vai
ser sempre um número negativo e por isso troca o sinal. No fundo, 0+

Erro ou sugestão? Contacta-me: mailto:ricardoferreira.contactar@gmail.com 73


Vê no YouTube: https://www.youtube.com/channel/UCkRcdeyQ50TWFmk7vyuzf_g
Ricardo Ferreira

é um “número”positivo e 0− é um “número”negativo. Atenção:


isto já não é verdade, por exemplo, para 3− , porque tender para 3 por
valores menores que 3 é um número positivo!

(4) Usando apenas limites clássicos, limites notáveis e álgebra de limites,


calcula os seguintes limites.

ln x ln x
(a) lim (d) lim
x→+∞ x2 x→+∞ x−1
ln x ln x
(b) lim 525 (e) lim −525
x→+∞ x x→+∞ x
ln x ln x
(c) lim √ 3 (f) lim √ −3
x→+∞ ( x) x→+∞ ( x)

Dica: Quando vemos uma raı́z, devemos imediatamente passá-la a


potência (trabalhar com raı́zes é chato, mas trabalhar com potências
é fácil!)
Se p ≥ 1, então para resolver limx→+∞ ln x/xp basta usar o limite
notável ln x/x. Se p ≤ −1, então o limite é imediato. Por isso, pri-
meiro de tudo, tenta substituir!

(5) Usando apenas limites clássicos, limites notáveis e álgebra de limites,


calcula os seguintes limites.

e2x ex
(a) lim (d) lim √
x→+∞ x x→+∞ x
ex
e2x (e) lim √
(b) lim x→+∞ 3 x
x→+∞ x5

e4x e2x
(c) lim (f) lim √
x→+∞ x5 x→+∞ x

Dica: Escreve e2x = ex × ex


Qualquer função exponencial ex tende para +∞ mais rapidamente do

raı́z. Assim, para comparar ex e x, temos de escrever
que qualquer √
ex na forma e2x . No fundo, o segredo é “meter tudo para dentro da
raı́z”.

Erro ou sugestão? Contacta-me: mailto:ricardoferreira.contactar@gmail.com 74


Vê no YouTube: https://www.youtube.com/channel/UCkRcdeyQ50TWFmk7vyuzf_g
Ricardo Ferreira

(6) Usando apenas limites clássicos, limites notáveis e álgebra de limites,


calcula os seguintes limites.

(a) lim (5x5 + 3x3 + x) (ln x)2


x→−∞ (f) lim
x→+∞ x2
(b) lim (2x4 + 4x2 + 6) e2x
x→−∞ (g) lim
x→+∞ x2
1− ex
(c) lim e2x
x→0 x (h) lim
x→−∞ x2
ex −1 sin x(ex − 1)
(d) lim (i) lim
x→+∞ x x→0 x2
ex + x2 + 3x ln x(ex − 1)
(e) lim (j) lim
x→+∞ x x→0+ x3

Primeiro de tudo, tenta substituir!

10.4 Truques II: Inversão


Por vezes, temos coisas que quase são limites notáveis, só que o numerador
e o denominador estão trocados. Por exemplo,
x
lim .
x→+∞ ln x
Para resolver este problema, basta lembrar-nos das regras das frações:

x 1
= ln x .
ln x x

Assim, aplicando a álgebra de limites, temos que


x 1
lim = ln x
x→+∞ ln x limx→+∞ x
E agora já podemos aplicar o nosso limite notável! Vê outro exemplo em
https://youtu.be/2o_yyxSq6aw?t=481

(1) Combinando limites clássicos e limites notáveis com álgebra de limites


e inversão, calcula os seguintes limites.
x x2
(a) lim (d) lim
x→0 ex −1 x→0 ex − 1
3x x3 + 3x2 + 5x
(b) lim (e) lim
x→0 ex −1 x→0 ex − 1
3x x
(c) lim (f) lim
x→0 1 − ex x→+∞ ln x

Erro ou sugestão? Contacta-me: mailto:ricardoferreira.contactar@gmail.com 75


Vê no YouTube: https://www.youtube.com/channel/UCkRcdeyQ50TWFmk7vyuzf_g
Ricardo Ferreira

x x ln x
(g) lim (l) lim
x→0+ ln x x→+∞ ex
x3
(h) lim x2
x→+∞ ln x (m) lim
x x→0+ sin x(1 − ex )
(i) lim √
x→+∞ ex
( x)5
x3 (n) lim
(j) lim x x→0+ sin x(1 − ex )
x→+∞ e
x3 sin x + x2
(k) lim x (o) lim
x→−∞ e x→0+ xex − x

Primeiro de tudo, tenta substituir!!

10.5 Truques III: Intermediário


Por vezes, temos limites como
ex − 1
lim ,
x→0 sin x

em que não conseguimos comparar os limites ex − 1 e sin x diretamente,


mas podemos compará-los indiretamente através do x (que vamos chamar
de “intermediário”).
Pelos limites notáveis (ex − 1)/x e sin x/x, nós sabemos que tanto ex − 1
como sin x se comportam como x perto de 0. Por isso, ex − 1 tem de se
comportar como sin x perto de 0.
Mas na prática, como é que transmitimos esta ideia de comparar ex − 1
e sin x indiretamente através do x? Ora, é simples: multiplicamos por 1.
Ou mais especificamente, dividimos em cima em baixo pelo nosso
intermediário (neste caso, x). Fica então

(ex − 1)/x
lim
x→0 sin x/x

E agora que já fizemos aparecer os nossos limites notáveis, concluı́mos que
o limite é 1, como esperávamos.
Vê outro exemplo em https://youtu.be/2o_yyxSq6aw?t=791

(1) Criando um intermediário, calcula o limite


ln x
lim .
x→+∞ ex

Pelos limites notáveis ex /xp e ln x/x, nós sabemos que ex tende para
+∞ mais rapidamente do que qualquer função polinomial e que qualquer
função polinomial tende para +∞ mais rapidamente do que ln x. Sendo
assim, ex tende para +∞ mais rapidamente do que ln x.

Erro ou sugestão? Contacta-me: mailto:ricardoferreira.contactar@gmail.com 76


Vê no YouTube: https://www.youtube.com/channel/UCkRcdeyQ50TWFmk7vyuzf_g
Ricardo Ferreira

(2) Usando todas as técnicas que aprendeste até agora, calcula os seguintes
limites.

ex − 1 (ln x)2 x ln x
(a) lim (d) lim (g) lim
x→0 sin x x→+∞ e2x x→+∞ ex
3
e2x − 1 (ln x)2 x ln x
(b) lim (e) lim (h) lim
x→0 sin x x→+∞ ex x→+∞ ex
ex (ln x)525 x3 (ln x)2
(c) lim (f) lim (i) lim
x→+∞ ln x x→+∞ ex x→+∞ ex

Nem sempre o intermediário é x (muitas vezes é xp ). Com este truque,


vemos que que a função ex tende para +∞ mais rapidamente do que
qualquer função logarı́tmica ou polinomial (ou qualquer combinação das
duas).

(3) Usando todas as técnicas que aprendeste até agora, bem como as fórmulas
da trigonometria, calcula os seguintes limites.
tan x sin x tan x
(a) lim (b) lim (c) lim
x→0 x x→0 tan x x→0 ex − 1

A função tan x comporta-se como as funções x, sin x, ex − 1 perto de 0.

10.6 Truques IV: Pôr em evidência


Tal como nas sucessões, quando temos indeterminações ∞ − ∞ ou ∞/∞,
nós já sabemos que são os monomómios mais fortes que “mandam”.
Na prática, para ilustrar esta ideia, o que fazemos é colocar o monómio
mais forte em evidência.
Por exemplo, se tivermos

lim (3x3 − x2 − 2),


x→+∞

temos uma indeterminação ∞ − ∞. Para desempatar, nós vamos colocar o


monómio mais forte, 3x3 , em evidência. Fica então
 
3 1 2
lim x 1 + −
x→+∞ 3x 3x3

Com isto, todos os limites dentro do parênteses tendem para 0 exceto o


primeiro. Resta-nos assim
lim x3 ,
x→+∞

que é o limite do monómio mais forte.

Erro ou sugestão? Contacta-me: mailto:ricardoferreira.contactar@gmail.com 77


Vê no YouTube: https://www.youtube.com/channel/UCkRcdeyQ50TWFmk7vyuzf_g
Ricardo Ferreira

E pensamos da mesma maneira para frações. Se tivermos, por exemplo

ex + 1
lim ,
x→+∞ 2x2 + ln x

temos uma indeterminação ∞/∞. Para desempatar, nós vamos colocar o


monómio mai fortes do numerador e o monómio mais forte do denominador
em evidência (neste caso, ex e 2x2 ). Fica então

1
ex 1 +

ex
lim ln x

x→+∞ 2x2 1 + 2x2

Com isto, todos os termos dentro de parênteses tendem para 0 exceto os


primeiros. Resta-nos assim
ex
lim ,
x→+∞ 2x2

que é o limite do monómio mais forte do numerador a dividir pelo monómio


mais forte do denominador.

(1) Utilizando todas as técnicas que aprendeste até agora, calcula os seguin-
tes limites.

(a) lim (x3 − x2 ) (j) lim (xex − ln x)


x→+∞ x→+∞

(b) lim (x3 − x2 ) (k) lim (ex − x ln x)


x→−∞ x→+∞

(c) lim (x2 − 2x − 3) (l) lim (ex − e2x + 1)


x→+∞ x→+∞

(d) lim (x2 − 2x − 3) ex


(m) lim
x→−∞ x→+∞ x3 + 3x2
(e) lim (ex − 2x + 3) ln x − ex
x→+∞ (n) lim
x→+∞ 2x2 − 3
2 x
(f) lim (x − 2e + 3) ln x − x3
x→+∞
(o) lim
x→+∞ 3x3 + 4x
(g) lim (3x3 − ln x)
x→+∞
ln x − x3
(p) lim
(h) lim (2ex − ln x) x→+∞ 1 − x2
x→+∞
4x4 + x3 ln x + 1
(i) lim (ln x + 2x2 − 3ex ) (q) lim
x→+∞ x→+∞ 1 − 2x4

O monómio mais forte manda!

Erro ou sugestão? Contacta-me: mailto:ricardoferreira.contactar@gmail.com 78


Vê no YouTube: https://www.youtube.com/channel/UCkRcdeyQ50TWFmk7vyuzf_g
Ricardo Ferreira

10.7 Truques V: Conjugado


Para resolver certos limites mais obscuros como
√ √ 
lim x+1− x
x→+∞

ou
1 − cos x
lim ,
x→0 x
o truque é multiplicar o numerador e o denominador pelo “conju-
gado”. √ √
A ideia é que não conseguimos√trabalhar com x + 1 − x ou 1 − cos x,

mas conseguimos trabalhar com ( x + 1)2 − ( x)2 e 1 − cos2 x.
Vê o exemplo de (1 − cos x)/x em https://youtu.be/2o_yyxSq6aw?t=
1086.

(1) Usando todas as técnicas que aprendeste até agora, calcula os seguintes
limites.
√ √ √ √
(a) lim ( x + 1 − x) x2 + 1 − x + 1
x→+∞ (g) lim
√ √ x→0+ x2
x
(b) lim ( e − 1 − e ) x
x→+∞ √
1 ex − 1
(c) lim √ x √ x (h) lim
x→+∞ e −1− e +1 x→0 x
1
(d) lim √ √ ex/2 − 1
x→+∞ ln x + 1 − ln x − 1 (i) lim
p
(e) lim ( x2 + 1 − x)
x→0 x
x→+∞
√ √
x2 + 1 − x+1 ex/2 − 1
(f) lim (j) lim √ √
x→0+ x x→0+ x2 + 1 − x + 1

(2) Usando todas as técnicas que aprendeste até agora, calcula os seguintes
limites.
1 − cos x sin x − tan x
(a) lim (d) lim
x→0 x x→0+ x2
1 − cos x tan x − sin x
(b) lim (e) lim √ √
x→0 x2 x→0+
x + 1 − x2 + 1
3
x
(c) lim
x→0 cos x − 1
+

Trigonometria? Usa as fórmulas!.

Erro ou sugestão? Contacta-me: mailto:ricardoferreira.contactar@gmail.com 79


Vê no YouTube: https://www.youtube.com/channel/UCkRcdeyQ50TWFmk7vyuzf_g
Ricardo Ferreira

10.8 Truques VI: Mudança de variável


A mudança de variável é o truque mais importante dos limites, porque não
só nos permite passar limites quase notáveis a limites notáveis, como
também nos permite mudar o valor para o qual x está a tender. Vê
exemplos destes dois usos em https://youtu.be/2o_yyxSq6aw?t=543
As três mudanças de variável clássicas são

y =x−k y = −x y = 1/x

A primeira permite passar um limite x → k a um limite y → 0. A


segunda permite mudar entre 0− e 0+ , ou −∞ e +∞. A terceira, permite
mudar entre 0+ e +∞ ou 0− e −∞. Qualquer outra mudança de variável é
uma combinação destas três.
Para passar limites notáveis a quase notáveis, ainda temos a mudança
de variável
y = kx
que podemos combinar com as anteriores. Mas ao contrário das anteriores,
x → 0 fica y → 0 e x → ±∞, fica x → ±∞.
E como fazer uma mudança de variável? Dá jeito incluir três
informações:
y =x−2
Se x → 2, então y = x − 2 → 0
x=y+2
A primeira linha é a mudança de variável propriamente dita (essa in-
cluı́mos sempre). As duas linhas seguintes dizem-nos como escrever o limite
na nova variável: a segunda linha diz-nos para onde é que o y tende e a
terceira linha diz-nos que onde estiver x, temos de substituir por y + 2.
Assim, se tivermos o limite

ex−2 − 1
lim
x→2 x2 − x − 2

e fizermos a mudança de variável y = x − 2, então já sabemos que y → 0 e


que x = y + 2. Fica então

e(y+2)−2 − 1
lim .
y→0 (y + 2)2 + (y + 2) − 2

e é só resolver.

Erro ou sugestão? Contacta-me: mailto:ricardoferreira.contactar@gmail.com 80


Vê no YouTube: https://www.youtube.com/channel/UCkRcdeyQ50TWFmk7vyuzf_g
Ricardo Ferreira

(1) Usando todas as técnicas que aprendeste até agora, calcula os seguintes
limites.

ex−2 − 1 ex + 3
(a) lim (f) lim
x→2 x − 2 x→+∞ ln(ex + 3)

ex−2 − 1 x2 + 2x − 3
(b) lim (g) lim
x→2 (x − 2)(x + 2) x→+∞ ln(x + 3)

ex−2 − 1 x2 + 2x − 3
(c) lim (h) lim
x→2 x2 − x − 2 x→+∞ ln(x + 3) + ln(x − 1)
e2x−3 − 1 e5x − 1
(d) lim 2 (i) lim
x→ 32 4x − 4x − 3 x→0 sin(5x)
x+3 tan(5x)
(e) lim (j) lim 2
x→+∞ ln(x + 3) x→0 5x − 5x

Mudança de variável? Primeiro decide qual (y = x − 2). Depois, vê o


que tens de substituir (x = y + 2 e se x → 2, então y = x − 2 → 0).

(2) Usando todas as técnicas que aprendeste até agora, calcula os seguintes
limites.

(a) lim x ln x (d) lim xex


x→0+ x→−∞
(b) lim x2 ln x (e) lim ex ln(−x)
x→0+ x→−∞
(c) lim (x3 − 2x) ln x
x→0+

Indeterminação 0 × ∞? Transforma-a numa indeterminação ∞/∞ ou


0/0! (por exemplo, através de uma mudança de variável). No fundo,
como 1/∞ = 0, então as indeterminações 0 × ∞, ∞/∞ e 0/0 são todas
a mesma indeterminação!

Erro ou sugestão? Contacta-me: mailto:ricardoferreira.contactar@gmail.com 81


Vê no YouTube: https://www.youtube.com/channel/UCkRcdeyQ50TWFmk7vyuzf_g
Ricardo Ferreira

10.9 Truques VII: Criar constante


Na maioria das vezes, temos situações como

ln(3x)
lim ,
x→+∞ x
em que dá quase para fazer uma mudança de variável+limite notável, só
que dava jeito ter um 3 no denominador. Para isso, multiplicamos por 1,
mais precisamente, por 3/3. Assim, um dos 3 vai para onde nós queremos e
outro vai para fora do limite. Agora, podemos facilmente aplicar a mudança
de variável+limite notável.
Vê outro exemplo em https://youtu.be/2o_yyxSq6aw?t=1010

(1) Usando todas as técnicas que aprendeste até agora, calcula os seguintes
limites.

e3x − 1 x e−3x − 1
(a) lim (d) lim (g) lim
x→0 sin(−5x)
x→0 x x→0 sin(−3x)

e−x − 1 ln(3x) e−3x − 1


(b) lim (e) lim (h) lim
x→0 x x→+∞ x x→0 sin(5x)

e3x − 1 ln(5x) tan(2x)


(c) lim (f) lim (i) lim 3x
x→0 x2 x→+∞ x3 x→0 e − 1

(2) Calcula os seguintes limites. Faz isto de duas maneiras: uma, através
de da mudança de variável; outra, através das fórmulas de sin(2x) e
cos(2x).

sin(2x) tan(2x) cos(2x) − 1


(a) lim (b) lim (c) lim
x→0 x x→0 x x→0 x

10.10 Truques VIII: Um limite seminotável


O limite
ln(x + 1)
lim .
x→0 x
calcula-se de uma maneira muito especial: através da mudança de variável

y = ln(x + 1)

Assim, vemos que x = ey − 1 e nós sabemos esse limite notável. No fundo,


isto funciona porque ln(x + 1) é a inversa de ex − 1 — daı́ lhe chamarmos
um limite seminotável.
Vê este e mais exemplos em https://youtu.be/2o_yyxSq6aw?t=1227

Erro ou sugestão? Contacta-me: mailto:ricardoferreira.contactar@gmail.com 82


Vê no YouTube: https://www.youtube.com/channel/UCkRcdeyQ50TWFmk7vyuzf_g
Ricardo Ferreira

(1) Calcula os seguintes limites.

ln(x + 1) ln(3x + 1) ln(5x + 3)


(a) lim (d) lim (g) lim
x→0 x x→0 x x→0− x
x ln(5x + 1) 2
(b) lim (e) lim ln(x + 1)
x→0 ln(x + 1) x→0 x (h) lim
x→0 x2
ln(x + 1) ln(x + 5)
(c) lim √ (f) lim
x→0 x+1−1 x→0+ x

A mudança de variável y =numerador resulta sempre! Mas primeiro,


tenta substituir!

10.11 Truques IX: Continuidade


Consideremos o limite
lim ln(x + 1).
x→0+

Quando substituı́mos imediatamente e dizemos que o limite é 0, o que nós


estamos a fazer secretamente é trocar o limite com a função, ou seja,
 
lim ln(x + 1) = ln lim (x + 1)
x→0+ x→0+

Vemos então que isto dá ln(1+ ), ou seja, 0.


De facto, este é um padrão geral: se f é uma função contı́nua, então
podemos trocar a função e o limite.
E as ótimas notı́cias é que todas as funções que estudámos são contı́nuas!
Ou seja, desde que estejamos dentro do domı́nio da função, podemos sempre
trocar a função e o limite.
De facto, nós pudemos trocar a função e o limite apenas porque ln(· · · ) é
uma função contı́nua e porque x + 1 convergia dentro do domı́nio de ln(· · · ).

(1) Trocando a função e o limite, calcula os seguintes limites. Sempre que


trocares, indica qual é a função contı́nua que estás a trocar.

4 − x3
   
1
(a) lim ln 1 − (f) lim tan
x→+∞ x x→+∞ e2x
2
(b) lim (ln x) 2
x→−∞
(g) lim ex
x→+∞
p
(c) lim x2 + x
x→+∞
2
(h) lim e(x −2x+4)
 
1
(d) lim sin x→+∞
x→+∞ ln x
(e) lim cos (ex ) (i) lim esin(x)/x
x→−∞ x→0

Erro ou sugestão? Contacta-me: mailto:ricardoferreira.contactar@gmail.com 83


Vê no YouTube: https://www.youtube.com/channel/UCkRcdeyQ50TWFmk7vyuzf_g
Ricardo Ferreira

10.12 Como calcular qualquer limite


Limites? Resolvem-se em dois passos!

(I) Primeiro, tenta substituir! É bem possı́vel que o limite não seja
indeterminação.

(II) Mas se for uma indeterminação, então usa limites notáveis. Para
saberes o que fazer, presta atenção a duas coisas:

ˆ para onde o x está a tender;


ˆ para os termos dentro do limite.

Se repararmos, todos os limites notáveis acontecem quando x → +∞


ou quando x → 0. Por isso, se o nosso limite for outro: por exemplo
x → −∞ ou x → 2, então temos que fazer uma mudança de variável.
Depois, os termos dentro do limite dizem-nos quais limites notáveis
usar. Por exemplo, se envolver funções trigonométricas, provavelmente usa-
mos o limite notável sin x/x (por isso, também já sabemos que queremos
x → 0). Se envolver logaritmos, provavelmente usamos ln x/x (por isso, que-
remos x → +∞). Se envolver exponenciais, pode tanto envolver (ex − 1)/x
como ex /xp , por isso temos de perceber se é mais fácil fazer aparecer ex − 1
ou apenas ex (e assim decidir se queremos x → 0 ou x → +∞). Se envol-
ver raı́zes quadradas, provavelmente usamos um conjugado. E se envolver
polinómios, provavelmente pomos em evidência os monómios mais fortes.

(1) Utilizando todas as técnicas que aprendeste (incluindo as fórmulas dos


logaritmos e da trigonometria), resolve os seguintes limites.

x − e5x cos(2x) − 1
(a) lim (h) lim
x→+∞ 3e3x − ln x x→0 x2
(b) lim (e2x − 2x3 − 3x2 ) ex−2 − 2
x→+∞ (i) lim
ex−1 − 1 x→2+ x2 − 2x
(c) lim √
x→1 x − x + 1 e2x + ex − 2
 π (j) lim
(d) lim x − tan x
x→0 x
x→ π2 − 2 ln x(sin x)
(k) lim
(e) lim (2x2 − x)ex x→0+ x2
x→−∞
ln(x2 + x) tan(4x) − sin(3x)
(f) lim (l) lim
x→0+ x x→0 e5x − 1
1 ex
(g) lim 2 (m) lim √
x→0+ x ln x x→+∞ x+1

Primeiro de tudo, tenta substituir!!!

Erro ou sugestão? Contacta-me: mailto:ricardoferreira.contactar@gmail.com 84


Vê no YouTube: https://www.youtube.com/channel/UCkRcdeyQ50TWFmk7vyuzf_g
Ricardo Ferreira

11 Limites 2 — Continuidade e Assı́ntotas


11.1 Limites segundo Heine
É a partir de limites de sucessões que definimos limites de funções.
Nós dizemos que o limite de f (x) quando x tende para a existe se para
qualquer sucessão un que tenda para a, o limite de f (un ) for sempre o
mesmo. E a esse limite da sucessão f (un ) para uma qualquer sucessão un
será o limite da nossa função. No fundo,

lim f (x) = lim f (un )


x→a
para qualquer un → a

É importante notar que a sucessão un tem de estar no domı́nio de


f (só assim é que podemos falar em f (un )). Além disso, o a pode ser ±∞
e não precisa de estar no domı́nio (só precisa ser o limite de uma sucessão
que está no domı́nio).
É a partir daqui que definimos limites laterais: para definir o limite por
valores inferiores a a, limx→a− f (x), basta exigirmos que a nossa sucessão
qualquer un seja sempre inferior a a, ou seja, un < a, isto é, un → a− . Do
mesmo modo, definimos limx→a+ f (x). No fundo,

lim f (x) = lim f (un ) para qualquer un → a+


x→a+
lim f (x) = lim f (un ) para qualquer un → a−
x→a−

Como definimos limites de funções através de limites de sucessões, então


os limites de funções herdam todas as propriedades simpáticas dos
limites de sucessões, nomeadamente a álgebra de limites.
Vê a definição de limite de uma função, bem como as suas primeiras
propriedades em https://youtu.be/pAzn5msVxyQ?t=434.
Ou percebe intuitivamente esta definição através de um exercı́cio de
exame: https://youtu.be/-TZ8kM0_Or0?t=3546.

Erro ou sugestão? Contacta-me: mailto:ricardoferreira.contactar@gmail.com 85


Vê no YouTube: https://www.youtube.com/channel/UCkRcdeyQ50TWFmk7vyuzf_g
Ricardo Ferreira

11.2 Continuidade
Seja a 6= ±∞ um valor que está no domı́nio de f . Uma função f é contı́nua
em a quando o limite
lim f (x)
x→a
existe.
Mas como a está no domı́nio de f , então se o limite existir, terá de ser
igual a f (a). Isto porque a sucessão un = a é uma sucessão que está no
domı́nio de f e que tende para a, e por isso

lim f (x) = lim f (un ) = f (a)


x→a

.
Assim, podemos dizer que f é contı́nua em a ∈ Df se e só se

lim f (x) = f (a).


x→a

Como definimos continuidade através de limites de funções, então a con-


tinuidade herda todas as propriedades simpáticas dos limites de
funções, nomeadamente a álgebra de continuidade (ou seja, a soma, dife-
rença, produto e quociente de funções contı́nuas também é ela uma função
contı́nua).
Na prática, vamos usar a continuidade em duas ocasiões:

(I) Primeiro, para verificar se uma função é contı́nua. Mas como to-
das as funções que estudámos são contı́nuas, então pela álgebra
de continuidade, todas as funções que nós veremos serão contı́nuas. A
única exceção é se a função for definida por ramos. Nesse caso, para
verificar se a função é contı́nua, temos de calcular os limites laterais,
ou seja,
lim f (x) = lim f (x) = f (a)
x→a− x→a+

(II) Segundo, se f for uma função contı́nua, então podemos trocar a


função e o limite, ou seja,

lim f (un ) = f (lim un )

Isto é útil não só para calcular limites de sucessões e funções em geral,
mas principalmente para calcular lim f (un ): primeiro calculamos o
limite de un e só depois avaliamos a função f nesse limite.

Nota: Escrever f (lim un ) é só uma abreviatura para limx→a f (x) em que
un → a (no fundo, “trocar a função e o limite” é apenas a definição de limite
segundo Heine).

Erro ou sugestão? Contacta-me: mailto:ricardoferreira.contactar@gmail.com 86


Vê no YouTube: https://www.youtube.com/channel/UCkRcdeyQ50TWFmk7vyuzf_g
Ricardo Ferreira

Aprende tudo em

ˆ https://youtu.be/pAzn5msVxyQ?t=1008 (Definição)

ˆ https://youtu.be/pAzn5msVxyQ?t=1521 (Como demonstrar conti-


nuidade?)

(1) Seja (un ) a sucessão de termo geral


1
un = 3 − .
n
Seja f a função de domı́nio ] − ∞, 3[ definida por f (x) = log2 (3 − x).
Vamos calcular lim f (un ) de duas maneiras.

(a) Calcula lim un . Com isto, e sabendo que lim f (un ) = f (lim un ),
calcula lim f (un ).
(b) Pensando na monotonia e limite da sucessão un , mostra que os ter-
mos da sucessão un estão no domı́nio de f . Com isto, determina
f (un ) (substituindo x por un ) e calcula lim f (un ).

Ou calculamos lim f (un ) diretamente ou, como f é contı́nua, podemos


usar lim f (un ) = f (lim un ). Vê um exemplo em https: // youtu. be/
8vGjLdOI7Ak? t= 253

(2) Seja (un ) a sucessão de termo geral

(−1)n
un = .
n+1
Seja f a função de domı́nio R\{0} definida por f (x) = 1/ |x|. Vamos
calcular lim f (un ) de duas maneiras.

(a) Calcula lim un . Com isto, e sabendo que lim f (un ) = f (lim un ),
calcula f (un ).
(b) Mostra que os termos da sucessão un estão no domı́nio de f . Com
isto, calcula diretamente lim f (un ).

(3) Seja (un ) a sucessão de termo geral


n+1
un = .
2n + 4
Seja f a função de domı́nio − 21 , 12 definida por f (x) = tan(πx).
 

(a) Estuda a sucessão un quanto à monotonia e calcula lim un .


(b) Usando o teu resultado em (a), calcula lim f (un ).

Erro ou sugestão? Contacta-me: mailto:ricardoferreira.contactar@gmail.com 87


Vê no YouTube: https://www.youtube.com/channel/UCkRcdeyQ50TWFmk7vyuzf_g
Ricardo Ferreira

(4) Considera a função f de domı́nio R\{0} definida por


1
f (x) = −.
x2
Desenha o gráfico da função f e calcula limx→0 f (x). A função f é
contı́nua em x = 0? Justifica.
Uma função só pode ser contı́nua ou não contı́nua em pontos do seu
domı́nio!
(5) Considera a função f , de domı́nio R\{0} definida por f (x) = sin x.
Considera também as funções g e h, de domı́nio R, definidas por
( (
1 se x = 0 0 se x = 0
g(x) = h(x) =
f (x) se x 6= 0. f (x) se x 6= 0.

(a) Desenha os gráficos de f , g e h.


(b) A função f é contı́nua em x = 0? Justifica.
(c) Verifica se a função g é contı́nua em x = 0.
(d) Verifica se a função h é contı́nua em x = 0.

Uma função f é contı́nua em a se limx→a− f (x) = limx→a+ f (x) = f (a).


E não te esqueças: a tem de estar no domı́nio!
(6) Considera a função f , de domı́nio R, definida por

 −x2 + 4x − 3
 se x < 1
 ex−1 − 1



f (x) = log3 (1/9) se x = 1

 4 − 4x

se x > 1.


sin(2x − 2)

Verifica se a função f é contı́nua em x = 1.


(7) Considera a função f , de domı́nio R, definida por

 2−x

se x < 2
f (x) = ln(3 − x)
log2 (x3 − 2x2 + x) se x ≥ 2.

Verifica se a função f é contı́nua em x = 2.


(8) Para um certo número real k, a função f , de domı́nio R, é definida por

 ekx − 1

 se x < 0
f (x) = sin(2x)
  π  π
3 cos x − 2 sin x −
 + tan x + se x ≥ 0.
2 4
Sabendo que a função f é contı́nua em x = 0, determina o valor de k.

Erro ou sugestão? Contacta-me: mailto:ricardoferreira.contactar@gmail.com 88


Vê no YouTube: https://www.youtube.com/channel/UCkRcdeyQ50TWFmk7vyuzf_g
Ricardo Ferreira

(9) Considera a função f , de domı́nio R, definida por


 x+2
e −1
se x ≤ −2
f (x) = 2x − 4
(x + 2) ln(x + 2) se x > −2.

Calcula limx→−2+ f (x). Com isto, conclui imediatamente que f não é


contı́nua em x = −2.
Quando um dos limites laterais é ±∞, então a função não é contı́nua
nesse ponto. Isto porque f (a) nunca pode ser ±∞.

11.3 Assı́ntotas Verticais


Quando um dos limites laterais é ±∞, dizemos que a função tem uma
assı́ntota vertical nesse ponto.
Mas onde é que procuramos estas assı́ntotas? Ora, se a função for
contı́nua em x = a, então sabemos imediatamente que os limites laterais
são iguais a f (a) e por isso nunca podem ser ±∞.
Assim, se houverem assı́ntotas verticais, elas estarão nos pontos onde
a função possa não ser contı́nua. Estes pontos são:
ˆ os extremos abertos do domı́nio

ˆ os “extremos” de cada ramo (numa função definida por ramos).


Por exemplo, se f é a função de domı́nio [−1, 1[, definida por
(
· · · se − 1 ≤ x < 0
f (x) = ,
· · · se 0 ≤ x < 1

então procuramos a assı́ntota vertical nos extremos abertos do domı́nio


(neste caso, x = −1) e nos “extremos” dos ramos (neste caso, x = 0).
No fundo,

(1)Procurar nos pontos onde a função possa não ser contı́nua


(extremos abertos do domı́nio e extremos dos ramos)
(2)Verificar se algum limite lateral é ± ∞.

Lembra-te: basta que um dos limites laterais seja ±∞ para que exista
assı́ntota nesse ponto.
Vê exemplos de como e onde procurar assı́ntotas verticais em https:
//youtu.be/pAzn5msVxyQ?t=1844.

(1) Considera a função f , de domı́nio − π2 , π2 , definida por f (x) = tan x.


 

Desenha o gráfico de f e determina os valores de x para os quais f tem


uma assı́ntota vertical.

Erro ou sugestão? Contacta-me: mailto:ricardoferreira.contactar@gmail.com 89


Vê no YouTube: https://www.youtube.com/channel/UCkRcdeyQ50TWFmk7vyuzf_g
Ricardo Ferreira

Uma função pode ter qualquer número de assı́ntotas verticais. Por


exemplo, tan x tem um número infinito de assı́ntotas verticais! (em
x = π/2 + kπ)

(2) Considera a função f , de domı́nio R, definida por


(
−1/x se x 6= 0
f (x) =
0 se x = 0.

Desenha o gráfico de f . A função f tem uma assı́ntota vertical em


x = 0? Justifica.
Para existir assı́ntota vertical em x = a, não interessa o que acontece
no ponto x = a — apenas interessam os limites laterais.

(3) Considera a função f , de domı́nio [0, 2]\{1}, definida por



2
 x −1

se 0 ≤ x < 1
f (x) = ln(1 − x)

ln(x − 1) tan(πx/2) se 1 < x ≤ 2.

Vamos verificar se a função f tem assı́ntotas verticais.

(a) A função f é contı́nua no seu domı́nio. Porquê? Conclui que se


houver assı́ntota vertical, será em x = 1.
(b) Calcula limx→1− f (x). Com este resultado, ainda não podes concluir
se f tem assı́ntota vertical em x = 1. Porquê?
(c) Calculando limx→1+ f (x), verifica se f tem assı́ntota vertical em
x = 1.

Para que uma função tenha assı́ntota vertical, basta que um dos limites
laterais seja ±∞ — não é preciso que os dois sejam.

Erro ou sugestão? Contacta-me: mailto:ricardoferreira.contactar@gmail.com 90


Vê no YouTube: https://www.youtube.com/channel/UCkRcdeyQ50TWFmk7vyuzf_g
Ricardo Ferreira

(4) Considera a função f , de domı́nio [0, 2], definida por


 √
x−1
se 0 ≤ x < 1


f (x) = tan(x − 1)
log (x + 1)−3  se 1 ≤ x ≤ 2.

2

Em dois passos, vamos mostrar que a função f não tem assı́ntotas ver-
ticais.

(a) A função f é contı́nua em [0, 2]\{1}. Porquê? Conclui que se houver


assı́ntota vertical, será em x = 1.
(b) Mostra que f não tem assı́ntota vertical em x = 1. Conclui que f
não tem assı́ntotas verticais.

Para mostrar que a função não tem assı́ntota vertical em x = a temos


de verificar os dois limites laterais (isto, claro, quando a não está nos
extremos do domı́nio).

(5) Considera a função f , de domı́nio ] − 1, 0], definida por

cos(x + 1) − 1
f (x) = .
sin(x + 1)

Em dois passos, vamos mostrar que a função f não tem assı́ntotas ver-
ticais.

(a) A função f é contı́nua no seu domı́nio. Porquê? Conclui que se


houver assı́ntota vertical, será em x = −1.
(b) Mostra que f não tem assı́ntota vertical em x = −1. Conclui que f
não tem assı́ntotas verticais.

Para verificar se a função tem assı́ntota vertical num extremo aberto


do domı́nio, só calculamos um limite lateral (aquele que pertence ao
domı́nio).

Erro ou sugestão? Contacta-me: mailto:ricardoferreira.contactar@gmail.com 91


Vê no YouTube: https://www.youtube.com/channel/UCkRcdeyQ50TWFmk7vyuzf_g
Ricardo Ferreira

11.4 Assı́ntotas Não Verticais


Mas e se agora, em vez de considerarmos limites quando x → a, considerar-
mos limites quando x → ±∞?
Por vezes, acontece algo inédito: a função f comporta-se como uma
reta em ±∞! Ou, formalmente, se a reta tiver equação y = mx + b, então

lim f (x) − (mx + b) = 0
x→±∞

Quando isto acontece, dizemos que f tem uma assı́ntota em ±∞. Se o


declive m for igual a 0, então temos a equação de uma reta horizontal —
por isso, chamamos-lhe assı́ntota horizontal. Por outro lado, se o declive
não é 0, então chamamos-lhe assı́ntota oblı́qua.
Primeiro, onde procuramos estas assı́ntotas? A resposta é óbvia: em
±∞. Por isso, no máximo, existem duas assı́ntotas não verticais: uma para
−∞, outra para +∞.
E depois, como descobrimos estas assı́ntotas? Primeiro, calculamos o
declive m e depois a ordenada na origem b — fazemos isto calculando dois
limites.
(1) Procurar em ± ∞
(2) m = lim f (x)/x
x→±∞

(3) b = lim f (x) − mx
x→±∞

Percebe porque calculamos assı́ntotas não verticais desta maneira e vê


exemplos em https://youtu.be/pAzn5msVxyQ?t=2222

(1) Considera a função f , de domı́nio ]1, +∞[, definida por

3x2 − x + 3
f (x) = .
x−1
(a) A função f não tem assı́ntota em −∞. Porquê?
(b) Verifica se a função f tem assı́ntota em +∞. Se tiver, indica a
equação dessa assı́ntota.

Para verificar se existe assı́ntota não vertical, primeiro calculamos o


declive m (através do limite de f (x)/x). E com isto, calculamos a or-
denada na origem b (através do limite de f (x) − mx)

Erro ou sugestão? Contacta-me: mailto:ricardoferreira.contactar@gmail.com 92


Vê no YouTube: https://www.youtube.com/channel/UCkRcdeyQ50TWFmk7vyuzf_g
Ricardo Ferreira


(2) Considera a função f , de domı́nio [0, +∞[, definida por f (x) = ex x.

(a) A função f não tem assı́ntota em −∞. Porquê?


(b) Verifica se a função f tem assı́ntota em +∞. Se tiver, indica a
equação dessa assı́ntota.

Se ao tentar calcular o declive m, o limite de f (x)/x for igual a ±∞,


então sabemos logo que aı́ não existe assı́ntota não vertical.

(3) Considera a função f , de domı́nio R, definida por

e2x
f (x) =
ex − x
Estuda a função f quanto à existência de assı́ntotas não verticais. Se
existir alguma assı́ntota, indica a sua equação.
Se o domı́nio é R, então temos de procurar as assı́ntotas em −∞ e em
+∞.

(4) Considera a função f , de domı́nio ] − ∞, 3], definida por f (x) = xex − 1.

(a) A função f não tem assı́ntota em +∞. Porquê?


(b) Mostra que a função f tem assı́ntota horizontal em −∞ e indica a
equação dessa assı́ntota.

Se a função f for o quociente de dois polinómios, então podemos usar a


divisão de polinómios para descobrirmos mais facilmente as assı́ntotas.

11.5 Recapitulando
(1) Considera a função f , de domı́nio 0, π2 , definida por
 

sin(2x)
f (x) =
tan x
Vamos mostrar que f não tem assı́ntotas.

(a) Para que valores de a ∈ R, incluindo ±∞, podem existir assı́ntotas?


Justifica.
(b) Mostra que f não tem assı́ntotas nos valores que mencionaste em
(a). Conclui que f não tem assı́ntotas.

Erro ou sugestão? Contacta-me: mailto:ricardoferreira.contactar@gmail.com 93


Vê no YouTube: https://www.youtube.com/channel/UCkRcdeyQ50TWFmk7vyuzf_g
Ricardo Ferreira

(2) Considera a função f , de domı́nio ] − 3, +∞[, definida por

ex−1 − 1

se x < 1


 2
−x − 2x + 3
f (x) = 4
 x −1

 se x ≥ 1.
x3 + x2
(a) Verifica se a função f é contı́nua em x = 1.
(b) Estuda a função f quanto à existência de assı́ntotas.

Para caçar qualquer assı́ntota, procuramos em ±∞ e nos pontos onde


a função pode não ser contı́nua (extremos abertos do domı́nio e “extre-
mos” dos ramos).

(3) Seja k um número real. Considera a função f , de domı́nio ]0, +∞[,


definida por
kx2 + 1
f (x) = .
x − ln x
Sabe-se que f tem uma assı́ntota em +∞. Determina o valor de k de
modo a que essa assı́ntota seja uma assı́ntota horizontal.

Erro ou sugestão? Contacta-me: mailto:ricardoferreira.contactar@gmail.com 94


Vê no YouTube: https://www.youtube.com/channel/UCkRcdeyQ50TWFmk7vyuzf_g
Ricardo Ferreira

12 Derivadas — Definição, Declive da Reta Tan-


gente, Monotonia e Concavidade
12.1 Taxa Média de Variação e Derivada
A taxa média de variação de f no intervalo [a, b] é dada por

f (b) − f (a)
b−a

Geometricamente, é o declive da reta que liga o ponto (a, f (a)) ao ponto


(b, f (b)).
https://youtu.be/xYD3jzQPGbY?t=18

(1) Calcula a taxa média de variação das seguintes funções nos seguintes
intervalos.

(a) f (x) = 525, em [−1, 1]


(b) f (x) = x, em [−1, 1]
(c) f (x) = −2x + 4, em [−1, 1]
(d) f (x) = x2 , em [−1, 1]
(e) f (x) = x3 − 7x + 2, em [−3, 0]
(f) f (x) = x ln x, em [e−1 , e]
 
π 3π
(g) f (x) = sin x, em ,
2 2
 
1 −π π
(h) f (x) = , em ,
2 + sin(3x) 6 6
 
525 π 7π
(i) f (x) = , em ,
5 + 4 cos(2x) 6 6
(j) f (x) = cos x, em [−1, 1]
Dica: pensa nas reduções ao primeiro quadrante

Erro ou sugestão? Contacta-me: mailto:ricardoferreira.contactar@gmail.com 95


Vê no YouTube: https://www.youtube.com/channel/UCkRcdeyQ50TWFmk7vyuzf_g
Ricardo Ferreira

12.2 Como calcular derivadas


Para calcular derivadas, primeiro partimos das derivadas clássicas.

k0 = 0 (sin x)0 = cos x (ex )0 = ex


(xp )0 = pxp−1 , p ∈ R (cos x)0 = − sin x (ln x)0 = 1/x

E depois, é só combiná-las! Fazemos isto através das regras das derivadas.

(f + g)0 = f 0 + g 0
(f g)0 = f 0 g + f g 0
 0
f f 0g − f g0
=
g g2
(g ◦ f )0 (x) = f 0 (x)g 0 f (x)


Além destas, convém saber a regra

(kf )0 = kf 0 ,

ou seja, podemos tirar constantes para fora da derivada (mas se


a esquecermos, também não faz mal, porque isto é só aplicar a regra do
produto (f g)0 = f 0 g + f g 0 ).
E com isto, conseguimos calcular qualquer derivada!
Vê exemplos de como calcular derivadas, incluindo a derivada da com-
posta, em https://youtu.be/xYD3jzQPGbY?t=530.

(1) Lembrando que k 0 = 0, calcula a derivada das seguintes funções.

(a) 1 (f) 1/2 (k) ln(4)


(b) 2 (g) 0.33
(l) sin(5)
(c) 3 (h) 0.3333 . . .
√ (m) π
(d) 525 (i) 2
(e) −525 (j) e3 (n) 0

Derivada da constante? É zero!

(2) Lembrando que (xp )0 = pxp−1 para qualquer número real p, calcula a
derivada das seguintes funções.

(a) x (d) x4 (g) x−2

(b) x2 (e) x−1 (h) 1/x2

(c) x3 (f) 1/x (i) 1/x5

Erro ou sugestão? Contacta-me: mailto:ricardoferreira.contactar@gmail.com 96


Vê no YouTube: https://www.youtube.com/channel/UCkRcdeyQ50TWFmk7vyuzf_g
Ricardo Ferreira

√ √
(j) 1/x−1 (n) 3 x (r) 1/ 3 x
(k) x1/2 (o) x1/4 √ 5
√ (s) 1/ 4 x
(l) x (p) x5/4
√4
(m) x 1/3 (q) x−3/5 (t) 1/ x5

A derivada de xp funciona para qualquer p real! E isto inclui raı́zes e


denominadores!

(3) Lembrando que (kf )0 = kf 0 ,ou seja, que podemos tirar constantes para
fora da derivada, calcula a derivada das seguintes funções.

(a) x (h) 3x2 (o) 2x5/4


(b) 4x (i) −2x2 √4
(p) 3 x3
(c) −x (j) (1/2)x2

(d) −3x (k) x2 /2 (q) 2/ x
(e) (1/3)x (l) 5x3 √
(r) 1/(2 x)
(f) x/3 (m) −3x3

3
(g) x2 (n) −x3 /3 (s) 1/(4 x4 )

Constantes? Tira para fora!

(4) Lembrando que (f + g)0 = f 0 + g 0 , calcula a derivada das seguintes


funções.

(a) x2 + x (m) 2x3 − 3x2


(b) x3 + x2 (n) (x4 /2) + (x−3/5 /3)
(c) x + 1/x (o) x2 + (x + 1)

(d) x4 + x (p) x2 + x + 1
(e) x1/3 + x−1/3 (q) x2 − (x + 1)

4

3
(f) x3 + x4 (r) x2 − x − 1

4 √
(g) x5 + ( 4 x)5
(s) 2x2 − 3x + 4
(h) x + 1
(t) x4 + x3 + x2 + x + 1
(i) x − 3
(u) x4 − 3x3 + 6x2 − 3x + 1
(j) x + 525 √  √  √
5 5
(v) 5( 5 x)2 − 1/ 4 x2 + 5 32
(k) −3x + 525

(l) 525x + 525 (w) x−1/2 − 4(2x2 − 3 x)

Soma? Separa!

Erro ou sugestão? Contacta-me: mailto:ricardoferreira.contactar@gmail.com 97


Vê no YouTube: https://www.youtube.com/channel/UCkRcdeyQ50TWFmk7vyuzf_g
Ricardo Ferreira

(5) Lembrando que (sin x)0 = cos x e que (cos x)0 = − sin x, calcula a deri-
vada das seguintes funções.

(a) − sin x (d) −(sin x)/2 + (cos x)/6


(b) − cos x (e) 2(sin x − cos x)

(c) cos x + sin x (f) 5 sin x − 7 cos x + 4 x3

(6) Lembrando que (f g)0 = f 0 g + f g 0 , calcula a derivada das seguintes


funções.

(a) x sin x (h) (x + 1) sin x


(b) x cos x (i) (x2 − 2x) sin x
(c) sin x cos x
(j) (2x3 − 6x) cos x
(d) sin x(3 cos x)
(k) x(sin x cos x)
(e) sin x(525 cos x)
(f) sin x(− cos x) (l) x sin x cos x

(g) x(2 sin x) (m) x2 sin x(− cos x)

Produto? “Derivada da primeira vezes a segunda mais derivada da se-


gunda vezes a primeira”.

(7) Lembrando que  0


f f 0g − f g0
= ,
g g2
calcula a derivada das seguintes funções.
cos x −1
(a) sin x/x (h) (m)
sin x + cos x 525 sin x cos x
(b) x/ sin x cos x
(i) sin x cos x
(c) cos x/x 1 − x2 (n)
cos x + 3 x2
(d) x/(3 cos x) (j)
1 − x2
x2 sin x − 1
(e) cos x/x3 x2 − x (o)
√ (k) x2
(f) 2 x/ cos x x2 − 2
1 1
(g) cos x/ sin x (l) (p) sin x −
sin x cos x x3

Quociente? “Derivada da primeira vezes a segunda, menos derivada da


segunda vezes a primeira, a dividir pela segunda ao quadrado”.

(8) Sabendo que tan x = sin x/ cos x, calcula (tan x)0 .


Não precisas memorizar que (tan x)0 = 1/ cos2 (x) — basta aplicar a
regra do quociente.

Erro ou sugestão? Contacta-me: mailto:ricardoferreira.contactar@gmail.com 98


Vê no YouTube: https://www.youtube.com/channel/UCkRcdeyQ50TWFmk7vyuzf_g
Ricardo Ferreira

(9) Usando a derivada da composta (g ◦ f )0 (x) = f 0 (x)g 0 (f (x)), vamos cal-


cular a derivada de
sin(2x).
Neste caso, a função que está lá fora é sin(· · · ), ou seja, g(x) = sin x. E
a função que está lá dentro é esse · · · , ou seja, f (x) = 2x.

(a) Primeiro, calcula “a derivada do que está lá dentro”, ou seja, (2x)0 .
Já temos f 0 (x).
(b) Segundo, calcula “a derivada da função que está lá fora”(sin x)0 e
“avalia-a na função que está lá dentro”, ou seja, substitui x por 2x.
Já temos g 0 (f (x)).
(c) Multiplicando os valores em (a) e (b), obtém a derivada de sin(2x).
Temos f 0 (x) e g 0 (f (x)), por isso temos (g ◦ f )0 (x)!

Composta? “Derivada do que está lá dentro vezes derivada do que está
lá fora avaliada no que está lá dentro!”

(10) Usando a derivada da composta (g ◦ f )0 (x) = f 0 (x)g 0 (f (x)), vamos cal-


cular a derivada de
(1 + cos x)3 .
Neste caso, a função que está lá fora é (· · · )3 , ou seja, g(x) = x3 . E a
função que está lá dentro é esse · · · , ou seja, f (x) = x + cos x.

(a) Primeiro, calcula “a derivada do que está lá dentro”, ou seja, (x +


cos x)0 .
Já temos f 0 (x).
0
(b) Segundo, calcula “a derivada da função que está lá fora” x3 e
“avalia-a na função que está lá dentro”, ou seja, substitui x por
x + cos x.
Já temos g 0 (f (x)).
(c) Multiplicando os valores em (a) e (b), obtém a derivada de (x +
cos x)3 .
Temos f 0 (x) e g 0 (f (x)), por isso temos (g ◦ f )0 (x)!

Composta? “Derivada do que está lá dentro vezes derivada do que está
lá fora avaliada no que está lá dentro!”

Erro ou sugestão? Contacta-me: mailto:ricardoferreira.contactar@gmail.com 99


Vê no YouTube: https://www.youtube.com/channel/UCkRcdeyQ50TWFmk7vyuzf_g
Ricardo Ferreira

(11) Lembrando a derivada da composta (g ◦ f )0 (x) = f 0 (x)g 0 (f (x)), calcula


a derivada das seguintes funções.

(a) sin(−3x) (g) sin2 x (l) x2 − 4x
(b) cos(x3 + 4x) (h) cos3 x (m) (x3 − 2x)5
3
(c) cos(−2x + 3x )2
√ (i) sin3/2 x (n) (cos x + sin x)2
(d) cos( x) √
(j) sin x
(e) sin(cos x) (o) sin2 (3x)
1
(f) tan(2x) (k) √ (p) cos5 (2x)
sin x

Nota: Escrever sinp x é o mesmo que escrever (sin x)p . O mesmo acon-
tece com cos x e tan x.
Composta? “Derivada do que está lá dentro vezes derivada do que está
lá fora avaliada no que está lá dentro!”

(12) Lembrando que (ex )0 = ex , calcula a derivada das seguintes funções.

(a) 525ex ex (k) e(x


3 −3x2 +9x)
(g)
(b) ex + 1 x
(l) e−3x + e2x + e−x
(c) ex + 2x 4x3 − x4
(h) e2x − e−2x
(d) xex ex (m)
x
(e) ex sin x (i) e2x
√ 2 e2x cos x
(f) 2ex x (j) ex (n)
x2

(13) Usando a = eln a , calcula a derivada das seguintes funções.


2 +2x
(a) 3x (b) 5x (c) 7x

Não precisas memorizar que (a··· )0 = (· · · )0 a··· ln a — basta usares que


a = eln a e a derivada da composta. Vê um exemplo em https: //
youtu. be/ sG0CRKLQjd8? t= 2530 .

(14) Lembrando que (ln x)0 = 1/x, calcula a derivada das seguintes funções.

(a) 525 ln x ln x (n) ln(x3 + 7x)


(h)
x2
(b) ln x + 1
ex ln x (o) ln(x4 − 4x3 + 5x2 )
(c) ln x + 2x (i) (p) ln(x4 ) ln(x5 )
x
(d) ln x + 3ex (j) ln(3x) x ln(3x)
(q)
(e) x ln x (k) ln(x/4) e2x
(f) ex ln x (l) ln(−4x2 ) (r) (ln x)5
3
(g) cos x ln x (m) ln(525x + 4) (s) ln(5x + 1)

Erro ou sugestão? Contacta-me: mailto:ricardoferreira.contactar@gmail.com100


Vê no YouTube: https://www.youtube.com/channel/UCkRcdeyQ50TWFmk7vyuzf_g
Ricardo Ferreira

(··· )0
Derivada de ln(· · · )? É (··· ) !

(15) Sabendo que loga (x) = ln x/ ln a, calcula a derivada das seguintes funções.

(a) log3 (x) (b) log5 (3x2 + 1) (c) log7 (x3 + e2x )

(··· )0
Não precisas memorizar que (loga (· · · ))0 = (··· ) ln a — basta usares que
loga (· · · ) = ln(· · · )/ ln a.

12.3 Aplicações I: calcular limites


No fundo, a derivada é um limite. Por isso é que inicialmente calculávamos
derivadas fazendo limites. Mas agora, sabendo nós calcular derivadas, po-
demos ir ao contrário — podemos usar derivadas para calcular limites.

f (x) − f (a)
lim = f 0 (a)
x→a x−a
Até poderı́amos dizer que este é um “limite notável”!
Vê alguns exemplos desta técnica no caso especial x = 0 em https:
//youtu.be/2o_yyxSq6aw?t=1400.

(1) Considera uma função f , de domı́nio R, cuja derivada, de domı́nio R, é


dada por f 0 (x) = 3x + ln x. Calcula

f (x) − f (1)
lim
x→1 1−x

(2) Considera uma função f , de domı́nio R− , cuja derivada, de domı́nio R− ,


é dada por f 0 (x) = x log2 (−x3 ). Calcula

x2 − 4
lim
x→−2 f (x) − f (−2)

(3) Considera uma função f , de domı́nio R, cuja segunda derivada, de


domı́nio R, é dada por f 00 (x) = (x + 3)ex + 1. Calcula

f 0 (x) − f 0 (0)
lim .
x→0 x2 − x
Dica: A segunda derivada f 00 (x) é a derivada da função f 0 (x).

Erro ou sugestão? Contacta-me: mailto:ricardoferreira.contactar@gmail.com101


Vê no YouTube: https://www.youtube.com/channel/UCkRcdeyQ50TWFmk7vyuzf_g
Ricardo Ferreira

12.4 Aplicações II: declive da tangente ao gráfico


A derivada de f no ponto de abcissa a, f 0 (a), tem um claro significado
geométrico.

f 0 (a) = Declive da reta tangente ao gráfico de f no ponto de abcissa a.

Assim, quando te pedem pelo “declive da reta tangente ao gráfico de f no


ponto de abcissa a”, o que é que eles querem?

f 0 (a)!

Vê um exemplo do Exame 2020 2ª fase em https://youtu.be/mW9tClygTGs?


t=4507.

(1) Considera a função f , de domı́nio R, definida por f (x) = x2 + 2x.


Calcula o declive da reta tangente ao gráfico de f no ponto de abcissa
1.
Declive da reta tangente ao gráfico de f no ponto de abcissa a??? f 0 (a),
pá!

(2) Considera a função f , de domı́nio ] − ∞, 2[, definida por f (x) = log3 (8 −


x3 ). Calcula o declive da reta tangente ao gráfico de f no ponto de
abcissa −1.
Declive da reta tangente ao gráfico de f no ponto de abcissa a??? f 0 (a),
pá!

(3) Considera as funções f e g, de domı́nio R, definidas por f (x) = 2x e


g(x) = x3 − 4x3 + 1 Calcula o declive da reta tangente ao gráfico de f ◦ g
no ponto de abcissa 0.
Declive da reta tangente ao gráfico de f no ponto de abcissa a??? f 0 (a),
pá!

(4) No plano, considera a circunferência de raio 1 centrada na origem.


Nós sabemos que essa circunferência é o conjunto de pontos que satisfa-
zem a equação x2 + y 2 = 1. Mas se resolvermos em ordem a y, ficamos
com p
y = ± 1 − x2 .

De facto, podemos representar uma circunferência através de duas funções,


p p
f (x) = 1 − x2 g(x) = − 1 − x2 ,

em que a primeira define a semicircunferência acima do eixo dos xx e


em que a segunda a semicircunferência abaixo do eixo dos xx.

Erro ou sugestão? Contacta-me: mailto:ricardoferreira.contactar@gmail.com102


Vê no YouTube: https://www.youtube.com/channel/UCkRcdeyQ50TWFmk7vyuzf_g
Ricardo Ferreira

(a) Calcula as derivadas f 0 (x) e g 0 (x).


(b) Seja A o ponto de coordenadas (3/5, 4/5). Usando a derivada f 0 (x),
calcula o declive da reta tangente à circunferência no ponto A.
(c) Verifica a tua resposta em (a), calculando esse mesmo declive através
do produto escalar.
Dica: lembra-te que a reta tangente à circunferência em A é per-
pendicular à reta OA.
(d) Usando a tua resposta em (a), calcula a equação da reta tangente à
circunferência no ponto A.

(e) Seja B o ponto de coordenadas (−1/2, − 3/2). Usando a derivada
correta (f 0 (x) ou g 0 (x)), calcula o declive da reta tangente à circun-
ferência no ponto B.
(f) Verifica a tua resposta em (d), calculando esse mesmo declive através
da tangente da inclinação da reta.
Dica: lembra-te que se a reta OB tem inclinação α, então a reta
tangente à circunferência em B tem inclinação α + π/2 (ou seja, as
retas são perpendiculares).
(g) Usando a tua resposta em (d), determina a equação da reta tangente
à circunferência no ponto B.
(h) Seja C o ponto de coordenadas (4/5, −3/5). Usando a derivada
correta (f 0 (x) ou g 0 (x)), determina a equação da reta tangente à
circunferência no ponto C.

(5) Sejam f e g duas funções de domı́nio 0, π2 , definidas por f (x) = sin(2x)


 

e g(x) = ln(ex − 1).

(a) Sabe-se que o eixo Ox interseta os gráficos de f e g nos pontos A e


B respetivamente. Determina as coordenadas de A e B.

Erro ou sugestão? Contacta-me: mailto:ricardoferreira.contactar@gmail.com103


Vê no YouTube: https://www.youtube.com/channel/UCkRcdeyQ50TWFmk7vyuzf_g
Ricardo Ferreira

(b) Sabe-se que a reta tangente ao gráfico de f no ponto A e a reta


tangente ao gráfico de g no ponto B intersetam-se no ponto C.
Calcula a abcissa de C.

12.5 Aplicações III: monotonia e extremos


Acabámos de ver que a derivada é o declive da reta tangente ao gráfico
nesse ponto. Pensando assim, percebemos rapidamente que uma função

f é crescente quando f 0 > 0 e que é decrescente quando f 0 < 0. E se


f 0 = 0, então temos um extremo relativo (um máximo relativo ou um mı́nimo
relativo).
f 0 > 0 : Crescente
f 0 < 0 : Decrescente
f 0 = 0 : Extremo Relativo
No fundo, estudar a monotonia e extremos de f é estudar o sinal e zeros da
derivada f 0 .

Monotonia e extremos de f = Sinal e zeros de f 0

Assim, estudar a monotonia e extremos de f é só fazer um quadro de sinal!!!


(de f 0 ).
A única coisa que temos de ter cuidado é que nem sempre f 0 = 0 cor-
responde a um extremo relativo — é preciso também que a monotonia
mude.
Revê como fazer quadros de sinal em https://youtu.be/bUU9zb5TVcc?
t=3383. E vê exemplos em

ˆ https://youtu.be/xYD3jzQPGbY?t=1121 (Explicação e exemplos)

ˆ https://youtu.be/8vGjLdOI7Ak?t=5489 (Exercı́cio do Exame 2020


1ª fase)

ˆ https://youtu.be/lPJHSR17cLo?t=4462 (Exercı́cio do Exame 2021


1ª fase)

Erro ou sugestão? Contacta-me: mailto:ricardoferreira.contactar@gmail.com104


Vê no YouTube: https://www.youtube.com/channel/UCkRcdeyQ50TWFmk7vyuzf_g
Ricardo Ferreira

(1) As funções x e −x ajudam-nos a lembrar a relação entre a monotonia


de f e o sinal da derivada f 0 .

(a) Desenha os gráficos de x e de −x. Qual é a monotonia de x e de


−x?
(b) Calcula a derivada de x e de −x.

Se f 0 (x) > 0, qual é a monotonia? Lembra-te de x! E se f 0 (x) < 0, qual


é a monotonia? Lembra-te de −x!
(2) Considera a função f (x) = x3 .

(a) Desenha o gráfico de x3 .


(b) Mostra que f 0 (x) = 0 no ponto x = 0.
(c) Estuda a função x3 quanto à monotonia, indicando os intervalos
de monotonia. A função x3 tem um extremo relativo em x = 0?
Justifica.

Extremo relativo? Só se a monotonia mudar!!!


(3) Seja f a função de domı́nio ]0, +∞[, definida por
ln x + 1
f (x) = .
x
(a) Estuda, no intervalo ]1/2, 2[, a função f quanto à monotonia e
quanto à existência de extremos relativos. Na tua resposta, indica
os intervalos de monotonia, bem como os extremos relativos, caso
existam. Quantos extremos relativos existem?
(b) Tal como em (a), estuda a função f quanto à monotonia e quanto à
existência de extremos relativos, mas agora no intervalo [1/2, 2]. E
agora, quantos extremos relativos existem?

Intervalo fechado [a, b]? Temos sempre extremos relativos em x = a e


em x = b!
(4) Seja f a função de domı́nio R, definida por f (x) = (x2 − 2x)2 . Estuda
a função f quanto à monotonia e quanto à existência de extremos rela-
tivos. Na tua resposta, indica os intervalos de monotonia, bem como os
extremos relativos, caso existam.
Extremos relativos? São os valores de y!!!
(5) Seja f uma função, cuja derivada, de domı́nio R+ , é dada por f 0 (x) =
−x2 ln(2x). Estuda a função derivada f 0 quanto à monotonia e quanto
à existência de extremos relativos. Na tua resposta, indica os intervalos
de monotonia, bem como os extremos relativos, caso existam.
Função derivada? É uma função como as outras!

Erro ou sugestão? Contacta-me: mailto:ricardoferreira.contactar@gmail.com105


Vê no YouTube: https://www.youtube.com/channel/UCkRcdeyQ50TWFmk7vyuzf_g
Ricardo Ferreira

12.6 Aplicações IV: concavidade e pontos de inflexão


No fundo, estudar a monotonia e extremos de f é estudar o sinal e zeros da
derivada f 0 .

Monotonia e extremos de f = Sinal e zeros de f 0

Assim, estudar a monotonia e extremos de f é só fazer um quadro de sinal!!!


(de f 0 ).
Vê exemplos de como estudar a concavidade e pontos de inflexão em

ˆ https://youtu.be/xYD3jzQPGbY?t=2072 (Explicação e exemplos)

ˆ https://youtu.be/mW9tClygTGs?t=5842 (Exercı́cio do Exame 2020


2ª fase)

(1) As funções x2 e −x2 ajudam-nos a lembrar a relação entre a concavidade


de f e o sinal da segunda derivada f 00 .

(a) Desenha os gráficos de x2 e de −x2 .


Nós dizemos que x2 tem a “concavidade voltada para cima” e que
−x2 tem a “concavidade voltada para baixo”.
(b) Calcula a segunda derivada de x2 e de −x2 .

Se f 00 (x) > 0, qual é a concavidade? Lembra-te de x2 ! E se f 00 (x) < 0,


qual é a concavidade? Lembra-te de −x2 !

(2) Considera a função f (x) = x4 .

(a) Desenha o gráfico de x4 .


(b) Mostra que f 00 (x) = 0 no ponto x = 0.
(c) Estuda a função x4 quanto ao sentido das concavidades. A função
x4 tem um ponto de inflexão em x = 0? Justifica.

Ponto de inflexão? Só se a concavidade mudar!!!

(3) Para as seguintes funções f , estuda a função f quanto ao sentido das


concavidades e quanto à existência de pontos de inflexão. Na tua res-
posta, inclui:

ˆ os intervalos em que f tem concavidade voltada para cima;


ˆ os intervalos em que f tem concavidade voltada para baixo;
ˆ as abcissas dos pontos de inflexão de f , caso existam.

(a) Função f , de√domı́nio ]0, 1[, cuja derivada, de domı́nio ]0, 1[, é dada
por f 0 (x) = x − x2 .

Erro ou sugestão? Contacta-me: mailto:ricardoferreira.contactar@gmail.com106


Vê no YouTube: https://www.youtube.com/channel/UCkRcdeyQ50TWFmk7vyuzf_g
Ricardo Ferreira

(b) Função f , de domı́nio R, cuja derivada, de domı́nio R, é dada por


f 0 (x) = −x5 − 5x3 .
(c) Função f , de domı́nio R, dada por f (x) = xex + 1.

(4) Seja f uma função de domı́nio R. Sabe-se que f 0 (2) = 0, ou seja, f tem
um extremo relativo em x = 2.

(a) Supõe que f 00 (2) = 525. A função f tem um máximo ou um mı́nimo


em x = 2?
(b) Supõe agora que f 00 (2) = −525. Nesse caso, a função f tem um
máximo ou um mı́nimo em x = 2?

f 0 = 0 e f 00 > 0? Mı́nimo!
f 0 = 0 e f 00 < 0? Máximo!
(Basta pensar em x2 e −x2 ).

Erro ou sugestão? Contacta-me: mailto:ricardoferreira.contactar@gmail.com107


Vê no YouTube: https://www.youtube.com/channel/UCkRcdeyQ50TWFmk7vyuzf_g
Ricardo Ferreira

13 Teorema de Bolzano–Cauchy
Intuitivamente, o teorema de Bolzano diz-nos que “se a função f é contı́nua
em [a, b], então f passa por todos os valores entre f (a) e f (b) em ]a, b[)”.
Mas na prática, nós vamos usar sempre o mesmo caso especial: “se f é
contı́nua em [a, b] e se f (a) e f (b) têm sinais contrários, então f passa por
um zero em ]a, b[.

No fundo, este teorema é uma maneira de mostrar que a função f


tem um zero em ]a, b[ — basta saber que f (a) é negativo e f (b) é positivo
(ou vice-versa) e que a função f é contı́nua em [a, b].
Ou dizendo de outra maneira, o teorema permite-nos mostrar que a
equação f (x) = 0 tem solução em ]a, b[.
Mas claro, não nos vão aparecer sempre equações f (x) = 0. Também
nos vão aparecer equações f (x) = k ou até f (x) = g(x)! Mas isto resolve-
se facilmente — basta passar tudo para um lado: fica f (x) − k = 0 ou
f (x)−g(x) = 0. E agora, se dermos um novo nome a essas funções da direita
(f (x) − k ou f (x) − g(x)) e chamarmo-lhes de h(x), então nós ficamos com
uma equação h(x) = 0! E agora sim, já podemos aplicar o teorema!
No fundo, a estratégia geral é a seguinte.

(1) Tudo para um lado: h(x) = 0


(2) Aplicar o corolário:
• h(a) < 0
• h(b) > 0
• h contı́nua em [a, b]
Conclusão: h(x) = 0 tem solução em ]a, b[!

E claro, onde está h(a) < 0 e h(b) > 0, também poderia estar h(a) > 0 e
h(b) < 0 — o que importa é que h(a) e h(b) tenham sinais diferentes.

Erro ou sugestão? Contacta-me: mailto:ricardoferreira.contactar@gmail.com108


Vê no YouTube: https://www.youtube.com/channel/UCkRcdeyQ50TWFmk7vyuzf_g
Ricardo Ferreira

Vê como aplicar esta estratégia na prática em:

ˆ https://youtu.be/pAzn5msVxyQ?t=3071 (Explicação e exemplos)

ˆ https://youtu.be/mW9tClygTGs?t=4771 (Exercı́cio do Exame 2020


2ª fase)

ˆ https://youtu.be/lPJHSR17cLo?t=4882 (Exercı́cio do Exame 2021


1ª fase)

(1) Seja f a função de domı́nio R, definida por f (x) = 2 sin x − 1. Vamos


mostrar, recorrendo ao teorema dei Bolzano–Cauchy, que a função f tem
πh
pelo menos um zero no intervalo 0, .
2
(a) Mostra que f (0) < 0.
(b) Mostra que f (π/2) > 0.
(c) Justifica porque é que a função f é contı́nua em [0, π/2].

Graças a (a), (b) e (c), podemos aplicar o teorema de Bolzano–Cauchy


i πh
e concluı́mos que f tem pelo menos um zero no intervalo 0, !
2
(2) Seja f a função de domı́nio R, definida por f (x) = 1/x. Mostra que
f (−1) < 0 e que f (1) > 0. Pelo teorema de Bolzano–Cauchy, podemos
concluir que f tem um zero em ] − 1, 1[? Justifica.
f (a) < 0 e f (b) > 0? Não chega!!! A função tem de ser contı́nua em
[a, b]!

(3) Seja f a função de domı́nio R\{0}, definida por f (x) = 1/x. Mos-
tra, recorrendo
 ao teorema de Bolzano–Cauchy, que existe um ponto no
1
intervalo −2, − tal que f (x) = −1.
2
Tudo para um lado!

(4) Seja f a função de domı́nio R+ , definida por f (x) = (1 − ln x)2 . Mostra,


recorrendo ao teorema de Bolzano–Cauchy, que a função derivada f 0

tem pelo menos um zero no intervalo ] e, e2 [.

(5) Seja f a função de domı́nio R, definida por f (x) = xex . Mostra, recor-
rendo ao teorema de Bolzano–Cauchy, que existe pelo menos um ponto
em ] − 1, 1[ tal que a reta tangente ao gráfico de f nesse ponto tem
declive 1.
Declive da reta tangente ao gráfico de f no ponto de abcissa a??? f 0 (a),
pá!

Erro ou sugestão? Contacta-me: mailto:ricardoferreira.contactar@gmail.com109


Vê no YouTube: https://www.youtube.com/channel/UCkRcdeyQ50TWFmk7vyuzf_g
Ricardo Ferreira

(6) Seja f e g as funções, de domı́nio R+ , definidas por f (x) = x2 e g(x) =


1 + ln x. Mostra, recorrendo ao teorema de Bolzano–Cauchy, que a
equação f (x) = g(x) tem pelo menos uma solução no intervalo ]1, e[.
Tudo para um lado!

(7) Sejam f e g as funções de domı́nio R definidas por f (x) = x sin x e


g(x) = cos(2x). Mostra, recorrendo
i πao h teorema de Bolzano–Cauchy,
que existe um ponto no intervalo 0, onde os gráficos das funções f
2
e g se intersetam.

(8) Mostra, recorrendo ao teorema de Bolzano–Cauchy, que a equação x3 +


2 = 4x tem uma solução em R.
Não diz o intervalo? Escolhe tu o intervalo!

Erro ou sugestão? Contacta-me: mailto:ricardoferreira.contactar@gmail.com110


Vê no YouTube: https://www.youtube.com/channel/UCkRcdeyQ50TWFmk7vyuzf_g
Ricardo Ferreira

14 Combinatória 1 — Como Contar


14.1 Estratégia Geral
Para resolvermos qualquer exercı́cio de combinatória, precisamos de dois
passos.

(1) “Ensinar a avozinha”


(2) Traduzir para matemática
Primeiro de tudo, “ensinamos a avozinha”, ou seja, explicamos todo
o nosso processo em Português. As únicas palavras que precisamos são

“e”, “ou”, “escolher”, “ordenar”.

Depois, é só traduzir para linguagem matemática: ficava então

×, +, n Ck , n!.

A partir de agora, vamos ver como cada uma destas palavras se relaciona
com o respetivo sı́mbolo matemático.

14.2 Como contar I: “E (quantas opções)?”


O primeiro conceito fundamental de contagem é o “E quantas opções?”
Sempre que dizemos “E quantas opções”, escrevemos o sinal × seguido do
respetivo número de opções.

“E” = ×
Por exemplo, se temos 2 camisas, 3 pares de calças e 5 pares de sapatos,
de quantas maneiras é que nos podemos vestir?
Primeiro, ensinamos a avozinha, ou seja, descrevemos passo a passo
como escolher a roupa:

“Escolhe a camisa E escolhe as calças E escolhe os sapatos”

Depois, é só traduzir para matemática — onde estiver “e” escrevemos


o sinal ×:

Escolhe a camisa × escolhe as calças × escolhe os sapatos,

E onde estiver “escolhe · · · ”, escrevemos o número de opções:

2 × 3 × 5,

E agora é só fazer as contas!


Vê outro exemplo em https://youtu.be/7Id1tJUNQT0?t=63.

Erro ou sugestão? Contacta-me: mailto:ricardoferreira.contactar@gmail.com111


Vê no YouTube: https://www.youtube.com/channel/UCkRcdeyQ50TWFmk7vyuzf_g
Ricardo Ferreira

(1) Num certo café, o menu de almoço inclui uma bebida, uma sandes e
um pacote de batatas fritas. Sabe-se que esse menu tem três opções de
bebida, quatro opções de sandes e duas opções de batatas fritas.
Vamos ver de quantas maneiras diferentes podemos escolher o menu de
almoço.

(a) Primeiro, “ensina a avozinha”, ou seja, descreve, passo a passo e em


Português, como escolher o menu.
(b) E agora, “traduz para matemática” o que escreveste em (a). Com
isto, determina o número de maneiras diferentes de escolher o menu.

Primeiro, ensina a avozinha! Depois, traduz para matemática!

(2) O Gustavo quer pintar as quatro divisões da sua casa. Sabe-se que o
Gustavo tem cinco cores de tinta (e vai pintar cada divisão de uma só
cor).
Vamos ver de quantas maneiras diferentes o Gustavo pode pintar as
cinco divisões da sua casa.

(a) Primeiro, “ensina a avozinha”, ou seja, descreve, passo a passo e em


Português, como pintar as cinco divisões da casa.
(b) “Traduz para matemática” o que escreveste em (a). Com isto, de-
termina o número de maneiras diferentes de pintar as cinco divisões.

Quando temos n opções e fazemos essa escolha k vezes, então temos


nk maneiras de o fazer. A este caso especial do “E quantas opções?”
chamamos de “arranjos com repetição” e escrevemos n A0k .

(3) O André, para proteger o seu telemóvel, quer escolher um código de


quatro dı́gitos, usando apenas os dı́gitos de 1 a 4. De quantas maneiras
é que o André pode escolher o código se:

(a) puder escolher qualquer código


(b) o primeiro dı́gito for 4
(c) o primeiro dı́gito for 4 e o último dı́gito for 2
(d) o último dı́gito for um número ı́mpar
(e) o código for um número (estritamente) menor que 3000.
(f) o código for uma capicua (um número que se lê de igual forma da
esquerda para a direita ou da direita para a esquerda)
(g) nenhum dos dı́gitos do código se repetir

Erro ou sugestão? Contacta-me: mailto:ricardoferreira.contactar@gmail.com112


Vê no YouTube: https://www.youtube.com/channel/UCkRcdeyQ50TWFmk7vyuzf_g
Ricardo Ferreira

14.3 Como contar II: “Ou”


Quando a nossa primeira escolha afeta as opções seguintes, então
não podemos contar tudo de uma só vez. Em vez disso, temos de separar
em casos. E aı́, usamos o “ou”, que corresponde ao sinal +.

“Ou” = +
Por exemplo, num café, vendem dois menus: o menu pizza e o menu
hamburguer. No menu pizza, temos a pizza e o acompanhamento pode ser
batatas fritas ou salada. No menu hamburguer, temos o hamburguer e o
acompanhamento pode ser batatas fritas, salada ou nuggets. De quantas
maneiras diferentes é que podemos escolher o nosso menu?
Ora, primeiro, ensinamos a avozinha. Neste caso, não podemos dizer

“Escolhe o menu E escolhe o acompanhamento”,

porque cada menu tem acompanhamentos diferentes. Ou seja, o menu que


escolhemos afeta as nossas opções de acompanhamento. Por isso,
temos de separar em casos.

“Escolhe E escolhe o OU Escolhe E escolhe o


pizza acompanhamento hamburguer acompanhamento”

E depois traduzimos para matemática. Como o menu pizza tem 2


opções de acompanhamento e o menu hamburguer tem 3 opções de acom-
panhamento, então fica
1 × 2 + 1 × 3.
E está feito!

(1) O André, para proteger o seu telemóvel, quer escolher um código de


quatro dı́gitos, usando apenas os dı́gitos de 1 a 4. Vamos ver de quantas
maneiras é que o André pode escolher este código, sabendo que o código
é um número (estritamente) menor que 2300.
Como o primeiro dı́gito afeta as nossas opções para os dı́gitos
seguintes, temos de separar em casos.

(a) Primeiro, pensamos no caso em que o primeiro dı́gito é 1. Nesse


caso, de quantas maneiras podemos escolher esse código?
(b) Segundo, pensamos no caso em que o primeiro dı́gito é 2. Nesse
caso, de quantas maneiras podemos escolher esse código?
(c) Terceiro, pensamos no caso em que o primeiro dı́gito é 3 ou 4. Nesse
caso, de quantas maneiras podemos escolher esse código?

Erro ou sugestão? Contacta-me: mailto:ricardoferreira.contactar@gmail.com113


Vê no YouTube: https://www.youtube.com/channel/UCkRcdeyQ50TWFmk7vyuzf_g
Ricardo Ferreira

(d) Combinando as tuas respostas em (a), (b) e (c), determina o número


de maneiras de escolher esse código.

“Ou”? É +!

(2) Quantos números naturais existem no intervalo ]230, 570[, contendo ape-
nas os algarismos 2, 3, 5 e 7?
“Se a primeira escolha afeta as opções seguintes, então separa em ca-
sos.”

14.4 Como contar III: “Ordenar”


Na última alı́nea, vimos o caso especial em que tı́nhamos 4 números para
colocar em 4 espaços, mas sem os repetir.
Ensinando a avozinha, ficava:

“Escolhe 1º dı́gito E escolhe 2º dı́gito E escolhe 3º dı́gito E escolhe 4º dı́gito”

E depois, era só traduzir. Para o primeiro dı́gito, temos 4 opções. Para
o segundo dı́gito, já não podemos repetir o primeiro, por isso 3 opções. Para
o terceiro dı́gito, 2 opções. E para o quarto dı́gito, 1 opção. Ficava então

4 × 3 × 2 × 1.

Ou, no caso geral, se tivermos n dı́gitos para colocar em n espaços, sem os


repetir, então o número de maneiras é

n × (n − 1) × (n − 2) × · · · × 2 × 1,

Ou seja, para o primeiro dı́gito, temos n opções; para o segundo, temos


(n − 1) opções; e assim sucessivamente, até chegarmos ao último dı́gito, em
que só temos 1 opção.
Se pensarmos bem, escolher um código de quatro dı́gitos em que nenhum
deles se repete é pegar em 1, 2, 3, 4 e “ordenar” esses números. E aqui o
número de maneiras é 4 × 3 × 2 × 1.
Do mesmo modo, escolher um código de n dı́gitos em que nenhum deles
se repete é pegar em 1, 2, . . . , n − 1, n e “ordenar” esses números. E aqui o
número de maneiras é n × (n − 1) × · · · × 2 × 1.
E esta é a ideia chave: para “ordenar” n coisas, há n × (n − 1) ×
· · · × 2 × 1 maneiras de o fazer. Mas para não estarmos sempre a escrever
n × (n − 1) × · · · × 2 × 1, vamos abreviá-lo e escrever n!, o chamado, “n
fatorial”.
n! = n × (n − 1) × · · · × 2 × 1
Assim, sempre que quisermos “ordenar” n coisas, já não precisamos de pen-
sar em “E quantas opções?” — basta escrever n!

Erro ou sugestão? Contacta-me: mailto:ricardoferreira.contactar@gmail.com114


Vê no YouTube: https://www.youtube.com/channel/UCkRcdeyQ50TWFmk7vyuzf_g
Ricardo Ferreira

“Ordenar” = n!
Por definição, dizemos que 0! = 1, ou seja, só há uma maneira de ordenar
zero coisas.
Vê outro exemplo em https://youtu.be/7Id1tJUNQT0?t=126.

(1) De quantas maneiras diferentes podemos:

(a) escolher um código de três dı́gitos, usando 1, 2 e 3, sem repetir


nenhum dos dı́gitos?
(b) distribuir quatro bolas de cores diferentes por quatro caixas numer-
adas de 1 a 4?
(c) distribuir cinco pessoas por um carro de cinco lugares (sabendo que
todos podem conduzir)
(d) distribuir cinco pessoas por um carro de cinco lugares, sabendo que
só uma delas pode conduzir.
(e) distribuir cinco pessoas por um carro de cinco lugares, sabendo que
duas delas podem conduzir.
(f) distribuir cinco pessoas por um carro de cinco lugares, sabendo que
só uma delas não pode conduzir.
(g) distribuir cinco pessoas (dois adultos e três crianças) por um carro
de cinco lugares, sabendo que os dois adultos ocupam os dois lugares
da frente e as três crianças os três lugares de trás.
(h) distribuir cinco pessoas (três adultos e duas crianças) por um carro
de cinco lugares, sabendo que as duas crianças ocupam dois dos três
lugares de trás.

Ordenar? n!

14.5 Como contar IV: “Escolher”


Já sabemos “ordenar”. E com isso, podemos definir o conceito de “escolher”.
Aprende como em https://youtu.be/7Id1tJUNQT0?t=206
No fundo, para em n escolher k, primeiro ordenamos todos os n elemen-
tos, mas depois a ordem dos k que escolhemos não interessa, e a ordem dos
n−k que não escolhemos também não interessa. Em linguagem matemática:

n n!
Ck =
k!(n − k)

Assim, quando quisermos em n escolher k, é só usar n Ck

“Escolher” = n Ck

Erro ou sugestão? Contacta-me: mailto:ricardoferreira.contactar@gmail.com115


Vê no YouTube: https://www.youtube.com/channel/UCkRcdeyQ50TWFmk7vyuzf_g
Ricardo Ferreira

Por exemplo, para jogar no Euromilhões, temos de escolher 5 números e


2 estrelas de entre 50 números e 12 estrelas. De quantas maneiras diferentes
é que podemos escolher?
Ora, primeiro, ensinamos a avozinha.

Em 50 escolhe 5 números E em 12 escolhe 2 estrelas

E depois, traduzimos para matemática.


50
C5 × 12 C2

Feito!

(1) O Henrique quer levar seis livros para ler na praia: dois romances, três
livros de ficção cientı́fica e um livro de autoajuda. Sabe-se que o Hen-
rique tem em casa cinco romances, quatro livros de ficcão cientı́fica e
sete livros de autoajuda.
Vamos ver de quantas maneiras diferentes o Henrique pode escolher os
seis livros para levar para a praia.

(a) Primeiro, “ensina a avozinha”, ou seja, descreve, passo a passo e em


Português, como escolher os seis livros.
(b) E agora, “traduz para matemática” o que escreveste em (a). Com
isto, determina o número de maneiras diferentes de escolher os seis
livros.

(2) O Roberto vai de férias para o estrangeiro e quer levar onze livros para
ler: quatro romances, dois livros de ficcção cientı́fica e cinco livros de
autoajuda. Sabe-se que o Roberto vai levar cinco livros na mala de mão
e seis livros na mala de porão tal que

ˆ leve um livro de ficcão cientı́fica em cada mala,


ˆ levar exatamente três livros de autoajuda na mala de porão.

Vamos ver de quantas maneiras é que o Roberto pode distribuir os onze


livros pelas duas malas.

(a) Primeiro, “ensina a avozinha”, ou seja, descreve, passo a passo e em


Português, como distrbuir os onze livros pelas duas malas.
(b) E agora, “traduz para matemática” o que escreveste em (a). Com
isto, determina o número de maneiras diferentes de escolher os seis
livros.

Ensina bem a avozinha!!!

Erro ou sugestão? Contacta-me: mailto:ricardoferreira.contactar@gmail.com116


Vê no YouTube: https://www.youtube.com/channel/UCkRcdeyQ50TWFmk7vyuzf_g
Ricardo Ferreira

(3) Considere, num plano α, duas retas paralelas r e s. Assinalam-se na


reta r, cinco pontos distintos e, na reta s, quatro pontos distintos.

(a) Determina o número de quadriláteros que podemos definir com os


pontos assinalados nas duas retas.
(b) Determina o número de triângulos que podemos definir com os pon-
tos assinalados nas duas retas.

(4) Um baralho de cartas convencional contém 52 cartas. Estas estão divi-


didas em 4 naipes (ouros, copas, paus, espadas) com 13 cartas cada de
diferentes sı́mbolos (números do 2 ao 10, valete, dama, rei, ás).
Determina o número de maneiras de:

(a) escolher uma carta do baralho


(b) escolher três cartas do baralho
(c) escolher cinquenta e uma cartas do baralho
(d) escolher três cartas do baralho tal que nehuma seja um ás
(e) escolher quatro cartas de copas
(f) escolher três cartas de copas e duas cartas de paus
(g) escolher quatro cartas de copas, duas cartas de paus e doze cartas
de espadas
(h) escolher quatro cartas, uma de cada naipe
(i) escolher quatro cartas, todas do mesmo naipe
(j) escolher quatro cartas, três de um certo naipe e uma de um outro
naipe
(k) escolher quatro cartas, duas de um certo naipe e duas de um outro
naipe
(l) escolher quatro cartas, uma de cada naipe e todas com sı́mbolos
diferentes
(m) escolher quatro cartas, duas de um certo naipe e duas de um outro
naipe, todas com sı́mbolos diferentes
(n) escolher três cartas, ou todas de copas ou nenhuma de copas.

Erro ou sugestão? Contacta-me: mailto:ricardoferreira.contactar@gmail.com117


Vê no YouTube: https://www.youtube.com/channel/UCkRcdeyQ50TWFmk7vyuzf_g
Ricardo Ferreira

14.6 Estratégia Geral II e Truques Especiais


Já temos tudo!!! Já tı́nhamos a nossa estratégia geral:

(1) “Ensinar a avozinha”


(2) Traduzir para matemática
E agora, temos também as nossas quatro palavras-chave:

“E” ×
“Ou” +
n
“Escolher” Ck
“Ordenar” n!

E não precisamos de mais nada!

14.6.1 Onde estão os arranjos n Ak ???


Mas e os arranjos sem repetição n Ak ? Onde estão eles? Ora, não precisamos
de arranjos n Ak , porque n Ak é apenas “escolher e ordenar”, ou seja, n Ck ×k!.
Percebe porquê em https://youtu.be/7Id1tJUNQT0?t=349.

(1) Vamos resolver um exercı́cio clássico de arranjos n Ak , mas sem usar ar-
ranjos n Ak . Basta ensinarmos a avozinha como sempre fizemos: usando
“E”, “Ou”, “Escolher”, “Ordenar”.
No âmbito do aniversário de uma escola secundária, onze professores
— dois de História, cinco de Matemática e quatro de Português — vão
tirar uma fotografia, estando todos dispostos lado a lado.
Vamos ver de quantas maneiras podemos posicionar os onze professores
na fotografia.

(a) Primeiro, “ensina a avozinha”, ou seja, descreve, passo a passo e em


Português, como posicionar os onze professores na fotografia.
(b) E agora, “traduz para matemática” o que escreveste em (a), ou seja,
escreve uma expressão que dê o número de maneiras diferentes de
posicionar os onze professores na fotografia.

Arranjos n Ak ? Não precisamos deles!

Erro ou sugestão? Contacta-me: mailto:ricardoferreira.contactar@gmail.com118


Vê no YouTube: https://www.youtube.com/channel/UCkRcdeyQ50TWFmk7vyuzf_g
Ricardo Ferreira

14.6.2 Truques I: negação


Quando, na questão, vemos um

“pelo menos”

o melhor é usar a técnica da negação: primeiro contamos todas as opções e


a essas retiramos os casos que não nos interessam. Aprende este truque em
https://youtu.be/7Id1tJUNQT0?t=1106.

(2) Dentro de um saco, estão três bolas azuis e quatro bolas brancas in-
distinguı́veis ao tato. De quantas maneiras é que podemos retirar três
bolas do saco de modo que pelo menos uma das bolas seja azul?
Vamos resolver isto de duas maneiras.

(a) Primeiro, separando em três casos: no caso de escolher uma bola


azul, no de escolher duas bolas azuis e no de escolher três bolas
azuis.
(b) Segundo, usando a negação: calculamos o “ total”, ou seja, as
maneiras de escolher três das sete bolas, e subtraı́mos os “casos
que correm mal”, ou seja, as maneiras de escolher zero bolas azuis.
Verifica que a tua resposta é igual à que deste em (a).

“Pelo menos”? Negação!!!

(3) O André, para proteger o seu telemóvel, quer escolher um código com
quatro dı́gitos, usando os dı́gitos de 0 a 9 e sem repetir dı́gitos. Quantos
códigos pode o André escolher de modo que:

(a) pelo menos um dos dı́gitos seja ı́mpar?


(b) pelo menos três dos seus dı́gitos sejam números pares?

“Pelo menos”? Negação!!! (mas às vezes é melhor não :P)

Erro ou sugestão? Contacta-me: mailto:ricardoferreira.contactar@gmail.com119


Vê no YouTube: https://www.youtube.com/channel/UCkRcdeyQ50TWFmk7vyuzf_g
Ricardo Ferreira

14.6.3 Truques II: caixa


Quando, na questão, vemos um

“juntos”

o melhor é usar a técnica da caixa: primeiro, aquilo que queremos que


fique junto colocamos numa caixa. Depois, ordenamos as caixas e, por
fim, ordenamos o conteúdo dentro de cada caixa. Aprende este truque em
https://youtu.be/7Id1tJUNQT0?t=1230.

(4) A Inês terminou o secundário e quer guardar seis livros — três de


Matemática, dois de Inglês, um de Filosofia — numa prateleira do seu
quarto.

(a) Primeiro, a Inês arranjou três caixas: numa escreveu “Matemática”,


noutra “Inglês” e noutra “Filosofia”. E depois, em cada caixa, colo-
cou os respetivos livros (empilhados uns em cima dos outros). Por
fim, a Inês colocou as três caixas em cima da prateleira (empilhadas
umas em cima das outras). De quantas maneiras é que a Inês pode
colocar os livros e as caixas na prateleira?
(b) A Inês concluiu que usar caixas não era prático. Por isso, decidiu
empilhar os seis livros uns em cima dos outros na prateleira. De
quantas maneiras é que a Inês pode fazer isto de modo que os livros
de cada disciplina fiquem juntos?

“Juntos”? Mete-os numa caixa!

(5) Todos os membros de uma comitiva olı́mpica — dois treinadores, dois


dirigentes e cinco atletas — vão tirar uma fotografia, estando dispostos
lado a lado.
De quantas maneiras podemos posicionar os membros da comitiva sabendo
que:

(a) os dois dirigentes ficam juntos


(b) os dois dirigentes ficam juntos e os cinco atletas ficam juntos
(c) os dois dirigentes ficam juntos, os cinco atletas ficam juntos e os
dois treinadores ficam juntos.

Não precisa ficar junto? Então dá-lhe uma caixa individual!

Erro ou sugestão? Contacta-me: mailto:ricardoferreira.contactar@gmail.com120


Vê no YouTube: https://www.youtube.com/channel/UCkRcdeyQ50TWFmk7vyuzf_g
Ricardo Ferreira

14.6.4 Truques III: sanduı́che


Quando, na questão, vemos um

“não juntos”

o melhor é usar a técnica da sanduı́che: primeiro, colocamos os que po-


dem ficar juntos. E depois, nos espaços vazios entre eles, colocamos os
que não podem ficar juntos. Aprende este truque em https://youtu.be/
7Id1tJUNQT0?t=1335.

(6) Todos os membros de uma comitiva olı́mpica — dois treinadores, dois


dirigentes e cinco atletas — vão tirar uma fotografia, estando dispostos
lado a lado.
De quantas maneiras podemos posicionar os membros da comitiva de
modo que:

(a) os dois dirigentes não fiquem juntos?


(b) os quatro não atletas (dois dirigentes e dois treinadores) não fiquem
juntos?
(c) os cinco atletas não fiquem juntos?

“Não fiquem juntos”? Ensaduı́cha-os entre os que podem ficar juntos!

14.6.5 Recapitulando
(7) Uma turma de mestrado, composta por dois professores (um do sexo
masculino e um do sexo feminino) e nove alunos (três do sexo masculino
e seis do sexo feminino), vão tirar uma fotografia.
Para isso, os onze membros da turma vão posicionar-se lado a lado, di-
vididos em duas filas: três membros ficarão à frente, sentados, enquanto
que os outros oito membros ficarão atrás, em pé.

(8) Escreve a expressão que dê o número maneiras de posicionar os onze


membros da turma de modo que:

(a) não haja qualquer restrição.


(b) todos os alunos do sexo masculino fiquem sentados.
(c) nenhum aluno do sexo masculino fique sentado.
(d) pelo menos um dos alunos do sexo masculino fique sentado.
(e) os três alunos do sexo masculino fiquem em pé e juntos.
(f) os três alunos do sexo masculino fiquem juntos.
(g) os dois professores fiquem em pé e separados

Erro ou sugestão? Contacta-me: mailto:ricardoferreira.contactar@gmail.com121


Vê no YouTube: https://www.youtube.com/channel/UCkRcdeyQ50TWFmk7vyuzf_g
Ricardo Ferreira

(h) os três alunos do sexo masculino fiquem em pé e separados e a


professora do sexo feminino fique sentada.
(i) os dois professores fiquem juntos e os três alunos do sexo masculino
fiquem em pé e separados

Primeiro? Ensina a avozinha: e, ou, escolher, ordenar! E depois?


Traduz para matemática: ×, +, n Ck , n!

Erro ou sugestão? Contacta-me: mailto:ricardoferreira.contactar@gmail.com122


Vê no YouTube: https://www.youtube.com/channel/UCkRcdeyQ50TWFmk7vyuzf_g
Ricardo Ferreira

15 Combinatória 2 — Triângulo de Pascal, Binómio


de Newton e Equações
15.1 Triângulo de Pascal
Como calcular as combinações n Ck ? A maneira mais fácil é através da
fórmula
n n!
Ck = .
k!(n − k)!
Mas para valores mais pequenos, pode ser muito útil o triângulo de Pascal.
Aprende tudo sobre este triângulo em https://youtu.be/7Id1tJUNQT0?t=
509.
No fundo, quando desenhamos

1
1 1
1 2 1
1 3 3 1
1 4 6 4 1
..
.

o que estamos realmente a desenhar é um triângulo com todas as com-


binações n Ck .

0
n= 0 C0
1 1
n= 1 C0 C1
2 2 2
n= 2 C0 C1 C2
3 3 3 3
n= 3 C0 C1 C2 C3
4 4 4 4 4
n= 4 C0 C1 C2 C3 C4
. ..
n = .. .

Ou seja, desenhando o triângulo, sabemos todas as combinações n Ck . Ao


desenhá-lo, usamos que os extremos do triângulo são 1 e que para obter
o elemento seguinte temos de somar os dois elementos adjacentes
anteriores.
n
C0 = n C0 = 1
n
Ck = n−1 Ck−1 + n−1 Ck

Erro ou sugestão? Contacta-me: mailto:ricardoferreira.contactar@gmail.com123


Vê no YouTube: https://www.youtube.com/channel/UCkRcdeyQ50TWFmk7vyuzf_g
Ricardo Ferreira

E ao fazermos o triângulo, vemos ainda que este é simétrico e que a


soma dos números da linha n é igual a 2n .

n
Ck = n Cn−k
n
C0 + n C1 + · · · + n Cn = 2n

Mas talvez a melhor maneira de pensar no triângulo de Pascal é que a


linha n é um buffet com n pratos.
Por exemplo, a linha 3 do triângulo de Pascal é um buffet com 3 pratos.
Se o buffet tem 3 pratos, então para cada um dos pratos temos duas
opções: ou comemos ou não comemos — temos então

2 × 2 × 2 = 23

opções. Mas também podemos pensar de outra maneira: dos 3 pratos, ou


comemos 0 ou comemos 1 ou comemos 2 ou comemos 3 — temos então
3
C0 + 3 C1 + 3 C2 + 3 C3

opções. Conclusão?
3
C0 + 3 C1 + 3 C2 + 3 C3 = 23 ,

ou seja, a soma dos números linha 3 do triângulo é 23 .


Ou, regra geral, a linha n do triângulo de Pascal é um buffet com n
pratos. Para cada um dos pratos, ou comemos, ou não comemos, por isso
temos 2n opções. Por outro lado, se temos n pratos, ou comemos 0, ou
comemos 1, ou comemos 2, ou comemos · · · , ou comemos n, por isso temos
n
C0 + n C1 + · · · + n Cn opções. Conclusão?
n
C0 + n C1 + · · · + n Cn = 2n ,

ou seja, a soma da linha n do triângulo é 2n !


É também pensando num buffet com n pratos que vemos que a linha n
do triângulo de Pascal tem n + 1 elementos: n C0 , n C1 , . . . , n Cn .

(1) Qual é a ordem da linha do triângulo de Pascal em que

(a) o segundo elemento é 13


(b) o penúltimo elemento é 13
(c) o penútimo elemento é 525
(d) o segundo elemento é o quádruplo do primeiro elemento
(e) o penúltimo elemento é o quı́ntuplo do último elemento
(f) o último elemento é um sétimo do segundo elemento

Erro ou sugestão? Contacta-me: mailto:ricardoferreira.contactar@gmail.com124


Vê no YouTube: https://www.youtube.com/channel/UCkRcdeyQ50TWFmk7vyuzf_g
Ricardo Ferreira

(g) a linha tem 5 elementos


(h) a linha tem 13 elementos
(i) a linha tem 525 elementos
(j) a soma dos seus elementos é 28
(k) a soma dos seus elementos é 214
(l) a soma dos seus elementos é 2048
(m) a soma dos elementos da linha seguinte é 64
(n) a soma dos dois primeiros elementos é 16
(o) a soma dos dois últimos elementos é 27
(p) a soma do segundo e do penúltimo elementos é 26
(q) a soma do primeiro elemento com os dois últimos elementos é 27
(r) a soma dos dois primeiros com os dois últimos elementos é 28
Linha de ordem n? Primeiro e último é 1. Segundo e penúltimo é
n. Tem n + 1 elementos. E soma é 2n .

(2) O elemento do meio é sempre o maior elemento. Se n for par, então


só há um elemento do meio: n Cn/2 . Se n for ı́mpar, então temos dois
elementos do meio: n C(n−1)/2 e n C(n+1)/2
Sabendo isto, qual é o valor do maior elemento:

(a) da linha de ordem 1?


(b) da linha de ordem 2?
(c) da linha de ordem 3?
(d) da linha de ordem 4?
(e) da linha de ordem 8?
(f) da linha de ordem 11?
(g) da linha de ordem 300? (escreve a tua resposta na forma n Ck )
(h) da linha de ordem 401? (escreve a tua resposta na forma n Ck )

(3) Lembrando que n Ck = n Cn−k , ou seja, que o triângulo de Pascal é


simétrico, resolve as seguintes questões.

(a) Sabendo que o segundo elemento de uma linha é 7 qual o valor do


penúltimo elemento?
(b) Sabendo que o valor do antepenúltimo elemento é 66, qual o valor
do terceiro elemento?
(c) Sabendo que a soma dos três primeiros elementos é 22, qual é a
soma dos três últimos elementos?

Erro ou sugestão? Contacta-me: mailto:ricardoferreira.contactar@gmail.com125


Vê no YouTube: https://www.youtube.com/channel/UCkRcdeyQ50TWFmk7vyuzf_g
Ricardo Ferreira

(d) Sabendo que a soma do primeiro, do terceiro e do penúltimo ele-


mentos é 56, qual é a soma do segundo e do terceiro elementos?
(e) Sabendo que a linha de ordem n do triângulo de Pascal satisfaz
n
C1 = n C2 , determina o valor de n.
(f) Sabendo que a linha de ordem n do triângulo de Pascal satisfaz
n
C4 = n C6 , determina o valor de n.
(g) Determina quantos elementos da linha de ordem 8 são maiores do
que 8 C3 .
(h) Determina quantos elementos da linha de ordem 8 são maiores do
que 8 C1 .
(i) Determina quantos elementos da linha de ordem 525 são maiores do
que 525 C3 .

(4) Lembrando que n Ck = n−1 Ck−1 + n−1 Ck , ou seja, que a soma de dois
elementos adjacentes é igual ao elemento da linha seguinte,
responde às seguintes questões.

(a) Desenha o triângulo de Pascal até à linha 4.


(b) Quais os maiores elementos da linha 3? Qual o maior elemento da
linha 4? Como é que estes três elementos se relacionam? (Escreve
as tuas respostas usando n Ck em vez de valores)
(c) Qual o maior elemento da linha 10? E quais os maiores elementos da
linha 9? Como é que estes três elementos se relacionam? (Escreve
as tuas respostas usando n Ck em vez de valores)
(d) Seja k um número ı́mpar. Sabe-se que os dois maiores elemento
linha de ordem k são iguais a 126. Qual é o valor do maior elemento
da linha k + 1?
(e) Seja k um número ı́mpar. Sabe-se que o maior elemento da linha
de ordem k + 1 é igual a 70. Qual é o valor do maior elemento da
linha k?

(5) Usando as propriedades do triângulo de Pascal, resolve as seguintes


questões:

(a) Sabe-se que uma linha do triângulo de Pascal tem 7 elementos. Qual
é a soma dos elementos da linha anterior?
(b) Sabe-se que a soma dos elementos de uma linha do triângulo de
Pascal é 128. Qual é a soma dos dois primeiros elementos da linha
seguinte?
(c) Sabe-se que a linha de ordem n do triângulo de Pascal satisfaz n C3 =
n
C10 . Quantos elementos da linha seguinte são maiores que n+1 C2 ?

Erro ou sugestão? Contacta-me: mailto:ricardoferreira.contactar@gmail.com126


Vê no YouTube: https://www.youtube.com/channel/UCkRcdeyQ50TWFmk7vyuzf_g
Ricardo Ferreira

Saber a ordem da linha do triângulo de Pascal é como saber o termo


geral nas progressões: sabes isso, sabes tudo!

(6) Considera a linha de ordem n do triângulo de Pascal.

(a) Supõe que a soma dos elementos dessa linha e da linha anterior é
12. Resolvendo a equação 2n + 2n−1 = 12, calcula n.
(b) Supõe agora que a soma dos elementos dessa linha e da linha seguinte
é 96. Resolvendo a respetiva equação, calcula n.
(c) Supõe agora que a soma dos elementos da linha anterior e da linha
seguinte é 40. Resolvendo a respetiva equação, calcula n.

15.2 Binómio de Newton


Umas das aplicações do triângulo de Pascal é o binómio de Newton. Aprende
tudo sobre este binómio em https://youtu.be/7Id1tJUNQT0?t=917.
No fundo,

(a + b)n = n C0 an + n C1 an−1 b + n C2 an−2 b2 + · · · + n Cn bn

Mas também podemos pensar em buffet! Por exemplo, consideremos


(a + b)2 , ou seja,

(a + b)(a + b) = a2 + 2ab + b2 .
Aqui temos um buffet de dois pratos: o primeiro prato é o primeiro
parênteses e o segundo prato é o segundo parênteses. Se não comermos esse
prato, então escolhemos a, e se comermos esse prato, então escolhemos b.
Assim, se pensarmos no termo com a2 , qual é o respetivo coeficiente?
Ora a2 significa “dos 2 pratos, comer 0 pratos” — e quantas maneiras há
de fazer isto? É claro: 2 C0 . Por isso é que o termo com a2 tem coeficiente
2
C0 = 1.
E se pensarmos no termo com ab, qual é o seu coeficiente? Ora ab
significa “dos 2 pratos, comer 1 prato” — e quantas maneiras há de fazer
isto? É claro: 2 C1 . Por isso é que o termo com ab tem coeficiente 2 C1 = 2.
E agora, no caso geral, em que temos (a + b)n ? Ora, este é um buffet
com n parênteses, ou seja, com n pratos. Neste caso, se pensarmos no
termo com an−k bk , qual é o seu coeficiente? Ora, an−k bk significa “dos n
pratos, comer k pratos” — e quantas maneiras há de fazer isto? É claro:
n
Ck ! Ou seja, o termo com ak bn−k tem coeficiente n Ck .
Se quisermos ser mais precisos, em vez de falarmos de pratos, podemos
falar em b’s. Se temos n parênteses, então temos n b’s. E o que significa
an−k bk ? Significa “desses n b’s, escolher k b’s.” Por isso é que o respetivo
coeficiente é n Ck .

Erro ou sugestão? Contacta-me: mailto:ricardoferreira.contactar@gmail.com127


Vê no YouTube: https://www.youtube.com/channel/UCkRcdeyQ50TWFmk7vyuzf_g
Ricardo Ferreira

(1) Vamos desenvolver


(x + 2)3
usando o binómio de Newton.

(a) Primeiro, descobrimos os coeficientes. Como o expoente é 3, então


os coeficientes são 3 C0 , 3 C1 , 3 C2 , 3 C3 . Descobre o valor destes coefi-
cientes desenhando a linha de ordem 3 do triângulo de Pascal.
(b) Agora, adicionamos o que falta. Usando a fórmula do binómio de
Newton (a + b)n = n C0 an + n C1 an−1 b + n C2 an−2 b2 + · · · + n Cn bn ,
com a = x e b = 2, desenvolve (x + 2)3 .

Primeiro? Escreve os coeficientes: n C0 , n C1 . . . , n Cn . Depois? Adiciona


o resto: an , an−1 b, . . . , bn .

(2) Vamos desenvolver


(x − 1)4
usando o binómio de Newton.

(a) Primeiro, como o expoente é 4, desenha a linha de ordem 4 do


triângulo de Pascal.
(b) Agora, usando a fórmula do binómio de Newton, com a = x e b =
−1, desenvolve (x − 1)4 .

Primeiro? Escreve os coeficientes: n C0 , n C1 . . . , n Cn . Depois? Adiciona


o resto: an , an−1 b, . . . , bn .

(3) Usando a fórmula do binómio de Newton

(a + b)n = n C0 an + n C1 an−1 b + n C2 an−2 b2 + · · · + n Cn bn ,

desenvolve:

(a) (x − 2)3 1 4
 
(e) x +
x
(b) (2x + 1)3 3
x 3 (f) x − x−1
(c) −1 (g) x2 + x1/2
5
3
√ 3
(d) (x2 + x)3 (h) 1 − 3

Erro ou sugestão? Contacta-me: mailto:ricardoferreira.contactar@gmail.com128


Vê no YouTube: https://www.youtube.com/channel/UCkRcdeyQ50TWFmk7vyuzf_g
Ricardo Ferreira

15.2.1 O termo de ordem k + 1


Na fórmula do binómio de Newton,

(a + b)n = n C0 an + n C1 an−1 b + n C2 an−2 b2 + · · · + n Cn bn ,

o termo de ordem 1 é o primeiro termo, ou seja, n C0 an , o termo de ordem 2


é o segundo termo, ou seja, n C1 an−1 b. Ou, regra geral, o termo de ordem
k + 1 é o (k + 1)-ésimo termo, ou seja,

n
Ck an−k bk

Falando em buffet, como o termo de ordem 1 é aquele em que comemos


0 pratos, então o termo de ordem k + 1 é aquele em que comemos k pratos,
ou seja, n Ck an−k bk .
É também importante lembrar que este termo geral só é diferente de 0
quando o k está no intervalo [0, n]. Afinal de contas, se tivermos n pratos, o
mı́nimo de pratos que podemos comer é 0 e o máximo de pratos que podemos
comer é n. Assim, temos 0 ≤ k ≤ n.
Mas afinal, para que serve este termo geral n Ck an−k bk ? Para lidar com
expoentes muito grandes, como

(a + b)13 .

Aqui, usar a fórmula do binómio de Newton seria doloroso. Mas se usarmos


o termo geral, então conseguimos extrair muita da informação e com muito
menos esforço!

(15) Considerando o termo geral de ordem k + 1, ou seja, n Ck an−k bk , vamos


extrair informção sobre os termos de

1 12
 
x2 + .
x

(a) Calcula o termo de ordem k + 1. Apresenta a tua resposta na forma


12
Ck x··· .
(b) Usando o termo de ordem k + 1 que obtiveste em (a), determina o
termo de ordem 3.
(c) Usando o termo de ordem k + 1, determina o termo central (ou seja,
o termo que está no centro).
(d) Usando o termo de ordem k + 1, determina, se existir, o termo de
grau 0, ou seja, o termo independente de x (o termo que não tem
nenhum x).
(e) Usando o termo de ordem k + 1, justifica porque é que não existe
um termo de grau 2 (ou seja, um termo com x2 ).

Erro ou sugestão? Contacta-me: mailto:ricardoferreira.contactar@gmail.com129


Vê no YouTube: https://www.youtube.com/channel/UCkRcdeyQ50TWFmk7vyuzf_g
Ricardo Ferreira

(f) Usando o termo de ordem k + 1, determina o termo de maior grau


(ou seja, o termo que tem x··· com o maior expoente).
(g) Usando o termo de ordem k + 1, determina o número de termos em
que o expoente de x é um número par.

Termo de ordem k + 1? É o (k + 1)-ésimo termo, ou seja, n Ck an−k bk .

(16) Seja n um número natural. Considera


 n
1 2
√ +x .
x

(a) Calcula o termo geral de ordem k + 1. Apresenta a tua resposta na


forma n Ck x··· .
(b) Supondo que o termo de ordem 8 é 36x13 , determina n.
(c) Supondo que o termo central é 70x9 , determina n.
(d) Supondo que n = 11, determina, caso exista, o termo independente
de x (ou seja, o termo sem nenhum x)
(e) Supondo que n = 10, determina, caso exista, o termo independente
de x (ou seja, o termo sem nenhum x).
(f) Determina o menor valor de n tal que exista um termo independente.
(g) Determina o menor valor de n tal que exista um termo tal que o
expoente de x seja −7/2.

15.3 Equações
Graças à simetria do triângulo de Pascal, ou seja, graças à propriedade de
que n Ck = n Cn−k , nós sabemos que a equação
n
Cx = n Ck ,

com incógnita x, tem duas soluções:

x = k ∨ x = n − k.

Seguindo a mesmo lógica, se tivermos


x
Cm = x Ck ,

com incógnita x, nós já sabemos que m = k ou m = x − k. Se m = k, então


qualquer x é uma solução. Mas se m 6= k, então temos uma única solução

x = m + k.

Erro ou sugestão? Contacta-me: mailto:ricardoferreira.contactar@gmail.com130


Vê no YouTube: https://www.youtube.com/channel/UCkRcdeyQ50TWFmk7vyuzf_g
Ricardo Ferreira

Por outro lado, para resolver equações do tipo


x
Cm = n Ck ,

n!
com incógnita x, basta usar a fórmula das combinações: n Ck = .
k!(n − k)!
Fica então
x! n!
=
m!(x − m)! k!(n − k)!
e é só resolver em ordem a x.
Resumindo,

n
Cx = n Ck :
x=k∨x=n−k
(
x x N ∪ {0}, se m = k
Cm = Ck : x =
m+k se m 6= k
x n!
Cm = n Ck : Usar n Ck = e resolver ordem a x.
k!(n − k)!

(1) Usando a fórmula e as propriedades de n Ck , determina os números in-


teiros não negativos que são soluções das seguintes equações.
525 525
(a) Cn = 1 (h) Cn + 525 Cn+1 = 526 Cn−1
525 526
(b) Cn = 525 (i) Cn − 525 Cn = 525
525
(c) Cn = 525 C2 (j) 525
Cn + 525 C525−n = 1050
525
(d) Cn = 525 C11 (k) 526
Cn − 525 C524−n = 525
12
(e) Cn = 12 C2n−3 (l) 526
C523−n − 525 C522−n = 525 C520
12
(f) Cn2 = 12 Cn (m) 3 Cn × 3 C3−n = 3 × 3 C3−n
525
(g) Cn−1 + 525 Cn = 1 (n) 3 Cn × 3 C3−n = 4 × 3 Cn − 3

Equação n Cx = n Ck ? Soluções x = k ∨ x = n − k (é só usar a simetria!)

(2) Usando a fórmula e as propriedades de n Ck , determina os números in-


teiros não negativos que são soluções das seguintes equações.
n
(a) C5 = n C7
n
(b) C2 = n C252
n−1
(c) C4 = n−1 C9
n2 2
(d) C9 = n C16
n+1
(e) C3 = n+1 Cn−3
n+1
(f) C3 = n C2 + n Cn−2
n+1
(g) C4 = 2 × n−1 C4 + 2 × n−1 Cn−4

Erro ou sugestão? Contacta-me: mailto:ricardoferreira.contactar@gmail.com131


Vê no YouTube: https://www.youtube.com/channel/UCkRcdeyQ50TWFmk7vyuzf_g
Ricardo Ferreira

Equação x Cm = x Ck ? Sabemos que m = k ou m = x − k. Logo, se


m 6= k, então x = m + k (é só somar os dois termos de baixo!)

(3) Usando a fórmula e as propriedades de n Ck , determina os números in-


teiros não negativos que são soluções das seguintes equações.
n
(a) C7 = 525 C7
n
(b) C525 = 7 C525
n
(c) C2 =3
n−1
(d) C2 = 3
2n+1
(e) C2 = 21
n2
(f) C2 = 36
n
(g) C325 = 525 C200
n
(h) C325 = 526 C201 − 525 C201
n+1
(i) C325 − n C325 = 524 C200 + 524 C323
n!
Equação x Cm = n Ck ? Usa a fórmula das combinações n Ck =
k!(n − k)!
e resolve em ordem a x.

(4) Seja n ≥ 3. Considera um polı́gono regular com n vértices. Determina


n, sabendo que:

(a) o número de segmentos de reta que podemos formar com os vértices


desse polı́gono é 15.
(b) o número de polı́gonos com n − 2 lados que podemos formar com os
vértices desse polı́gono é 21.
(c) o número de segmentos de reta que podemos formar com os vértices
desse polı́gono é igual ao número de quadriláteros que podemos
formar com os vértices de um hexágono regular.
(d) o número de quadriláteros que podemos formar com os vértices desse
polı́gono é igual ao número de octógonos que podemos formar com
os vértices desse polı́gono.

(5) Seja n ≥ 3. Considera um prisma regular, cujas bases são polı́gonos


regulares com n vértices. Determina n, sabendo que:

(a) o número de segmentos de reta que podemos formar com os vértices


desse polı́gono é 15.
(b) o número de segmentos de reta que podemos formar com os vértices
desse prisma e que sejam arestas desse prisma é 39.
(c) o número de segmentos de reta que podemos formar com os vértices
desse prisma e que não sejam arestas desse prisma é 16.

Erro ou sugestão? Contacta-me: mailto:ricardoferreira.contactar@gmail.com132


Vê no YouTube: https://www.youtube.com/channel/UCkRcdeyQ50TWFmk7vyuzf_g
Ricardo Ferreira

(d) o número de triângulos que podemos formar com os vértices desse


prisma e que não estejam contidos em nenhuma das bases é igual
ao quádruplo do número de arestas desse prisma.
(e) o número de segmentos de reta que podemos formar com os vértices
desse prisma e que estejam contidos numa das faces do prisma é
24.

Erro ou sugestão? Contacta-me: mailto:ricardoferreira.contactar@gmail.com133


Vê no YouTube: https://www.youtube.com/channel/UCkRcdeyQ50TWFmk7vyuzf_g
Ricardo Ferreira

16 Probabilidades — Regra de Laplace e Fórmulas


das Probabilidades
16.1 Probabilidades I: Regra de Laplace
A maneira mais básica de calcular probabilidade é

Casos Favoráveis
P (A) =
Casos Possı́veis

Por exemplo, se lançarmos um dado de 6 faces, qual é a probabilidade de


sair um número par?

ˆ Primeiro de tudo, casos possı́veis: são todos os resultados que pode-


mos obter se lançamos um dado de 6 faces. Ora, pode sair qualquer
um dos números de 1 a 6, por isso temos 6 casos possı́veis.

ˆ Agora, casos favoráveis: são todos os resultados em que sai um


número par. Ora, pode sair 2 ou 4 ou 6, por isso temos 3 casos
favoráveis.

Conclusão? A probabilidade de sair número par é casos favoráveis sobre


casos possı́veis, ou seja, 3/6 (ou simplificando, 1/2)!
No fundo, a regra de Laplace faz com que calcular probabilidades seja
um mero problema de Combinatória — basta calcular os casos favoráveis e
os casos possı́veis e dividir!
E já agora, de onde vem a regra de Laplace? Descobre tudo em https:
//youtu.be/MF0KriNT7A4?t=465.

(1) Considera uma pirâmide regular pentagonal, ou seja, uma pirâmide cuja
base é um pentágono regular. Escolhem-se, ao acaso, três vértices dessa
pirâmide.
Determina a probabilidade de o plano definido por esses três vértices
conter uma das faces da pirâmide. Apresenta a tua resposta na forma
de dı́zima finita.

(2) Escolhe-se, ao acaso, um número com cinco dı́gitos. Determina a prob-


abilidade de esse número ser simultaneamente:

ˆ um número par
ˆ uma capicua (um número que se lê de igual forma da esquerda para
a direita ou da direita para a esquerda)
ˆ um número maior ou igual a 27000.

Apresenta a tua resposta na forma de fração irredutı́vel.

Erro ou sugestão? Contacta-me: mailto:ricardoferreira.contactar@gmail.com134


Vê no YouTube: https://www.youtube.com/channel/UCkRcdeyQ50TWFmk7vyuzf_g
Ricardo Ferreira

(3) Uma turma de mestrado, composta por dois professores (um do sexo
masculino e um do sexo feminino) e sete alunos (dois do sexo masculino
e cinco do sexo feminino), vão tirar uma fotografia.
Para isso, os nove membros da turma vão posicionar-se lado a lado, di-
vididos em duas filas: três membros ficarão à frente, sentados, enquanto
que os outros seis membros ficarão atrás, em pé.
Sabendo que os nove membros da turma se posicionam ao acaso, de-
termina a probabilidade de os membros se posicionarem de modo que
simultanemante:

ˆ os dois professores fiquem em filas diferentes (um sentado e um em


pé),
ˆ os dois alunos do sexo masculino fiquem em pé e separados.

Apresenta a tua resposta na forma de dı́zima finita, arredondada às


centésimas.

16.2 Probabilidades II: Fórmulas e Tabela


16.2.1 Estratégia Geral
Exercı́cio de Probabilidades? E não dá para resolver pela regra de Laplace?
Então usa as fórmulas! Aprende de onde estas fórmulas vêm em https:
//youtu.be/MF0KriNT7A4?t=850.
No fundo, só precisamos de saber quatro:

P (A) = 1 − P (A)
P (A ∪ B) = P (A) − P (A ∪ B)
P (A ∪ B) = P (A) + P (B) − P (A ∩ B)
P (A ∩ B)
P (A|B) =
P (B)

Mas, felizmente, não precisamos de memorizar as duas primeiras — em vez


disso, vamos lembrar-nos delas fazendo uma tabela de dupla entrada.

A A

B P (A ∩ B) P A ∩ B P (B)
  
B P A∩B P A∩B P B

P (A) P A 1
Mas como fazemos uma tabela de dupla entrada? Aprende como em
https://youtu.be/MF0KriNT7A4?t=1952.

Erro ou sugestão? Contacta-me: mailto:ricardoferreira.contactar@gmail.com135


Vê no YouTube: https://www.youtube.com/channel/UCkRcdeyQ50TWFmk7vyuzf_g
Ricardo Ferreira

A A

B P (A ∩ B) P A ∩ B P (B)
  
B P A∩B P A∩B P B

P (A) P A 1
No fundo, a regra fundamental para preencher esta tabela é:

“o primeiro mais o segundo é igual ao terceiro”.

E é exatamente esta regra que esconde as tais duas fórmulas. 


Se olharmos para a última linha, vemos que P (A) + P A = 1 e se
olharmos para a última coluna, vemos que P (B) + P B = 1. Ou seja, a
tabela codifica a fórmula do complementar:

P (A) + P A = 1

E se olharmos para as duas linhas e para as duas colunas do meio, vemos


que a tabela também codifica a fórmula da diferença:

P (A ∩ B) + P A ∩ B = P (A)

No fundo,
 a tabela de dupla entrada  codifica as duas fórmulas P (A)+
P A = 1 e P (A ∩ B) + P A ∩ B = P (A) (ou seja, se construirmos uma
tabela de dupla entrada, já não precisamos de nos lembrar destas fórmulas!)

Erro ou sugestão? Contacta-me: mailto:ricardoferreira.contactar@gmail.com136


Vê no YouTube: https://www.youtube.com/channel/UCkRcdeyQ50TWFmk7vyuzf_g
Ricardo Ferreira

16.2.2 Como Calcular Probabilidades


E a partir da tabela, como calculamos probabilidades?

ˆ se quisermos calcular a probabilidade de um acontecimento (por exem-


plo, P (A)) ou de uma interseção (por exemplo P (A ∩ B)), é só olhar
para a tabela

ˆ se quisermos calcular uma união, é só usar a fórmula da união:


P (A ∪ B) = P (A) + P (B) − P (A ∩ B)

ˆ e se quisermos calcular uma condicionada, é só usar a fórmula da


condicionada: P (A|B) = P (A ∩ B)/P (B)

Mas cuidado! Também podemos preencher a tabela não com probabili-


dades mas sim com o número de casos favoráveis.

A A
B #A ∩ B #A ∩ B #B
B #A ∩ B #A ∩ B #B
#A #A #E
Neste caso, para calcular probabilidades, a estratégia é a mesma, mas
temos de dividir pelos casos possı́veis #E:

ˆ para um acontecimento, P (A) = #A/#E, e para uma interseção,


P (A ∩ B) = #A ∩ B/#E (é só olhar para a tabela)

ˆ para uma união, P (A ∪ B) = P (A) + P (B) − P (A ∩ B) (é só calcular


P (A),P (B), P (A ∩ B) pela tabela e usar a fórmula).

ˆ para uma condicionada, P (A|B) = #A ∩ B/#B (é só olhar para a


tabela)

A condicionada fica tão fácil de calcular porque

P (A ∩ B) #A ∩ B/#E #A ∩ B
P (A|B) = = = .
P (B) #B/#E #B

Erro ou sugestão? Contacta-me: mailto:ricardoferreira.contactar@gmail.com137


Vê no YouTube: https://www.youtube.com/channel/UCkRcdeyQ50TWFmk7vyuzf_g
Ricardo Ferreira

(1) Preenche as seguintes tabelas de dupla entrada, que contêm probabili-


dades.

A A A A
B 0.5 0.1 B
(a) (d)
B 0.2 0.2 B 1
1 0.1 1

A A A A
B 0.3 0.1 B 1/7
(b) (e)
B 0.4 B 2/7 1/7
1 1

A A A A
B 0.7 B 1/11
(c) (f)
B 0.2 B 3/11 6/11
0.8 1 1

“Primeiro mais o segundo é igual ao terceiro!” (por isso, se temos uma


linha ou uma coluna com um único espaço vazio, então conseguimos
preencher!)

(2) Dadas as seguintes probabilidades, constrói a respetiva tabela de dupla


entrada.
 
(a) P (A ∩ B) = 0.2, P A ∩ B = 0.3, P A ∩ B = 0.4.

(b) P A ∩ B = 0.1, P (A) = 0.7, P (B) = 0.8.
 
(c) P A ∩ B = 2/9, P (A) = 1/3, P B = 5/9.

Erro ou sugestão? Contacta-me: mailto:ricardoferreira.contactar@gmail.com138


Vê no YouTube: https://www.youtube.com/channel/UCkRcdeyQ50TWFmk7vyuzf_g
Ricardo Ferreira

(3) Considera a tabela de dupla entrada seguinte, que contém probabili-


dades.
A A
B 0.4 0.2 0.6
B 0.1 0.3 0.4
0.5 0.5 1

Olhando para a tabela, determina as seguintes probabilidades.


 
(a) P (A ∩ B) (e) P A (i) P A ∪ B
 
(b) P A ∩ B (f) P B (j) P (A|B)

(c) P A ∩ B (g) P (A ∪ B) (k) P (B|A)
 
(d) P (A) (h) P A ∪ B (l) P B|A

P (A) ou P (A ∩ B)? Vê na tabela! P (A ∪ B) ou P (A|B)? Usa as


fórmulas!

(4) Sabendo que P B = 0.6, P (A ∩ B) = 0.3 e P (A ∪ B) = 0.4, constrói a
respetiva tabela de dupla entrada e calcula P A ∩ B .
P (A) ou P (A ∩ B)? É direto! P (A ∪ B) ou P (A|B)? Usa as fórmulas!

(5) Sabendo que P B = 0.4, P (B|A) = 3/5 e P (A ∪ B)  = 0.9, constrói a
respetiva tabela de dupla entrada e calcula P A|B . Apresenta o teu
resultado na forma de fração irredutı́vel.
P (A) ou P (A ∩ B)? É direto! P (A ∪ B) ou P (A|B)? Usa as fórmulas!
Vê um exemplo em https: // youtu. be/ -TZ8kM0_ Or0? t= 2092

(6) Considera a tabela de dupla entrada seguinte, que contém casos fa-
voráveis.
A A
B 14 6 20
B 1 21 22
15 27 42

Olhando para a tabela, determina as seguintes probabilidades. Apre-


senta os teus resultados na forma de fração irredutı́vel.
 
(a) P (A ∩ B) (c) P A ∩ B (e) P A
 
(b) P A ∩ B (d) P (A) (f) P B

Erro ou sugestão? Contacta-me: mailto:ricardoferreira.contactar@gmail.com139


Vê no YouTube: https://www.youtube.com/channel/UCkRcdeyQ50TWFmk7vyuzf_g
Ricardo Ferreira


(g) P (A ∪ B) (i) P A ∪ B (k) P (B|A)
 
(h) P A ∪ B (j) P (A|B) (l) P B|A

P (A) ou P (A ∩ B)? Vê na tabela! P (A ∪ B) ou P (A|B)? Usa as


fórmulas!

(7) Num restaurante, trabalham diariamente 36 trabalhadores (homens e


mulheres). Diariamente, cada trabalhador tem direito a uma refeição
no restaurante: almoço ou jantar.
Num certo dia, sabe-se que:

ˆ Eram quatro os homens que almoçaram no restaurante.


ˆ Dois terços dos homens jantaram no restaurante.
ˆ Eram trinta e dois os trabalhadores que eram homens e/ou que
jantaram no restaurante.

Escolhe-se, ao acaso, um trabalhador que almoçou nesse dia. Deter-


mina a probabilidade de esse trabalhador ser homem. Apresenta o teu
resultado na forma de fração irredutı́vel.
Faz isto de duas maneiras:

(a) Determina essa probabilidade construindo uma tabela de dupla en-


trada com casos favoráveis.
(b) Determina essa probabilidade construindo uma tabela de dupla en-
trada com probabilidades.

Erro ou sugestão? Contacta-me: mailto:ricardoferreira.contactar@gmail.com140


Vê no YouTube: https://www.youtube.com/channel/UCkRcdeyQ50TWFmk7vyuzf_g
Ricardo Ferreira

17 Soluções
17.1 Geometria 1 — Produto escalar e comparação de planos
e retas
17.1.1 Colineares ou não colineares?
(1) (a) (2, 10, 6) = 2(1, 5, 3)
(b) 6/2 = 3, mas 5/1 = 5.
(c) Sim. (7, 9, 5) = −1(−7, −9, −5).

(2) (a) Os vetores diretores são colineares: (−6, 18, 21) = 23 (−4, 12, 14).
(b) Não, porque os vetores diretores são não colineares: −4/ − 2 = 2
mas 12/3 = 4.
(c) (x, y, z) = (3, 6, 9) + k(−2, 3, 7) (o vetor diretor pode ser qualquer
vetor colinear a (−2, 3, 7) e o ponto pode ser qualquer ponto que
satisfaz esta equação).

(3) (a) Os vetores normais são colineares: (2, 5, −7) = − 21 (−4, −10, 14).
(b) Não, porque os seus vetores normais (−4, −10, 14) e (−2, −5, 0) são
não colineares: −5/ − 10 = 1/2, mas 0/14 = 0.
(c) −2(x − 3) − 5(y − 2) + 0(z − 1) = 0, logo a equação reduzida é
−2x − 5y + 16 = 0.

(4) (a) O vetor diretor da reta e o vetor normal ao plano são colineares:
(−2, 8, −6) = − 32 (3, −12, 9).
(b) Não, porque o vetor diretor da reta e o vetor normal ao plano são
não colineares: 3/ − 4 = −3/4 mas 9/ − 9 = −1.
(c) (x, y, z) = (−1, 4, 3) + k(3, −12, 9).

17.1.2 Vetor perpendicular a dois vetores não colineares


(1) (5, −9, −5) (ou qualquer vetor colinear a este)

(2) (−19, 2, −4) (ou qualquer vetor colinear a este)

(3) (a) −1(x − 0) − 3(y − 0) + 2(z − 1) = 0, logo a equação reduzida é


−x − 3y + 2z − 2 = 0.
(b) −1(x − 1) − 3(y − 2) + 2(z − 1) = 0, logo a equação reduzida é
−x − 3y + 2z + 5 = 0.
(c) (x, y, z) = (2, 3, 5) + k(3, 5, −9).

Erro ou sugestão? Contacta-me: mailto:ricardoferreira.contactar@gmail.com141


Vê no YouTube: https://www.youtube.com/channel/UCkRcdeyQ50TWFmk7vyuzf_g
Ricardo Ferreira

17.1.3 Vetor perpendicular a outro


(1) O produto escalar dos dois vetores tem de ser igual a zero. Por exemplo:

(a) (2, −1)


(b) (0, 1)
(c) (4, −1, 0)
(d) (0, 1, 0)
(e) (1, 0, 0)

(2) (1, 1, 0) e (0, 0, 1).

(3) (a) O vetor diretor da reta e o vetor normal ao plano são perpendicu-
lares: (3, −5, 7) · (4, 1, −1) = 0.
(b) Por exemplo, r : (x, y, z) = (0, 0, 0) + k(5, 3, 0) e s : (x, y, z) =
(0, 0, 0) + k(7, 0, −3) (os vetores diretores das retas são perpendic-
ulares ao vetor normal do plano e são não colineares).
(c) Os vetores normais são perpendiculares: (3, −5, 7) · (5, 3, 0) = 0.
(d) Por exemplo, β : 5x + 3y = 0 e γ : 7x − 3z = 0 (os vetores
normais de β e γ são perpendiculares ao vetor normal de α e são
não colineares).

(4) (a) (1, 1, 0) e (0, 0, 1).


(b) Por exemplo, x + y = 0 e z = 0 (os vetores normais são perpendic-
ulares ao vetor diretor da reta e são não colineares).
(c) x + y − 7 = 0 e z − 7 = 0 (planos paralelos à reta r e que contêm
um ponto da reta r).
(d) Dois vetores colineares dão origem à mesma equação vetorial de
uma reta r — por isso só sabemos que o plano β contém a reta
r. Mas pelo exercı́cio anterior, existem dois planos diferentes que
contêm a reta r. Por outras palavras, saber que β contém a reta r
não é suficiente para determinar um único plano.

17.1.4 Produto escalar e ângulos

(1) (a) →

u ·→

v = u1 v1 + u2 v2 + u3 v3 = k→

u kk→

v k cos →−
u,→

 
\ v .
(b)  u v +u v +u v
cos →−
u,→


\ 1 1 2 2 3 3
v = .
k→

u kk→
−vk

(2) (a) B ÂC = cos(9/26) ≈ 1.22 radianos


 
(b) AB̂C = cos−1 √148 74√
86
≈ 0.86 radianos.

Erro ou sugestão? Contacta-me: mailto:ricardoferreira.contactar@gmail.com142


Vê no YouTube: https://www.youtube.com/channel/UCkRcdeyQ50TWFmk7vyuzf_g
Ricardo Ferreira

−−→ −→  
(c) BC · AC = 17. B ÂC = cos−1 √2617√34 ≈ 0.96 radianos.
Somando a amplitude dos ângulos do triângulo isósceles [ABC],
obtemos 3.14 ≈ π radianos, logo o nosso resultado está correto.
 
−−→
\ −→
(d) O ângulo B ÂC é agudo, logo cos AB, AC ≥ 0 e assim

 
−−→ −→ −−→ −→ −−→
\ −→
AB · AC = kABkkACk cos AB, AC ≥ 0

.
 
(3) (a) α = cos−1 √9426√78 ≈ 1.26 radianos.

 
(b) α = cos−1 − √9423√57 ≈ 1.25 radianos. (Cuidado!)

 
−1 √ 37√
(c) α = cos − 94 69 ≈ 1.09 radianos. (Cuidado!)

Erro ou sugestão? Contacta-me: mailto:ricardoferreira.contactar@gmail.com143


Vê no YouTube: https://www.youtube.com/channel/UCkRcdeyQ50TWFmk7vyuzf_g
Ricardo Ferreira

17.2 Geometria 2 — Lugares Geométricos


17.2.1 Equações e interseções de retas e planos
(1) (a) Por exemplo, (x, y, z) = (1, 2, 3) + k(4, 5, 6).
(b) Por exemplo, (x, y, z) = (1, 2, 3) + k(1, 1, 1).
(c) Por exemplo, pontos (−1, 7, 5), (−1, 6, 5), (−1, 358, 5) e vetores dire-
tores (0, −1, 0) e (0, −1411, 0).
(d) Por exemplo, (x, y, z) = (−1, 0, 5) + k(0, 1, 0) (as coordenadas do
ponto têm de respeitar x = −1 ∧ z = 5, e o vetor diretor tem de
ser colinear a (0, 1, 0)). O vetor diretor (0, 1, 0) é perpendicular aos
vetores normais dos dois planos. Isto porque se a reta pertence aos
dois planos, então o seu vetor diretor é paralelo aos dois planos e
por isso é perpendicular aos vetores normais desses planos.

(2) (a) k = 3/2.


(b) 9 = 7 + 0k não tem solução real k.
(c) Qualquer ponto da reta r satisfaz z = 7 e por isso (1, 2, 3) não
pertence à reta r.
(d) Por exemplo, (1, 0, 0), (2, 0, 0), (3, 0, 0) (colineares e z 6= 7)
(e) Por exemplo, (2, 5, −7) e (0, 3, 7) (k = 0 e k = 1)

(3) (a) 6 × (−1) + 3 × (−1) − 7 × (−1) + 2 = 0.


(b) 6 × 4 + 3 × 5 − 7 × (−1) + 2 = 50 6= 0.
(c) 6x + 3y − 7z − 48 = 0.

(4) (a) Temos k = 3/2, logo as coordenadas são (−9, 6, 12).


(b) A equação dá 4 = 0, logo não tem solução.
(c) Como o vetor diretor de s é perpendicular ao vetor normal a α,
então a reta s é paralela ao plano α, por isso a reta ou está contida
no plano α ou não o interseta. Mas como o ponto (−1, 0, −1) da
reta s não pertence ao plano α, então a reta s não interseta o plano
α.
(d) A equação dá 0 = 0, logo tem solução para todos os valores de k.
(e) Como o vetor diretor de t é perpendicular ao vetor normal a α,
então a reta s é paralela ao plano α, por isso a reta ou está contida
no plano α ou não o interseta. Mas como o ponto (0, 0, 0) pertence
à reta s e ao plano α, então a reta s está contida no plano α.

Erro ou sugestão? Contacta-me: mailto:ricardoferreira.contactar@gmail.com144


Vê no YouTube: https://www.youtube.com/channel/UCkRcdeyQ50TWFmk7vyuzf_g
Ricardo Ferreira

17.2.2 Superfı́cie Esférica


(1) x2 + (y + 6)2 + (z − 4)2 = 16.

(2) (x + 4)2 + (y − 7)2 + (z − 2)2 = 6.

(3) (x − 3)2 + (y − 2)2 + (z − 1)2 = 9.

(4) (x − 13)2 + (y + 9)2 + (z − 3)2 = 9.

(5) (x − 1)2 + (y − 5)2 + (z − 3)2 = 12.

17.2.3 Plano Tangente à Superfı́cie Esférica


(1) (a) z = 0
(b) z = 2
(c) y = 1
−−→
Em todos os casos, o vetor normal desse plano e o vetor CB são colin-
eares (ou seja, ”o raio da superfı́cie esférica é o vetor normal do
plano tangente”).

(2) 6x + y + 2z − 33 = 0

(3) (4, 0, 0).

17.2.4 Plano Mediador

(1) (a) z = 0 (b) x = 2 (c) x = y

−−→
Em todos os casos, o vetor normal a esse plano e o vetor AB são co-
lineares (ou seja, ”o segmento [AB] é o vetor normal ao plano
mediador de A e B”).

(2) x + 6y − 5z + 12 = 0

(3) (a) 2x + 4y − 7z + 12 = 0.
(b) 2x − 4y + 6z − 10 = 0

Erro ou sugestão? Contacta-me: mailto:ricardoferreira.contactar@gmail.com145


Vê no YouTube: https://www.youtube.com/channel/UCkRcdeyQ50TWFmk7vyuzf_g
Ricardo Ferreira

17.3 Trigonometria 1 — Definições e Equações


17.3.1 Trigonometria em ]0, π/2[
(1) (a) Ângulos iguais implica catetos 2
√ iguais o = a, logo 2a = 1, ou seja,
sin(π/4) = cos(π/4) = a = 2/2.
(b) Três ângulos iguais, logo três lados iguais. Como a é metade do
lado do triângulo maior, então cos(π/3) = a =√ 1/2. Pelo Teorema
2 2
de Pitágoras, o + a = 1, logo sen(π/3) = o = 3/2
(c) Adicionamos um triângulo igual ao primeiro abaixo deste. Três
ângulos iguais, logo três lados iguais. Como o é metade do lado
do triângulo maior, então sen(π/6) = a = √ 1/2. Pelo Teorema de
Pitágoras, o2 + a2 = 1, logo cos(π/6) = a = 3/2.

(d) √
tg α = sen α/ cos α, logo tg(π/6)) = 1/ 3, tg(π/4) = 1, tg(π/3) =
3.

17.3.2 Trigonometria em R
(1) (a) (r cos α, r sen α)
(b) x/ cos α = r/1, logo x = r cos α. y/ sen α = r/1, logo y = r sen α.
(c) Os pontos (r cos α, r sen α) satisfazem a equação x2 + y 2 = r2 .

17.3.3 Redução ao primeiro quadrante



(1) (a) sen(3π/4) = 2/2 (i) cos(2π/3) = −1/2

(b) sen(5π/4) = − 2/2 (j) cos(3π/6) = 0

(c) sen(−π/4) = − 2/2 (k) tg(3π/4) = −1

(d) sen(11π/4) = 2/2 (l) tg(5π/4) = 1
√ √
(e) cos(5π/6) = − 3/2 (m) tg(4π/3) = 3
√ √
(f) cos(7π/6) = − 3/2 (n) tg(7π/6) = 1/ 3

(g) sen(11π/6) = −1/2 (o) tg(5π/6) = −1/ 3

(h) sen(9π/6) = −1 (p) tg(−13π/3) = − 3

(2)

sen(π − α) = sen α sen(π + α) = −sen α sen(−α) = −sen α


cos(π − α) = − cos α cos(π + α) = − cos α cos(−α) = cos α
tg(π − α) = −tg α tg(π + α) = tg α tg(−α) = −tg α

Erro ou sugestão? Contacta-me: mailto:ricardoferreira.contactar@gmail.com146


Vê no YouTube: https://www.youtube.com/channel/UCkRcdeyQ50TWFmk7vyuzf_g
Ricardo Ferreira

(3) (a) sen(π/2 − α) = cos α, cos(π/2 − α) = sen α, tg(π/2 − α) = 1/tg α.


(b) Os dois ângulos agudos do triângulo retângulo são α e π/2 − α.
Se chamarmos de o e a os lados oposto e adjacente do triângulo,
então sen(π/2 − α) = a/h = cos α, cos(π/2 − α) = o/h = sen α,
tg(π/2 − α) = a/o = 1/tg α.
(c) cos(π/2 + α) = −sen α, sen(−π/2 − α) = − cos α e tg(−π/2 + α) =
−tg α:
ˆ cos(π/2 + α) = cos π − (π/2 − α) = − cos(π/2 − α) = −sen α


ˆ sen(−π/2 − α) = sen − (π/2 + α) = −sen(π/2 + α) = − cos α




ˆ tg(−π/2 + α) = tg − (π/2 − α) = −tg(π/2 − α) = −1/tg α




17.3.4 Equações Trigonométricas


(1) Seja k ∈ Z.

(a) x = kπ
(b) x = π/2 + kπ
(c) x = kπ
(d) x = π/2 + 2kπ
(e) x = π + 2kπ
(f) x = π/6 + 2kπ ∨ x = 5π/6 + 2kπ
(g) x = 4π/3 + 2kπ ∨ x = 5π/3 + 2kπ
(h) x = 5π/12 + 2kπ ∨ x = 7π/12 + 2kπ
(i) x = π/6 + 2kπ ∨ x = −π/6 + 2kπ
(j) x = 5π/6 + 2kπ ∨ x = −5π/6 + 2kπ
(k) x = −π/521 + 2kπ ∨ x = π/521 + 2kπ
(l) x = π/4 + kπ
(m) x = π/6 + kπ
(n) x = 2π/3 + kπ
(o) x = π/33 + kπ
(p) x = 5π/14 + 2kπ ∨ x = 9π/14 + 2kπ
(q) x = −π/7 + 2kπ ∨ x = 8π/7 + 2kπ
(r) x = π/3 + 2kπ ∨ x = −π/3 + 2kπ

(2) x = π/6 + 2kπ/3 ∨ x = π/2 + 2kπ, k ∈ Z .

(3) Seja k ∈ Z.

(a) x = −π/2, π/6

Erro ou sugestão? Contacta-me: mailto:ricardoferreira.contactar@gmail.com147


Vê no YouTube: https://www.youtube.com/channel/UCkRcdeyQ50TWFmk7vyuzf_g
Ricardo Ferreira

(b) x = −7π/16, −π/8, π/16


(c) x = −2π/3, −2π/15, 4π/15

(4) x = π/3 + kπ, k ∈ Z

Erro ou sugestão? Contacta-me: mailto:ricardoferreira.contactar@gmail.com148


Vê no YouTube: https://www.youtube.com/channel/UCkRcdeyQ50TWFmk7vyuzf_g
Ricardo Ferreira

17.4 Trigonometria 2 – Tangente e Declive


17.4.1 Visualizando a tangente
(1) (a) Tem soluções reais.
(b) Soluções: x = kπ, k ∈ Z.

(2) (a) A : (−2, −2 3)
√ √
(b) B : (4, 4 3). O declive é 3. (Cuidado!)

(c) Área = 12 3.
p
(3) (a) A abcissa do ponto A é 1/(t2 + 1).
p
(b) A ordenada do ponto A é t2 /(t2 + 1).
(c) 1 + tg2 α = 1/ cos2 α, 1/tg2 α + 1 = 1/sen 2 α.

17.4.2 Inclinação e Declive


√ √
(1) y = (−1/ 3)x − (1/ 3)

(2) (a) Declive = − 3

(b) r : y = − 3x + 4

(c) Área = 4 3.

17.4.3 Declive da reta perpendicular


√ √
(1) (a) y = (1/ 3)x + (4/ 3).
(b) y = (−1/2)x.

(2) (a) (x − 6)2 + (y − 3)2 = 25


(b) BD : y = (4/3)x − 5
(c) D : (3, −1)

Erro ou sugestão? Contacta-me: mailto:ricardoferreira.contactar@gmail.com149


Vê no YouTube: https://www.youtube.com/channel/UCkRcdeyQ50TWFmk7vyuzf_g
Ricardo Ferreira

17.5 Complexos 1 – Forma Algébrica e Trigonométrica


17.5.1 Forma Algébrica e Trigonométrica

(1) (a) Sim (f) Não


(b) Sim (g) Sim
(c) Não
(d) Não (h) Sim
(e) Não (i) Sim

(2) (a) Sim (f) Não


(b) Não (g) Não
(c) Não (h) Não
(d) Sim (i) Sim
(e) Não (j) Não

17.5.2 Somar e Multiplicar nos complexos


(1) (a) 11 + 2i
(b) 18 − 14i
(c) 5 − 12i
(d) 13i

(2) (a) ei(5π/6)


(b) ei(π/12)
(c) 2e−i(4π/21)
(d) 30e−i(12π/35)
(e) 6

(3) (a) Terceiro quadrante.


√ √
(b) ˆ −4 2 + 4 2i
ˆ 8ei(3π/4)

17.5.3 Simétrico: multiplicar por −1 (como subtrair)


√ √
(1) ˆ (−1/ 2) − (1/ 2)i
ˆ ei(5π/4) .

(2) (a) ei(6π/5)


(b) 2ei(3π/7)
(c) 3ei(π/2)

Erro ou sugestão? Contacta-me: mailto:ricardoferreira.contactar@gmail.com150


Vê no YouTube: https://www.youtube.com/channel/UCkRcdeyQ50TWFmk7vyuzf_g
Ricardo Ferreira

17.5.4 Potências de i: multiplicar por i


(1) i2 = −1, i3 = −i, i4 = 1

(2) i521 = i

(3) (a) i
(b) i
(c) −1
(d) −i

17.5.5 Conjugado (como dividir complexos)


(1) (a) (Fácil)
(b) (Fácil)

(2) (a) 1−i (g) i


(b) 7 + 2i (h) −1
(c) −6 + 7i
(i) 2e−i(π/4)
(d) −2 + 3i
(e) sen(π/16) − i cos(π/6) (j) 5ei(3π/7)
(f) −i (k) 3e−i(5π/3)

(3) (a) (1/5) − (2/5)i


(b) (4/25) + (3/25)i
(c) −(7/53) + (2/53)i
(d) −(7/2) + (1/2)i
(e) (8/13) − (14/13)i
(f) −i

17.5.6 Passar algébrica ↔ trigonométrica


(1) (a) i
(b) −i
(c) −1
(d) −1

(e) −( 3/2) − (1/2)i

(f) −(1/2) + ( 3/2)
(g) 2 cos(π/7) + i 2sen(π/7)
(h) 3 cos(6π/5) + i 2sen(6π/5)

Erro ou sugestão? Contacta-me: mailto:ricardoferreira.contactar@gmail.com151


Vê no YouTube: https://www.youtube.com/channel/UCkRcdeyQ50TWFmk7vyuzf_g
Ricardo Ferreira

√ √
(2) −1 + i = 2ei(3π/4) e 1 − i = 2ei(7π/4) .

(3) (a) (7 2)ei(π/4)

(b) (5 2)ei(5π/4)
(c) 4ei(4π/3)
(d) 4ei(π/3)

(e) (2 3)ei(2π/3)
(f) 6ei(7π/6)

17.5.7 Complexos e Trigonometria


(1) ei(6π/7)

(2) (a) ei(π/12) (g) 7ei(π/8)


(b) 4ei(π/12) (h) ei(π/12)
(c) 2e−i(π/12)
(i) ei(6π/7)
(d) ei(π/12)
(e) 3ei(5π/12) (j) e−i(7π/13)
(f) 7ei(5π/12) (k) ei(6π/7)

(3) ei(π/10)

17.5.8 Recapitulando

(1) (a) (1/4 2)e−i(π/4)
(b) 2ei(π/3)
√ √
(2) (3 3/2 − 3) + (3 3 + 3/2)i

Erro ou sugestão? Contacta-me: mailto:ricardoferreira.contactar@gmail.com152


Vê no YouTube: https://www.youtube.com/channel/UCkRcdeyQ50TWFmk7vyuzf_g
Ricardo Ferreira

17.6 Complexos 2 – Regiões e Equações


17.6.1 Regiões no Plano Complexo
(1) ˆ a≤x≤b
ˆ [a, b]
ˆ |x − (a + b)/2| ≤ (b − a)/2.

(2)

Erro ou sugestão? Contacta-me: mailto:ricardoferreira.contactar@gmail.com153


Vê no YouTube: https://www.youtube.com/channel/UCkRcdeyQ50TWFmk7vyuzf_g
Ricardo Ferreira

(3) (a) Im(z) < 0 ∧ Re(z) < 0


(b) Im(z) = 0 ∧ Re(z) > 0
(c) 2 ≤ Im(z) ≤ 4 ∧ 1 ≤ Re(z) ≤ 3
(d) −4 ≤ Im(z) ≤ 0 ∧ 0 ≤ Re(z) ≤ 6

(4)

Erro ou sugestão? Contacta-me: mailto:ricardoferreira.contactar@gmail.com154


Vê no YouTube: https://www.youtube.com/channel/UCkRcdeyQ50TWFmk7vyuzf_g
Ricardo Ferreira

(5) (a) Im(z) ≤ Re(z) ∧ Im(z) ≥ 0 ∧ Re(z) ≤ 3


(b) Im(z) ≤ Re(z) + 4 ∧ Im(z) ≤ −2Re(z) + 4 ∧ Im(z) ≥ −3.
(c) −5 ≤ Re(z) ≤ 0 ∧ (1/5)Re(z) + 1 ≤ Im(z) ≤ (1/5)Re(z) + 3.
(d) Im(z) ≤ −Re(z) + 4 ∧ Im(z) ≤ 4 ∧ Im(z) ≥ −2Re(z) − 8 ∧ Im(z) ≥
Re(z) − 8 ∧ Re(z) ≤ 4.

(6)

(7)

Erro ou sugestão? Contacta-me: mailto:ricardoferreira.contactar@gmail.com155


Vê no YouTube: https://www.youtube.com/channel/UCkRcdeyQ50TWFmk7vyuzf_g
Ricardo Ferreira

(8)

(9) (a) ˆ Im(z) > 0 ∧ Re(z) > 0


ˆ 0 < Arg(z) < π/2
(b) ˆ Im(z) < 0 ∧ Re(z) > 0
ˆ −π/2 < Arg(z) < 0
(c) ˆ Im(z) = 0
ˆ Arg(z) = 0 ∨ Arg(z) = π ∨ |z| = 0
(d) ˆ Im(z) > 0 ∧ Re(z) = 0
ˆ Arg(z) = π/2
(e) ˆ Im(z) = Re(z)
ˆ Arg(z) = π/4 ∨ Arg(z) = 5π/4 ∨ |z| = 0
(f) ˆ Im(z) = (tg α)Re(z)
ˆ Arg(z) = α ∨ Arg(z) = α + π ∨ |z| = 0
(g) ˆ Im(z) ≥ 0 ∧ Im(z) ≤ Re(z)
ˆ 0 ≤ Arg(z) ≤ π/4 ∨ |z| = 0
√ √
(h) ˆ Im(z) ≥ (1/ 3)Re(z) ∧ Im(z) ≥ − 3Re(z)
ˆ π/6 ≤ Arg(z) ≤ 2π/3 ∨ |z| = 0

Erro ou sugestão? Contacta-me: mailto:ricardoferreira.contactar@gmail.com156


Vê no YouTube: https://www.youtube.com/channel/UCkRcdeyQ50TWFmk7vyuzf_g
Ricardo Ferreira

(10) (a) Dica: Mostra que w satisfaz Re(w) < 0 ∧ Im(w) > −Re(w)
(b) Terceiro quadrante.
(c) Dica: Mostra que z satisfaz Arg(z) ∈]π/2, 2π/3[.

17.6.2 Equações
√ √
(1) (a) 0, i, ( 3/2) − (1/2)i, −( 3/2) − (1/2)i

(b) Área = 3 3/4.

(2) (a) z = (5/2) − (5/2)i


(b) z ∈ {0, 3i}
(c) z ∈ {i, −2i}
(d) z ∈ {0, 1 + i, −1 − i}
√ √
(e) z ∈ {−2, 1 − 3i, 1 + 3i}
(f) z ∈ {−2, 2}
(g) z ∈ {0, 4, 4ei(2π/5) , 4ei(4π/5) , 4ei(6π/5) , 4ei(8π/5) }
√ √ i(7π/32) √ i(15π/32)
(h) z ∈√{0, (1/ 2)ei(−π/32) √ , (1/ 2)e √ , (1/ 2)e ,
(1/√2)ei(23π/32) , (1/√2)ei(31π/32) , (1/ 2)ei(39π/32) ,
(1/ 2)ei(47π/32) , (1/ 2)ei(55π/32) }
(i) z ∈ {0, 2ei(π/35) , 2ei(15π/35) , 2ei(29π/35) , 2ei(43π/35) , 2ei(57π/35) }
(j) União do eixo real e do eixo imaginário: Im(z) = 0 ∨ Re(z) = 0
(k) z ∈ {0, 2}

17.6.3 Radiciação: a equação especial z n = w


(1) (Representar graficamente as soluções é fácil)

(a) z ∈ {1, ei(2π/3) , ei(4π/3) }


(b) z ∈ {−1, ei(5π/3) , ei(π/3) }
(c) z ∈ {eπ/21 , ei(15π/21) , ei(29π/21) }
(d) z ∈ {−1, ei(7π/5) , ei(9π/3) , ei(π/5) , ei(3π/5) }
(e) z ∈ {ei(π/4) , ei(3π/4) , ei(5π/4) , ei(7π/4) }
√ √ √ √
(f) z ∈ { 4 3, 4 3ei(π/2) , 4 3eiπ , 4 3ei(3π/2) }
√ √ √ √
(g) z ∈ { 4 7ei(π/16) , 4 7ei(3π/16) , 4 7ei(5π/16) , 4 7ei(7π/16) }
(h) Nas equações z n = reiθ , as soluções têm módulo r1/n . O argumento
é (θ/n) + 2kπ/n. Temos assim n soluções.

(2) Dica: As quatro soluções são simétricas duas a duas.

Erro ou sugestão? Contacta-me: mailto:ricardoferreira.contactar@gmail.com157


Vê no YouTube: https://www.youtube.com/channel/UCkRcdeyQ50TWFmk7vyuzf_g
Ricardo Ferreira

(3) (a) z 3 = 1
(b) z 3 = 125
(c) z 3 = 27ei(3π/4)
(d) z 4 = −4
(e) z 5 = ei(20π/7)
(f) z 6 = −1

(4) 2ei(8π/7) .

(5) (a) k = −3.


(b) k = 0.
(c) k = 1.
(d) k = −2.

17.6.4 Recapitulando
(1) (a) Eixo imaginário.
(b) Dica: escreve z na forma algébrica
(c)

(d) Dica: O eixo imaginário é definido por Re(z) = 0.


(e) Dica: Vê o que acontece quando fazes o conjugado de um ima-
ginário puro.
(f) Dica: escreve z na forma algébrica

Erro ou sugestão? Contacta-me: mailto:ricardoferreira.contactar@gmail.com158


Vê no YouTube: https://www.youtube.com/channel/UCkRcdeyQ50TWFmk7vyuzf_g
Ricardo Ferreira

(g)

(h) Dica: O eixo real é definido por Im(z) = 0.


(i) (Fácil)
(j) Dica: Vê o que acontece quando fazes o conjugado de um número
real.

(2) (a) |z − 2 − 2i| ≤ 1 ∨ |z + 2 − 2i| ≤ 1 ∧ 0 ≤ Arg(z) ≤ π/4 ∨ Im(z) ≤
0 ∧ 2 ≤ |z| ≤ 3
√ i(π/4) √ i(3π/4) √ i(5π/4)
(b) Dica:
√ Verifica que as raı́zes (2 2)e , (2 2)e , (2 2)e ,
(2 2)ei(7π/4) respeitam a condição em (a).
(c) θ = 0

Erro ou sugestão? Contacta-me: mailto:ricardoferreira.contactar@gmail.com159


Vê no YouTube: https://www.youtube.com/channel/UCkRcdeyQ50TWFmk7vyuzf_g
Ricardo Ferreira

17.7 Sucessões — Progressões, Limites e Propriedades


17.7.1 Progressões Aritméticas
(1) (a) un = −5 + 3(n − 1)
(b) u12 = 28
(c) Ordem 17
(d) 85
(e) 87

(2) Ordem 10.

(3) wn = −11 + 4n

17.7.2 Progressões Geométricas


(1) (a) un = (5/4) × 2n−1
(b) u11 = 1280
(c) Ordem 9.
(d) 1275/4
(e) 635

(2) vn = (3/2) × (−2)n .

(3) 26/3

(4) a = 10, b = 6

17.7.3 Limites de sucessões

(1) (a) 0 (i) +∞ (q) +∞


(b) 0 (j) +∞
(r) 4/3
(c) 0 (k) 0
(d) 0 (l) Não existe (s) 0
(e) 2 (m) +∞ (t) 0
(f) +∞ (n) −∞
(u) 0
(g) +∞ (o) −∞
(h) +∞ (p) 0 (v) +∞

Erro ou sugestão? Contacta-me: mailto:ricardoferreira.contactar@gmail.com160


Vê no YouTube: https://www.youtube.com/channel/UCkRcdeyQ50TWFmk7vyuzf_g
Ricardo Ferreira

(2) (a) e2 (h) e−2/3 (o) +∞


(b) e− 2 (i) e2 (p) e5
(c) e1/5 (j) e−6 (q) e1/2
(d) e (k) +∞ (r) +∞
(e) e2 (l) 0 (s) 0
(f) e1/2 (m) e (t) Não existe
(g) e−1/2 (n) e1/2 (u) 1

17.7.4 Propriedades das Sucessões

(1) (a) Crescente (f) Crescente


(b) Decrescente (g) Decrescente
(c) Crescente
(d) Decrescente (h) Decrescente
(e) Crescente (i) Decrescente

(2) (a) (a) +∞ (d) 0 (g) −1


(b) −∞ (e) 0 (h) −1/3
(c) +∞ (f) 2 (i) 0

(b) (a) [1, +∞[ (d) [0, 1] (g) [−1, −1/4]


(b) ] − ∞, −1] (e) [−1, 0] (h) [−1/3, 1/2]

(c) [1, +∞[ (f) [1, 2] (i) [0, 2 − 1]

(3) (a) ˆ un+1 − un é positivo em n = 1 mas negativo em n = 2.


ˆ u1 < u2 mas u2 > u3
(b) ˆ un+1 − un é positivo em n = 1 mas negativo em n = 2.
ˆ u1 < u2 mas u2 > u3
(c) ˆ un+1 − un é negativo em n = 1 mas positivo em n = 2.
ˆ u1 > u2 mas u2 < u3
(4) Não monótona, limitada por [−7, 1] e convergente para 0.
(5) Não monótona, limitada por [1/2, 2] e divergente (não existe limite).
(6) (a) Uma sucessão monótona e limitada é sempre convergente.
(b) Não monótona, limitada e convergente.

(7) (a) (−1)n (d) 1/n (g) 2 − 1/n


(b) 1/n (e) −1/n (h) (−1)n /n
(c) −1/n (f) 2 + 1/n

Erro ou sugestão? Contacta-me: mailto:ricardoferreira.contactar@gmail.com161


Vê no YouTube: https://www.youtube.com/channel/UCkRcdeyQ50TWFmk7vyuzf_g
Ricardo Ferreira

(8) (a) 6
(b) 2
(9) 3

Erro ou sugestão? Contacta-me: mailto:ricardoferreira.contactar@gmail.com162


Vê no YouTube: https://www.youtube.com/channel/UCkRcdeyQ50TWFmk7vyuzf_g
Ricardo Ferreira

17.8 Funções 1 — Funções Clássicas e Equações


Para verificar as respostas que envolvam desenhar gráficos, usa a tua calcu-
ladora gráfica ou visita:
ˆ https://www.desmos.com/calculator (Desmos)
ˆ https://www.geogebra.org/calculator (Geogebra)

17.8.1 As funções clássicas


(1) (Vê calculadora)
(2) (Vê calculadora)
(3) (Vê calculadora)
(4) (Vê calculadora) Nota: Crescente a negrito significa que a função é
crescente em todo o seu domı́nio.

(a) D = R, D0 = R, Zeros: x = 0, Negativa: ]−∞, 0[, Positiva: ]0, +∞[,


Crescente em R.
(b) D = R, D0 = [0, +∞[, Zeros: x = 0, Positiva: R\{0}. Decrescente
em ] − ∞, 0[, Crescente em ]0, +∞[, Mı́ximo y = 0.
(c) D = [0, +∞[x, D0 = [0, +∞[, Zeros: x = 0, Positiva: R\{0}. Cres-
cente em ]0, +∞[, Mı́nimo y = 0.
(d) D = R, D0 = R, Zeros: x = 0, Negativa: ]−∞, 0[, Positiva: ]0, +∞[,
Crescente em R, Máximo/mı́nimo y = 0.
(e) D = R, D0 = R, Zeros: x = 0, Negativa: ]−∞, 0[, Positiva: ]0, +∞[,
Crescente em R.
(f) D = R, D0 =]0, +∞[, Positiva: R, Crescente em R.
(g) D =]0, +∞[, D0 = R, Zeros: x = 1, Negativa: ]0, 1[, Positiva:
]1, +∞[, Crescente em ]0, +∞[.
(h) D = R\{0}, D0 = R\{0}, Negativa: ] − ∞, 0[, Positiva: ]0, +∞[.
Decrescente em ]0, +∞[, Crescente em ] − ∞, 0[.
(i) D = R, D0 = [−1, 1], Zeros: x = kπ, Negativa: ] − π + 2kπ, 2kπ[,
Positiva: ]2kπ, π + 2kπ[. Decrescente em ]π/2 + 2kπ, 3π/2 + 2kπ[,
Crescente em ] − π/2 + 2kπ, π/2 + 2kπ[, Máximo: y = 1, Mı́nimo:
y = −1.
(j) D = R, D0 = [−1, 1], Negativa: ]π/2 + 2kπ, 3π/2 + 2kπ[, Positiva:
] − π/2 + 2kπ, π/2 + 2kπ[. Decrescente em ]2kπ, π + 2kπ[, Crescente
em ] − π + 2kπ, 2kπ[, Máximo: y = 1, Mı́nimo: y = −1.
(k) D = R\{π/2 + kπ}, D0 = R, Negativa: ]π/2 + kπ, π + kπ[, Positiva:
]kπ, π/2 + kπ[. Crescente em R\{π/2 + kπ}.
(l) D = R, D0 = [0, +∞[, Zeros: x = 0, Positiva: R\{0}. Decrescente
em ] − ∞, 0[, Crescente em ]0, +∞[, Mı́nimo y = 0.

Erro ou sugestão? Contacta-me: mailto:ricardoferreira.contactar@gmail.com163


Vê no YouTube: https://www.youtube.com/channel/UCkRcdeyQ50TWFmk7vyuzf_g
Ricardo Ferreira

17.8.2 Funções compostas


(1) (a) g ◦ f (x) = −2x2 , f ◦ g(x) = 4x2
(b) g ◦ f (x) = 1/x3 − 1/3x, f ◦ g(x) = 1/(x3 − 3x)
2 +2x
(c) g ◦ f (x) = e2x + 2ex , f ◦ g(x) = ex
2 +2x
(d) g ◦ f (x) = e2x + 2ex , f ◦ g(x) = ex
(e) g ◦ f (x) = etan x , f ◦ g(x) = tan(ex )
(f) g ◦ f (x) = x2 , f ◦ g(x) = x2

(2) Dg◦f =]0, +∞[, que é diferente do domı́nio de x.


0
(3) (a) g ◦ f (x) = x, Dg◦f =]0, +∞[, Dg◦f =]0, +∞[
0
(b) g ◦ f (x) = x, Dg◦f = R, Dg◦f =R
0
(c) g ◦ f (x) = |x + 2| , Dg◦f = R, Dg◦f =R
0
(d) g ◦ f (x) = |ln x| , Dg◦f =]0, +∞[, Dg◦f = [0, +∞[
√ 0
(e) g ◦ f (x) = 1/ x, Dg◦f = R, Dg◦f = [0, 1]
0
(f) g ◦ f (x) = 1/ |x| , Dg◦f = R\{0}, Dg◦f =]0, +∞[
0
(g) g ◦ f (x) = 1/x2 , Dg◦f = R\{0}, Dg◦f = R\{0}
√ 0
(h) g ◦ f (x) = 1/ x, Dg◦f =]0, +∞[, Dg◦f =]0, +∞[

17.8.3 Calcular domı́nios


(1) Seja k ∈ Z um qualquer número inteiro.

(a) x 6= 2 (g) x 6= kπ/2


(b) x 6= 2 (h) x ≥ 3
(c) x 6= 1 ∧ x 6= 2 (i) x ≥ 3
(d) x 6= kπ/2 (j) x > −2
(e) x 6= π/6 + kπ/3 (k) x > −2
(f) x 6= 1/(π/2 + kπ) ∧ x 6= 0 (l) x 6= 1/2 + k/2

17.8.4 Funções Inversas I: logaritmos e equações


(1) Dica: Simplica a expressão fazendo e (ou a) elevado ao lado esquerdo,
e (ou a) elevado ao lado direito, e demonstra essa expressão.

(2) Dica: Utiliza o truque · · · = eln(··· ) .

(3) log2 (2) = 1, log2 (4) = 2 e por isso log2 (2 + 2) 6= log2 (2) log2 (2).

(4) (ln e)2 6= 2 ln e

Erro ou sugestão? Contacta-me: mailto:ricardoferreira.contactar@gmail.com164


Vê no YouTube: https://www.youtube.com/channel/UCkRcdeyQ50TWFmk7vyuzf_g
Ricardo Ferreira

(5) Resolvendo a equação, dá x = −1, mas o domı́nio da condição é x > 0.

(6) Equação com logaritmos: x = 0, equação com raı́z quadrada: x = ±1.



Fizemos os mesmos passos, só que substituindo ln x por x e ex por x2 .

(7) x = 4.

(8) Resolve as seguintes equações.



(a) x = (ln 5)/2 (k) x = ± 2
(b) x = (ln 5) (l) x = ln 2
(c) x = e−1 (m) x = −1
(d) x = e−1
(n) x = 0
(e) x = e−1
(o) x = ln 4
(f) x ∈ {e−1 , 1}
(p) x = log5 (4)
(g) Sem solução
(h) x = ln 2 (q) x = log3 (4)
(i) x = ±2 (r) x = log3 (2)
(j) Sem solução (s) x ∈ {e, e4 }

(9) x = 5/(log3 (2) + 1)

(10) Df =]3, 4[∪]4, +∞[

Erro ou sugestão? Contacta-me: mailto:ricardoferreira.contactar@gmail.com165


Vê no YouTube: https://www.youtube.com/channel/UCkRcdeyQ50TWFmk7vyuzf_g
Ricardo Ferreira

17.9 Funções 2 — Transformações, Inequações, Sinal e Zeros


Para verificar as respostas que envolvam desenhar gráficos, usa a tua calcu-
ladora gráfica ou visita:

ˆ https://www.desmos.com/calculator (Desmos)

ˆ https://www.geogebra.org/calculator (Geogebra)

17.9.1 Transformações de funções I: como visualizar


(1) (Vê calculadora)

(2) (Vê calculadora)

(3) (Vê calculadora)

(4) (a) π (e) 2π


(b) π/2
(f) 2π/3
(c) 2
(d) 4π (g) 2π/3

(5) (a) D = [−1, 1], D0 = [−1, 1]


(b) D = [−1, 1], D0 = [2, 4]
(c) D = [−1, 1], D0 = [−2, 2]
(d) D = [−1, 1], D0 = [1, 5]
(e) D = [−1, 1], D0 = [−9, −1]
(f) D = [2, 4], D0 = [−1, 1]
(g) D = [−1/2, 1/2], D0 = [−1, 1]
(h) D = [5/2, 7/2], D0 = [−1, 1]
(i) D = [14/3, 16/3], D0 = [−1, 1]
(j) D = [14/3, 16/3], D0 = [−9, −1]
(k) D = [5/3, 7/3], D0 = [−3, 5]

(6) (Vê calculadora)

17.9.2 Transformações de funções II: funções pares e ı́mpares

(1) (a) Ímpar (e) Ímpar (i) Não


(b) Par (f) Ímpar
(j) Par
(c) Ímpar (g) Par
(d) Par (h) Ímpar (k) Par e Ímpar

Erro ou sugestão? Contacta-me: mailto:ricardoferreira.contactar@gmail.com166


Vê no YouTube: https://www.youtube.com/channel/UCkRcdeyQ50TWFmk7vyuzf_g
Ricardo Ferreira

(2) f ◦ f (x) = x4 par, f ◦ g(x) = g ◦ f (x) = x6 par, g ◦ g(x) = x9 ı́mpar

(3) Dica: calcula f (−x) e coloca e−x em evidência.

(4) (a) Zero em x = −1, Crescente


(b) Zero em x = −1, Decrescente

17.9.3 Funções Inversas II: Como calcular


(1) (a) (Vê calculadora)
(b) f −1 (x) = x + 2
(c) (Fácil)

(2) Considera a função bijetiva g(x) = 2x − 4.

(a) (Vê calculadora)


(b) g −1 (x) = (x/2) + 2
(c) (Fácil)

(3) (a) (Vê calculadora)


(b) (
1/x se x < 0
h−1 (x) =
x2 se x ≥ 0

(c) Dica: verifica h ◦ h−1 (x) = x e h−1 ◦ h(x) = x separadamente para


cada ramo.

(4) (a) f −1 (x) = 1/(2x − 8) (d) f −1 (x) = ln(ex − 2)


2 −1
(b) f −1 (x) = (5 − 6x)/(3 − 2x) (e) f −1 (x) = ey

(c) f −1 (x) = ln (5−6x)/(3−2x) (f) f −1 (x) = −2 −

x

(5) Df −1 = [−1, 1], Df0 −1 =] − ∞, 4]

17.9.4 Resolver inequações


Dica: Podes ver o gráfico da tua função na calculadora gráfica — assim é
fácil verificar se os sinais estão certos.

(1) (a) x ∈] − ∞, −1/2] (e) x ∈ [e3 , +∞[


(b) x ∈] − ∞, −7[ (f) x ∈]0, e3 ] (Olha o domı́nio!)
(c) x ∈]7, +∞[
(g) x ∈]−ln 2, 0] (Olha o domı́nio!)
(d) x ∈] − ∞, 7[\{1/2, 5/3} (Olha
o domı́nio!) (h) x ∈] − ∞, 0]

Erro ou sugestão? Contacta-me: mailto:ricardoferreira.contactar@gmail.com167


Vê no YouTube: https://www.youtube.com/channel/UCkRcdeyQ50TWFmk7vyuzf_g
Ricardo Ferreira


(2) x ∈] ln( 4 e − 1)/2, +∞[

(3) (a) x ∈ [0, π] (c) x ∈]0, π/6[∪]11π/6, 2π[


(b) x ∈ {0}∪]π/2, π]∪]3π/2, 2π[
(Olha o domı́nio da tangente!) (d) x ∈]π/4, π/2[∪]5π/4, 3π/2[

(4) x = 0 (Olha o domı́nio!)

17.9.5 Zeros e Sinal


(1) (a) Zeros em x = −3, −2, 0, Negativa em ] − ∞, −3[∪] − 2, 0[, Positiva
em ] − 3, −2[∪]0, +∞[
(b) Zeros em x = 4, Negativa em ]1, 4[, Positiva em ]0, 1[∪]4, +∞[
(c) Zeros em x = 1/2, e, Negativa em ]0, 1/2[∪]e, +∞[, Positiva em
]1/2, e[

(2) Df =] − 2, −1/e[

(3) x ∈] − ∞, 0]∪]1, 9]

Erro ou sugestão? Contacta-me: mailto:ricardoferreira.contactar@gmail.com168


Vê no YouTube: https://www.youtube.com/channel/UCkRcdeyQ50TWFmk7vyuzf_g
Ricardo Ferreira

17.10 Limites 1 — Como Calcular Limites


Para verificar as respostas que envolvam desenhar gráficos, usa a tua calcu-
ladora gráfica ou visita:

ˆ https://www.desmos.com/calculator (Desmos)

ˆ https://www.geogebra.org/calculator (Geogebra)

Se o resultado de algum limite não coincidir com as soluções, verifica


também em

ˆ https://www.wolframalpha.com (WolframAlpha)

17.10.1 A estratégia geral


17.10.2 Os limites clássicos

(1) (a) +∞ (g) 0 (m) +∞


(b) +∞ (h) 0
(c) +∞ (i) +∞ (n) −∞
(d) 0 (j) −∞ (o) +∞
(e) −2 (k) +∞
(f) 4 (l) 0 (p) −∞

17.10.3 Truques I: Álgebra de Limites

(1) (a) 0 (f) 0 (k) 0


(b) 0 (g) 0 (l) 0
(c) 0 (h) 0 (m) 0
(d) 0 (i) 0 (n) 0
(e) 0 (j) +∞ (o) 0

(2) (a) +∞ (f) −∞


(b) −∞ (g) 0
(c) +∞
(d) −∞ (h) −∞
(e) +∞ (i) 0

(3) (a) +∞ (c) −∞


(b) +∞ (d) −∞

(4)

Erro ou sugestão? Contacta-me: mailto:ricardoferreira.contactar@gmail.com169


Vê no YouTube: https://www.youtube.com/channel/UCkRcdeyQ50TWFmk7vyuzf_g
Ricardo Ferreira

(a) 0 (d) +∞
(b) 0 (e) +∞
(c) 0 (f) +∞

(5) (a) +∞ (d) +∞


(b) +∞ (e) +∞
(c) +∞ (f) +∞

(6) (a) −∞ (f) 0


(b) +∞ (g) +∞
(c) −1 (h) 0
(d) +∞ (i) 1
(e) +∞ (j) −∞

17.10.4 Truques II: Inversão

(1) (a) 1 (f) +∞ (k) −∞


(b) 3 (g) 0 (l) 0
(c) −3 (h) +∞ (m) −1
(d) 0 (i) 0 (n) 0
(e) 5 (j) 0 (o) +∞

17.10.5 Truques III: Intermediário


(1) 1

(2) (a) 1 (d) 0 (g) 0


(b) 2 (e) 0 (h) 0
(c) 1 (f) 0 (i) 0

(3) (a) 1 (b) 1 (c) 1

17.10.6 Truques IV: Colocar em evidência

(1) (a) +∞ (g) +∞ (m) +∞


(b) −∞ (h) +∞ (n) −∞
(c) +∞ (i) −∞
(o) −1/3
(d) +∞ (j) +∞
(p) +∞
(e) +∞ (k) +∞
(f) −∞ (l) −∞ (q) −2

Erro ou sugestão? Contacta-me: mailto:ricardoferreira.contactar@gmail.com170


Vê no YouTube: https://www.youtube.com/channel/UCkRcdeyQ50TWFmk7vyuzf_g
Ricardo Ferreira

17.10.7 Truques V: Conjugado

(1) (a) 0 (f) −1/2


(b) 0 (g) −∞
(c) −∞ (h) 1/2
(d) +∞ (i) 1/2
(e) 0 (j) −1

(2) (a) 0 (d) 0


(b) 1/2
(c) −∞ (e) 0

17.10.8 Truques VI: Mudança de variável

(1) (a) 1 (f) +∞


(b) 1/4 (g) +∞
(c) 1/3 (h) +∞
(d) 1/4 (i) 1
(e) +∞ (j) −1

(2) (a) 0 (d) 0


(b) 0
(c) 0 (e) 0

17.10.9 Truques VII: Criar constante

(1) (a) 3 (d) −1/5 (g) 1


(b) −1 (e) 0 (h) −3/5
(c) 1 (f) 0 (i) 2/3

(2) (a) 2 (b) 2 (c) 0

17.10.10 Truques VIII: Um limite seminotável

(1) (a) 1 (d) 3 (g) −∞


(b) 1 (e) 5
(c) 2 (f) +∞ (h) 1

Erro ou sugestão? Contacta-me: mailto:ricardoferreira.contactar@gmail.com171


Vê no YouTube: https://www.youtube.com/channel/UCkRcdeyQ50TWFmk7vyuzf_g
Ricardo Ferreira

17.10.11 Truques IX: Continuidade

(1) (a) +∞ (f) 0


(b) +∞ (g) +∞
(c) +∞
(d) 0 (h) +∞
(e) 1 (i) 1

17.10.12 Como calcular qualquer limite

(1) (a) −∞ (h) 0


(b) +∞ (i) +∞
(c) 4/3
(j) 3
(d) −1
(k) +∞
(e) 0
(f) −∞ (l) 1/5
(g) −∞ (m) +∞

Erro ou sugestão? Contacta-me: mailto:ricardoferreira.contactar@gmail.com172


Vê no YouTube: https://www.youtube.com/channel/UCkRcdeyQ50TWFmk7vyuzf_g
Ricardo Ferreira

17.11 Limites 2 — Continuidade e Assı́ntotas


Para verificar as respostas que envolvam desenhar gráficos, usa a tua calcu-
ladora gráfica ou visita:
ˆ https://www.desmos.com/calculator (Desmos)

ˆ https://www.geogebra.org/calculator (Geogebra)

17.11.1 Limites segundo Heine


17.11.2 Continuidade
(1) (a) lim un = 3− (é importante indicar o “menos” para garantir que un
está no domı́nio de f ). lim f (un ) = −inf ty.
(b) Crescente, lim un = 3, logo un < 3 e por isso está no domı́nio de f .
lim f (un ) = −∞.

(2) (a) lim un = 0. lim f (un ) = +∞.


(b) un 6= 0. lim f (un ) = +∞.

(3) (a) Monótona crescente. lim un = 1/2


(b) +∞

(4) (Vê o gráfico na calculadora). limx→0 f (x) = +∞. O ponto x = 0 não


pertence ao domı́nio, por isso não podemos falar em continuidade nesse
ponto. Uma função só pode ser contı́nua ou não contı́nua em pontos do
seu domı́nio!

(5) (a)

(b) O ponto x = 0 não pertence ao domı́nio, por isso não podemos falar
em continuidade nesse ponto.
(c) Não é contı́nua em x = 0.
(d) Contı́nua em x = 0.

(6) Contı́nua em x = 1. (limx→1− f (x) = limx→1+ f (x) = f (1) = −2)

Erro ou sugestão? Contacta-me: mailto:ricardoferreira.contactar@gmail.com173


Vê no YouTube: https://www.youtube.com/channel/UCkRcdeyQ50TWFmk7vyuzf_g
Ricardo Ferreira

(7) Não contı́nua em x = 2 (limx→2− f (x) = −1 e limx→2+ f (x) = f (2) = 1)


Dica: usa o “limite seminotável” limx→0 ln(x + 1)/x

(8) k = 12 (limx→0− f (x) = k/2 e limx→0+ f (x) = f (0) = 6)

(9) limx→−2+ f (x) = +∞, por isso é impossı́vel ter limx→−2+ f (x) = f (a).

17.11.3 Assı́ntotas Verticais


(1) x = −π/2 e x = π/2 (função é contı́nua no seu domı́nio, por isso só
procuramos nos extremos abertos do domı́nio).

(2) Tem assı́ntota vertical em x = 0, porque limx→0− f (x) = +∞.

(3) (a) Cada ramo é uma função contı́nua porque resulta da soma e quo-
ciente de funções contı́nuas. Assim, f é contı́nua.
(b) Nós só verificámos que um limite lateral não é ±∞. Mas para que
f não tenha assı́ntota vertical, é preciso que os dois limites laterais
não sejam ±∞.
(c) Tem assı́ntota vertical em x = 1, porque limx→1+ f (x) = +∞.

(4) (a) Cada ramo é contı́nuo porque resulta da soma e quociente de funções
contı́nuas. Por isso, f é contı́nua em [0, 2]\{1} Se houver assı́ntota
vertical, será onde a função possa não ser contı́nua, ou seja, x = 1.
(b) Nenhum dos limites laterais é ±∞: limx→1+ f (x) = f (1) = −3 e
limx→1− f (x) = 2).

(5) (a) A função f é contı́nua porque resulta da soma e quociente de funções


contı́nuas.
(b) (limx→−1+ f (x) = −1/2 6= ±∞).

17.11.4 Assı́ntotas Não Verticais


(1) (a) A função tem domı́nio ]1, +∞[, por isso limx→−∞ f (x) não está
definido.
(b) y = 3x + 2

(2) (a) A função tem domı́nio [0, +∞[, por isso limx→−∞ f (x) não está
definido.
(b) Não tem assı́ntota em +∞.

(3) y = 0 (em −∞)

(4) (a) A função tem domı́nio ] − ∞, 3], por isso limx→+∞ f (x) não está
definido.
(b) y = −1

Erro ou sugestão? Contacta-me: mailto:ricardoferreira.contactar@gmail.com174


Vê no YouTube: https://www.youtube.com/channel/UCkRcdeyQ50TWFmk7vyuzf_g
Ricardo Ferreira

17.11.5 Recapitulando
(1) (a) Como o domı́nio é ]0, π/2[, não existem assı́ntotas em ±∞. E como
a função f é o quociente de funções contı́nuas, então f é contı́nua
em ]0, π/2[ e por isso não existem aı́ assı́ntotas verticais. assim,
só podem existir assı́ntotas nos extremos abertos do domı́nio, ou
seja,em x = 0 e x = π/2.
(b) limx→0+ f (x) = 2, limx→(π/2)− f (x) = 0.

(2) (a) Não é contı́nua em x = 1 (porque limx→1+ ∞ f (x) = f (1) = 0 mas


limx→1− ∞ f (x) = −1/4).
(b) Assı́ntota oblı́qua y=x+1 e assı́ntota vertical x=-3 (limx→−3 f (x) =
+∞).

(3) k = 0

Erro ou sugestão? Contacta-me: mailto:ricardoferreira.contactar@gmail.com175


Vê no YouTube: https://www.youtube.com/channel/UCkRcdeyQ50TWFmk7vyuzf_g
Ricardo Ferreira

17.12 Derivadas — Definição, Declive da Reta Tangente,


Monotonia e Concavidade
Para verificar as respostas que envolvam desenhar gráficos, usa a tua calcu-
ladora gráfica ou visita:

ˆ https://www.desmos.com/calculator (Desmos)

ˆ https://www.geogebra.org/calculator (Geogebra)

Se o resultado de alguma derivada não coincidir com as soluções, verifica


também em

ˆ https://www.wolframalpha.com (WolframAlpha)

17.12.1 Taxa Média de Variação e Derivada

(1) (a) 0 (f) −2/π


(b) 1 (g) 4/π
(c) −2
(d) 0 (h) 0
(e) 2 (i) 0 (Dica: cos α = cos(−α))

17.12.2 Como calcular derivadas


(1) 0

(2) (a) 1 (h) −2x−3 (o) (1/4)x−3/4


(b) 2x (i) −5x−6 (p) (5/4)x1/4
(c) 3x2 (j) 1
(q) (1/4)x−3/4
(d) 4x3 (k) (1/2)x−1/2
(r) (−1/3)x−4/3
(e) −x−2 (l) (1/2)x−1/2
(f) −x−2 (m) (1/3)x−2/3 (s) (−5/4)x−9/4
(g) −2x−3 (n) (1/3)x−2/3 (t) (−5/4)x−9/4

(3) (a) 1 (h) 6x (o) (5/2)x1/4


(b) 4 (i) −4x
(p) (9/4)x−1/4
(c) −1 (j) x
(d) −3 (k) x (q) −x−3/2
(e) 1/3 (l) 15x2
(r) (−1/4)x−3/2
(f) 1/3 (m) −9x2
(g) 2x (n) −x2 (s) (−1/3)x−7/3

Erro ou sugestão? Contacta-me: mailto:ricardoferreira.contactar@gmail.com176


Vê no YouTube: https://www.youtube.com/channel/UCkRcdeyQ50TWFmk7vyuzf_g
Ricardo Ferreira

(4) (a) 2x + 1 (m) 6x2 − 6x


(b) 3x2 + 2x (n) 2x3 − (1/5)x−8/5
(c) 1 − x−2 (o) 2x + 1
(d) 4x3 + (1/2)x−1/2 (p) 2x + 1
(e) (1/3)x−2/3 + (−1/3)x−4/3
(q) 2x − 1
(f) (3/4)x−1/4 + (4/3)x1/3
(r) 2x − 1
(g) (5/2)x1/4
(s) 4x − 3
(h) 1
(i) 1 (t) 4x3 + 3x2 + 2x + 1
(j) 1 (u) 4x3 − 9x3 + 12x − 3
(k) −3 (v) 2x−3/5 + (1/10)x−7/5
(l) 525 (w) (−1/2)x−3/2 + 16x + 6x−1/2

(5) (a) − cos x (d) (−1/2) cos x − (1/6) sin x


(b) sin x (e) 2 cos x + 2 sin x

(c) − sin x + cos x (f) 5 cos +7 sin x + 6 x

(6) (a) sin x + x cos x


(b) cos x − x sin x
(c) cos2 x − sin2 x
(d) 3 cos2 x − 3 sin2 x
(e) 525 cos2 x − 525 sin2 x
(f) − cos2 x + sin2 x

(g) 2 x cos x + x−1/2 sin x
(h) sin x + x cos x + cos x
(i) (2x − 2) sin x + (x2 − 2x) cos x
(j) (6x2 − 6x) cos x − (2x3 − 3x2 ) sin x
(k) sin x cos x + x cos2 x − x sin2 x
(l) sin x cos x + x cos2 x − x sin2 x
(m) −2x sin x cos x − x2 cos2 x + x2 sin2 x
x cos x − sin x
(7) (a)
x2
sin x − x cos x
(b) x
sin2 x
−x sin x + cos x
(c)
x2

Erro ou sugestão? Contacta-me: mailto:ricardoferreira.contactar@gmail.com177


Vê no YouTube: https://www.youtube.com/channel/UCkRcdeyQ50TWFmk7vyuzf_g
Ricardo Ferreira

−x sin x + cos x
(d)
3x2
−x sin x + 3 cos x
(e)
x4
−1/2 √
x + 2 x sin x
(f)
cos2 x
−1
(g)
sin2 x
−1
(h)
1 + 2 sin x cos x
− sin x + x2 sin x + 2x cos x
(i)
x4 − 2x2 + 1
− sin x + x2 sin x + 2x cos x + 6x
(j)
x4 − 2x2 + 1
2
−x − 2x + 2
(k)
x4 − 2x2 + 4
sin2 x − cos2 x
(l)
sin2 x cos2 x
− sin2 x + cos2 x
(m)
525 sin2 x cos2 x
x cos2 x − x sin2 x + 2 sin x cos x
(n)
x3
(o) cos x + 2x−3
(p) cos x + 2x−3

(8) (a) 2
(b) cos(2x)
(c) 2 cos(2x)

(9) (a) 1 − sin x


(b) 3(x + cos x)2
(c) 3(1 − sin x)(x + cos x)2

(a) −3 cos(−3x) (h) −3 sin x cos2 x


(b) −(3x2 + 4) sin(x3 + 4x) (i) (1/2) cos x sin1/2 x
(c) (6x2 − 3x) sin(−2x3 + 3x2 ) (j) (1/2) cos x(sin x)−1/2

(d) (1/2)x−1/2 sin( x) (k) (−1/2) cos x(sin x)−3/2
(e) − sin x cos(cos x)
(l) (1/2)(2x − 4)(x2 − 4x)−1/2
2
(f) 2 (m) 10(x2 − 1)(x3 − 2x)4
cos (2x)
(g) 2 sin x cos x (n) 2(cos2 x − sin2 x)

Erro ou sugestão? Contacta-me: mailto:ricardoferreira.contactar@gmail.com178


Vê no YouTube: https://www.youtube.com/channel/UCkRcdeyQ50TWFmk7vyuzf_g
Ricardo Ferreira

(o) 6 sin(3x) cos(3x) (p) −10 sin(2x) cos4 (2x)

(10) (a) 525ex (i) 2e2x


(b) ex 2
(j) 2xex
(c) ex + 2 3 −3x2 +9x)
(d) xex + ex (k) (3x2 − 6x + 9)e(x
(e) ex sin x + ex cos x (l) −3e−3x + 2e2x − e−x

(f) 2ex x + ex x−1/2
2x(e2x + e−2x ) + e2x − e−2x
xex − ex (m)
(g) x
x2
x − 8x3 + 12x2
4 2xe2x cos x − e2x sin x − e2x cos x
(h) (n)
ex x2
(11) (a) 3x ln 3 (b) 5x ln 5 (c) (2x + 2)7x ln 7

(12) (a) 525/x (l) 2/x


(b) 1/x 525
(m)
(c) (1/x) + 2 525x + 4
2x3 + 7
(d) (1/x) + 3ex (n) 3
x + 7x
(e) 1 + ln x
4x3 − 12x2 + 10x
(f) ex ln x + (ex /x) (o)
x4 − 4x3 + 5x2
(g) − sin x ln x + (cos x/x) (p) (40 ln x)/x (regras dos logaritmos)
1 − 2 ln x ln(3x) + 1 + 2x ln(3x)
(h) (q)
x3 e2x
xe ln x + ex − ex ln x
x
5(ln x)4
(i) (r)
x2 x
(j) 1/x
15(ln(5x + 1))2
(k) 1/x (s)
5x + 1
1 6x 3x2 + e2x
(13) (a) (b) 2 (c)
x ln 3 (3x + 1) ln 5 (x3 + ex ) ln 7

17.12.3 Aplicações I: calcular limites


(1) −3

(2) −2/3

(3) −4

17.12.4 Aplicações II: declive da tangente ao gráfico


(1) 4

Erro ou sugestão? Contacta-me: mailto:ricardoferreira.contactar@gmail.com179


Vê no YouTube: https://www.youtube.com/channel/UCkRcdeyQ50TWFmk7vyuzf_g
Ricardo Ferreira

(2) −1/(3 ln 3)

(3) 4 ln 2
−x x
(4) (a) f 0 (x) = √ , g 0 (x) = −f 0 (x) = √
2 1 − x2 2 1 − x2
(b) −3/4
(c) −3/4 (Dica: Se o vetor diretor da reta tangente tem coordenadas
(x, y), então tem de satisfazer (x, y) · (3/5, 4/5) = 0)
(d) (−3/4)x + (5/4)

(e) −1/ 3

(f) −1/ 3 (Dica: Se o declive de uma reta é tan α, então o declive da
reta perpendicular é tan(α + π/2))
√ √
(g) (−1/ 3) − (2/ 3)
(h) x = (4/3)x − (5/3)

(5) (a) A(π/3, 0), B(ln 2, 0)


(b) x = (π + ln 2)/5 (interseção de y = −3x + π e y = 2x − 2 ln 2)

17.12.5 Aplicações III: monotonia e extremos


(1) (a) (Vê gráfico na calculadora). A função x é crescente e −x é descres-
cente.
(b) x0 = 1, (−x)0 = −1

(2) (a) (Vê calculadora)


(b) f 0 (x) = 3x2 é zero em x = 0.
(c) Crescente em R. Não tem extremo em x = 0 porque a monotonia
não muda(a função é crescente tanto à esquerda como à direita de
x = 0)

(3) (a) (f 0 (x) = − ln x/x2 ). Crescente em ]1/2, 1[, decrescente em ]1, 2[.
Máximo relativo ln 2 (em x = 1). Existe um extremo relativo.
(b) Crescente em [1/2, 1], decrescente em [1,2]. Máximo relativo ln 2 (em
x = 1). Mı́nimos relativos 2 − 2 ln(2) e (ln(2) + 1)/2 (em x = 1/2 e
x = 2 respetivamente). Existem três extremos relativos.

(4) (f 0 (x) = 4x(x − 1)(x − 2)). Decrescente em ] − ∞, 0] ∪ [1, 2], crescente


em [0, 1] ∪ [1, +∞[. Máximo relativo 1 (em x = 1) e mı́nimo relativo 0
(em x = 0 e x = 2)

(5) (f 00 (x) = −x(2 ln(2x) + 1)). f 0 é crescente em ]0, e−1/2 /2)[, decrescente
em ]e−1/2 /2), +∞[. Máximo relativo e−1 /8 (em x = e−1/2 /2).

Erro ou sugestão? Contacta-me: mailto:ricardoferreira.contactar@gmail.com180


Vê no YouTube: https://www.youtube.com/channel/UCkRcdeyQ50TWFmk7vyuzf_g
Ricardo Ferreira

17.12.6 Aplicações IV: concavidade e pontos de inflexão


(1) (a) (Vê calculadora)
(b) (x2 )00 = 2 e (−x2 )00 = −2.

(2) (a) (Vê Calculadora)


(b) f 00 (x) = 12x é zero em x = 0.
(c) Concavidade voltada para cima em R. Não tem ponto de inflexão
em x = 0 porque o sentido das concavidades não muda (temos
f 00 (x) > 0 tanto à esquerda como à direita de x = 0)

(3) Para as seguintes funções f , estuda a função f quanto ao sentido das


concavidades e quanto à existência de pontos de inflexão. Na tua re-
sposta, inclui:

ˆ os intervalos em que f tem concavidade voltada para cima;


ˆ os intervalos em que f tem concavidade voltada para baixo;
ˆ as abcissas dos pontos de inflexão de f , caso existam.

(a) Concavidade voltada para cima em [0, 1/2]. Concavidade voltada


para baixo em [1/2, 1]. Ponto de inflexão em x = 1/2.
(b) Concavidade voltada para baixo em R. Não há ponto de inflexão (a
concavidade não muda).
(c) Concavidade voltada para cima em [−2, +∞[. Concavidade voltada
para baixo em ] − ∞, −2]. Ponto de inflexão em x = −2.

(4) (a) Mı́nimo


(b) Máximo

Erro ou sugestão? Contacta-me: mailto:ricardoferreira.contactar@gmail.com181


Vê no YouTube: https://www.youtube.com/channel/UCkRcdeyQ50TWFmk7vyuzf_g
Ricardo Ferreira

17.13 Teorema de Bolzano–Cauchy


(1) (a) f (0) = −1 < 0
(b) f (π/2) = 1 > 0
(c) A função f é contı́nua em [0, π/2] porque resulta da soma de duas
funções contı́nuas.

(2) f (−1) = −1 < 0, f (1) = 1 > 0. Não podemos aplicar o teorema de


Bolzano–Cauchy porque f não é contı́nua em ] − 1, 1[ (tanto que f nem
está definida em x = 0!)

(3) (Considera h(x) = f (x) + 1. h(-2)=3/2¿0, h(-1/2)=-1¡0, h é contı́nua


em [−2, −1/2] porque é a soma de funções contı́nuas. Pelo teorema,
existe solução em [−2, −1/2] para h(x) = 0, ou seja, para f (x) = −1).

(4) (f 0 (x) = −2(1 − ln x)/x. f 0 ( e) = −1/e < 0, f 0 (e) = 2/e2 > 0, f 0 é
contı́nua em [−2, −1/2] porque resulta da soma e quociente de funções
contı́nuas.

(5) (f 0 (x) = xex + ex . Considera h(x) = f 0 (x) − 1. h(−1) = −1 < 0, h(e) =


2e − 1 > 0, h é contı́nua em [−1, 1] porque resulta da soma e produto
de funções contı́nuas. Pelo teorema, existe solução em ] − 1, 1[ para
h(x) = 0, ou seja, para f’(x)=1).

(6) (Considera h(x) = f (x) − g(x). h(1) = −1 < 0, h(e) = e2 − 3 > 0, h


é contı́nua em [1, e] porque resulta da soma de funções contı́nuas. Pelo
teorema, existe solução em ]1, e[ para h(x) = 0, ou seja, para f(x)=g(x)).

(7) (Considera h(x) = f (x) − g(x). h(0) = −1 < 0, h(π/2) = (π/2) + 1 > 0,
h é contı́nua em [0, π/2] porque resulta da soma e produto de funções
contı́nuas. Pelo teorema, existe solução em ]0, π/2[ para h(x) = 0, ou
seja, para f(x)=g(x)).

(8) (Bolzano–Cauchy em [0, 1]: considera h(x) = x3 − 4x + 2.h(0) = 2 >


0, h(1) = −1 < 0, h é contı́nua em [0, 1] porque é uma função polinomial.
Pelo teorema, existe solução em ]0, 1[ para h(x) = 0, ou seja, para
x3 + 2 = 4x.)

Erro ou sugestão? Contacta-me: mailto:ricardoferreira.contactar@gmail.com182


Vê no YouTube: https://www.youtube.com/channel/UCkRcdeyQ50TWFmk7vyuzf_g
Ricardo Ferreira

17.14 Combinatória 1 — Como Contar


17.14.1 Estratégia Geral
17.14.2 Como contar I: “E (quantas opções)?”
(1) (a) “Escolhe a bebida E escolhe a sandes E escolhe as batatas fritas”.
(b) 3 × 4 × 2 = 24

(2) (a) “Pinta a 1ª divisão E pinta a 2ª divisão E pinta a 3ª divisão E pinta


a 4ª divisão”.
(b) 5 × 5 × 5 × 5 = 54 = 625.

(3) (a) 4 × 4 × 4 × 4 = 256 (e) 3 × 4 × 4 × 4 = 192


(b) 1 × 4 × 4 × 4 = 64
(f) 4 × 4 × 1 × 1 = 16
(c) 1 × 4 × 4 × 1 = 16
(d) 4 × 4 × 4 × 2 = 128 (g) 4 × 3 × 2 × 1 = 24

17.14.3 Como contar II: “Ou”


(1) (a) 1 × 4 × 4 × 4 = 64
(b) 1 × 2 × 4 × 4 = 32
(c) 0
(d) 64 + 32 + 0 = 96

(2) 1 × 1 × 4 + 1 × 4 × 4 + 1 × 3 × 4 + 0 = 32
(Separa em casos dependendo se o primeiro dı́gito é 2, 3, 5 ou 7).

17.14.4 Como contar III: “Ordenar”


(1) (a) 3! = 6
(b) 4! = 24
(c) 5! = 120
(d) 1 × 4! = 24
(e) 2 × 4! = 48
(f) 4 × 4! = 96
(g) 2! × 3! = 12
(h) 3 × 2! × 2! = 12

Erro ou sugestão? Contacta-me: mailto:ricardoferreira.contactar@gmail.com183


Vê no YouTube: https://www.youtube.com/channel/UCkRcdeyQ50TWFmk7vyuzf_g
Ricardo Ferreira

17.14.5 Como contar IV: “Escolher”


Nota: Sempre que fizermos n C1 , ou seja, se em n escolhermos 1, então
sabemos logo o número de maneiras: são n maneiras. Por isso, onde estiver
n, lembra-te que também podia estar n C1 , porque é a mesma coisa.

(1) (a) “Dos 5 romances escolhe 2 E dos 4 de ficcão cientı́fica escolhe 3 E


dos 7 de autoajuda escolhe 1.”
(b) 5 C2 × 4 C3 × 7 C1 = 10 × 4 × 7 = 280.

(2) (a) “Dos 2 de ficcão cientı́fica escolhe 1 para porão E dos 5 de autoajuda
escolhe 3 para porão E dos 4 romances escolhe 2 para porão.” (os
5 livros que restam Vão automaticamente para a mala de mão, ou
seja, 5 C5 = 1)
(b) 2 C1 × 5 C3 × 4 C2 = 2 × 10 × 6 = 120.

(3) (a) 5 C2 × 4 C2 = 60.


(b) 5 C1 × 4 C2 + 5 C2 × 4 C1 = 70.
52
(4) (a) C1 = 52 (h) 13 × 13 × 13 × 13 = 28561
52
(b) C3 = 22100 (i) 4 × 13 C4 = 2860
52
(c) C51 = 52 (j) 4 C2 × 13 C3 × 13 = 22308
48
(d) C3 = 17296 (k) 4 C2 × 13 C2 × 13 C2 = 36504
13
(e) C4 = 715 (l) 13 × 12 × 11 × 10 = 17160
13 13
(f) C3 × C2 = 22308 (m) 4 C2 × 13 C2 × 11 C2 = 25740
13
(g) C4 × 13 C2 × 13 C2 = 22308 (n) 13
C3 + 39 C3 = 9425

17.14.6 Estratégia Geral II e Truques Especiais


(1) (a) “Dos 11 espaços escolher 2 para prof. de História E dos 9 espaços
que restam, escolher 5 para prof. de Matemática E dos 4 espaços
que restam, escolher 4 para prof. de História E ordenar os 2 prof.
de História E ordenar os 5 prof. de Matemática E ordenar os 4 prof.
de Português”.
11
(b) C2 × 9 C5 × 4 C4 × 2! × 5! × 4! (= 39916800)

(2) (a) 3 C1 × 4 C2 + 3 C2 × 4 C1 + 3 C3 = 31
(b) 7 C3 − 4 C3 = 31
10
(3) (a) C4 × 4! − 5 C4 × 4! = 4920 (“total” − “caso em que todos os dı́gitos
são ı́mpares”)
(b) 5 C3 × 5 C1 × 4! + 5 C4 × 4! = 1320 (“3 dı́gitos pares OU 4 dı́gitos
pares”)

Erro ou sugestão? Contacta-me: mailto:ricardoferreira.contactar@gmail.com184


Vê no YouTube: https://www.youtube.com/channel/UCkRcdeyQ50TWFmk7vyuzf_g
Ricardo Ferreira

(4) (a) 3! × 3! × 2! = 72 (“ordenar as 3 caixas E ordenar os 3 livros da caixa


de Matemática E ordenar os 2 livros da caixa de Inglês.)
(b) 3! × 3! × 2! = 72 (“ordenar as 3 “caixas” E ordenar os 3 livros da
caixa de Matemática E ordenar os 2 livros da caixa de Inglês.)

(5) (a) 8!×2! = 80640 (“ordenar as 8 “caixas” E ordenar os 2 dirigentes”).


(b) 4!×2!×5! = 5760 (“ordenar as 4 “caixas” E ordenar os 2 dirigentes
E ordenar os 5 atletas E).
(c) 3! × 2! × 5! × 2! (“ordenar as 3 “caixas” E ordenar os 2 dirigentes
E ordenar os 5 atletas E ordenar os 2 treinadores).

(6) (a) 7! × 8 C2 × 2! = 282240 (“ordena os 7 não dirigentes E dos 8 espaços


vazios escolhe 2 para meter os dois dirigentes E ordena os 2 diri-
gentes”)
(b) 5! × 6 C4 × 4! = 43200 (“ordena os 5 atletas E dos 6 espaços vazios
escolhe 4 para meter os quatro não atletas E ordena os 4 não atle-
tas”)
(c) 4!× 5 C5 ×5! = 2880 (“ordena os 4 não atletas E dos 5 espaços vazios
escolhe 5 para meter os atletas E ordena os 5 atletas”)
11
(7) (a) C3 × 3! × 8!
(Dos 11 escolher 3 para sentar E ordenar os 3 sentados E ordenar
os 8 em pé).
(b) 3! × 8!
(Ordenar os 3 alunos do sexo masculino sentados E ordenar os 8 em
pé).
(c) 8 C3 × 3! × 8!
(Dos 8 não alunos do sexo masculino escolher 3 para sentar E or-
denar os 3 sentados E ordenar os 8 em pé).
11
(d) C3 × 3! × 8! − 8 C3 × 3! × 8!
(negação, usando (a) e (c))
(e) 8 C3 × 3! × 2! × 3! × 5!
(Dos 8 não alunos do sexo masculino escolher 3 para sentar E or-
denar os 3 sentados E ordenar as 2 “caixas” em pé E ordenar os 3
três alunos do sexo masculino em pé E ordenar os outros 5 membros
em pé).
(f) 3! × 8! + 11 C3 × 3! × 2! × 3! × 5!
(Os três alunos do sexo masculino ficam juntos sentados OU os três
alunos ficam juntos em pé)

Erro ou sugestão? Contacta-me: mailto:ricardoferreira.contactar@gmail.com185


Vê no YouTube: https://www.youtube.com/channel/UCkRcdeyQ50TWFmk7vyuzf_g
Ricardo Ferreira

(g) 9 C3 × 3! × 6! × 7 C2
(Dos 9 não professores escolher 3 para sentar E ordenar os 3 sentados
E ordenar os 6 não professores em pé E dos 7 espaços vazios entre
eles escolher 2 para colocar os professores).
(h) 3 C1 × 7 C2 × 3! × 5! × 6 C3
(Dos 3 lugares sentados escolher 1 para a professora sentar E dos
7 não alunos do sexo masculino e não professora escolher 2 para
sentar E ordenar os 3 sentados E ordenar os 5 não alunos do sexo
masculino e pé E dos 6 espaços vazios entre eles escolher 3 para
colocar os alunos do sexo masculino).
(i) 6 C1 × 2! × 2! × 5! × 6 C3 + 6 C3 × 3! × 4! × 5 C3 × 2!
(Os dois professores ficam juntos sentados OU os dois professores
ficam juntos em pé)
(Dos 6 não alunos do sexo masculino e não professores escolher 1
para sentar E ordenar as 2 “caixas” sentadas E ordenar os 2 pro-
fessores dentro da caixa E ordenar os 5 não alunos do sexo masculino
em pé E dos 6 espaços vazios entre eles escolher 3 para colocar os
alunos do sexo masculino OU Dos 6 não alunos do sexo masculino
e não professores escolher 3 para sentar E ordenar os 3 sentados E
ordenar as 4 “caixas” que podem ficar juntas (uma caixa com os
2 professores, e três caixas com uma aluna do sexo feminino cada)
E dos 5 espaços vazios entre essas caixas escolher 3 para colocar
os alunos do sexo masculino E ordenar os 2 professores dentro da
caixa).

Erro ou sugestão? Contacta-me: mailto:ricardoferreira.contactar@gmail.com186


Vê no YouTube: https://www.youtube.com/channel/UCkRcdeyQ50TWFmk7vyuzf_g
Ricardo Ferreira

17.15 Combinatória 2 — Triângulo de Pascal, Binómio de


Newton e Equações
17.15.1 Triângulo De Pascal

(1) (a) 13 (g) 4 (m) 5


(b) 13 (h) 12 (n) 15
(c) 525 (i) 524 (o) 26
(d) 4 (j) 8 (p) 13
(e) 5 (k) 14 (q) 25
(f) 7 (l) 11 (r) 13

(2) (a) 1 C0 = 1 C1 = 1 (e) 8 C4 = 70


(b) 2 C1 = 2 (f) 11
C5 = 11 C6 = 462
(c) 3 C1 = 3 C2 = 3 (g) 300
C150
4 401
(d) C2 = 6 (h) C200 = 401 C201

(3) (a) 7 (h) 5 elementos (todos os 9 ele-


(b) 66 mentos exceto 8 C0 , 8 C1 , 8 C7 e
8
(c) 22 C8 )
(d) 56 − 1 = 55
(i) 514 elementos (todos os 526
(e) n=3 elementos exceto os 8 el-
(f) n = 10 ementos 525 Ck com k ∈
(g) 1 elemento. {0, 1, 2, 3, 522, 523, 524, 525})

(4) (a)
1
1 1
1 2 1
1 3 3 1
1 4 6 4 1

(b) 3 C1 = 3 C2 = 3 e 4 C2 = 6. Temos a relação 3 C1 + 3 C1 = 4 C2 .


(c) 9 C4 = 9 C5 e 10
C5 . Temos a relação 9 C4 + 9 C5 = 10 C5 .
(d) 126 + 126 = 252
(e) 70/2 = 35

(5) (a) 26 = 64 (estamos na linha 6)

Erro ou sugestão? Contacta-me: mailto:ricardoferreira.contactar@gmail.com187


Vê no YouTube: https://www.youtube.com/channel/UCkRcdeyQ50TWFmk7vyuzf_g
Ricardo Ferreira

(b) 9 (estamos na linha 8)


(c) 8 C4 = 70 (estamos na linha 8)

(6) (a) n = 3
(b) n = 5
(c) n = 4

17.15.2 Binómio de Newton


(1) (a) 1 3 3 1
(b) x3 + 6x2 + 12x + 8.

(2) (a) 1 4 6 4 1.
(b) x4 − 4x3 + 6x2 − 4x + 1.

(3) (a) x3 − 6x2 + 12x − 8. (e) x4 + 4x2 + 6 + 4x−2 + x−4 .


(b) 8x3 + 12x2 + 6x + 1. (f) x3 − 3x + 3x−1 − x−3 .
x3 x2 (g) x10 + 5x17/2 + 10x7 + 10x11/2 +
(c) − + x − 1. 5x4 + x5/2 .
27 3

(d) x6 + 3x5 + 3x4 + x3 . (h) 10 − 6 3.

12
(4) (a) Ck x24−3k
(b) 66x15 (k = 2)
12
(c) C6 x9 (k = 5)
12
(d) C8 (k = 8).
(e) 24 − 3k = 2 não tem solução nos números inteiros não negativos.
(f) x24 (k = 0).
(g) 7 termos (24 − 3k é um número par parak ∈ {0, 2, 4, 6, 8, 10, 12}).
n
(5) (a) Ck x(5k−n)/2
(b) n = 9 (resolvendo (5k − n)/2 = 13 com k = 7)
(c) n = 8 (resolvendo (5k − n)/2 = 13 com k = n/2)
(d) (5k −11)/2 = 0 não tem solução nos números inteiros não negativos.
12
(e) C2 (k = 2 solução de (5k-10)/2=0)
(f) n = 5 (se a equação 5k − n = 0 tem solução, então temos n = 5k.
O menor valor de n é para k = 1)
(g) n = 7 (temos n = 5k + 7. O menor valor de n é para k = 0)

Erro ou sugestão? Contacta-me: mailto:ricardoferreira.contactar@gmail.com188


Vê no YouTube: https://www.youtube.com/channel/UCkRcdeyQ50TWFmk7vyuzf_g
Ricardo Ferreira

17.15.3 Equações

(1) (a) n ∈ {0, 525} (h) n ∈ {0, 263}


(b) n ∈ {1, 524} (i) n ∈ {2, 525}
(c) n ∈ {2, 523}
(j) n ∈ {1, 524}
(d) n ∈ {11, 514}
(k) n ∈ {1, 524}
(e) n ∈ {3, 5}
(f) n ∈ {0, 1, 3} (l) n ∈ {1, 2}
(g) n ∈ {0, 526} (m) n ∈ {0, 1, 2, 3}

(2) (a) n = 12 (d) n = 5 (g) n = 7


(b) n = 254 (e) n = 6
(c) n = 14 (f) n = 5

(3) (a) n = 525 (d) n = 4 (g) n = 525


(b) n = 7 (e) n = 3 (h) n = 525
(c) n = 3 (f) n = 3 (i) n = 525

(4) (a) n = 6 (n C2 = 15).


(b) n = 7 (n Cn−2 = 21).
(c) n = 6 (n C2 = 6 C4 ).
(d) n = 12 (n C4 = n C8 ).

(5) (a) n = 3 (2n C2 = 15).


(b) n = 13 (3n = 39).
(c) n = 13 (2n C2 − 3n = 16).
(d) n = 4 (2 × n × n C2 = 4 × 3n — temos de escolher 2 pontos de uma
base, 1 ponto da outra).
(e) n = 4 (2 × n C2 + 3n = 24 — cada base tem n C2 segmentos. As
faces laterais têm 3n segmentos no total: n arestas e 2n segmentos
diagonais).

Erro ou sugestão? Contacta-me: mailto:ricardoferreira.contactar@gmail.com189


Vê no YouTube: https://www.youtube.com/channel/UCkRcdeyQ50TWFmk7vyuzf_g
Ricardo Ferreira

17.16 Probabilidades — Regra de Laplace e Fórmulas das


Probabilidades
17.16.1 Probabilidades I: Regra de Laplace
5
C3 + 5
(1) P = 6 = 0.75
C3
1 × 3 × 10 × 1 × 1 + 3 × 10 × 10 × 1 × 1 33
(2) P = 4
=
10 10000
(Número começa com 2 OU número começa com 4, 6 ou 8)
2
C1 × 5 C2 × 3! × 4! × 3 C2
(3) P = 9 ≈ 0.71.
C3 × 3! × 6!

17.16.2 Probabilidades II: Fórmulas e Tabela

A A A A
B 0.5 0.1 0.6 B 0 0 0
(1) (a) (d)
B 0.2 0.2 0.4 B 0.9 0.1 1
0.7 0.3 1 0.9 0.1 1

A A A A
B 0.3 0.1 0.4 B 3/7 1/7 4/7
(b) (e)
B 0.2 0.4 0.6 B 2/7 1/7 3/7
0.5 0.5 1 5/7 2/7 1

A A A A
B 0 0.7 0.7 B 1/11 4/11 5/11
(c) (f)
B 0.2 0.1 0.3 B 3/11 3/11 6/11
0.2 0.8 1 4/11 7/11 1

A A A A
B 0.2 0.3 0.5 B 0.7 0.1 0.8
(2) (a) (b)
B 0.1 0.4 0.5 B 0 0.2 0.2
0.3 0.7 1 0.7 0.3 1

Erro ou sugestão? Contacta-me: mailto:ricardoferreira.contactar@gmail.com190


Vê no YouTube: https://www.youtube.com/channel/UCkRcdeyQ50TWFmk7vyuzf_g
Ricardo Ferreira

A A
B 1/9 3/9 4/9
(c)
B 2/9 3/9 5/9
1/3 2/3 1

(3) (a) P (A ∩ B) = 0.4



(b) P A ∩ B = 0.1

(c) P A ∩ B = 0.3
(d) P (A) = 0.5

(e) P A = 0.5

(f) P B = 0.4
(g) P (A ∪ B) = 0.5 + 0.6 − 0.4 = 0.7

(h) P A ∪ B = 0.5 + 0.6 − 0.2 = 0.9

(i) P A ∪ B = 0.5 + 0.4 − 0.3 = 0.6
(j) P (A|B) = 0.4/0.6 = 2/3
(k) P (B|A) = 0.4/0.5 = 4/5

(l) P B|A = 0.2/0.5 = 2/5

(4) P A ∩ B = 0.

A A
B 0.7 0.1 0.8
B 0 0.2 0.2
0.7 0.3 1

(5) P A|B = 3/4.

A A
B 0.45 0.15 0.6
B 0.3 0.1 0.4
0.75 0.25 1

(6) (a) P (A ∩ B) = 14/42 = 1/3



(b) P A ∩ B = 1/42

(c) P A ∩ B = 21/42 = 1/2
(d) P (A) = 15/42 = 5/14

Erro ou sugestão? Contacta-me: mailto:ricardoferreira.contactar@gmail.com191


Vê no YouTube: https://www.youtube.com/channel/UCkRcdeyQ50TWFmk7vyuzf_g
Ricardo Ferreira


(e) P A = 27/42 = 9/14

(f) P B = 22/42 = 11/21
(g) P (A ∪ B) = (15/42) + (20/42) − (14/42) = 1/2

(h) P A ∪ B = (27/42) + (20/42) − (6/42) = 41/42

(i) P A ∪ B = (27/42) + (22/42) − (21/42) = 2/3
(j) P (A|B) = 14/20 = 7/10
(k) P (B|A) = 6/27 = 2/9

(l) P B|A = 21/27 = 7/9

(7) 2/11
(Seja H o acontecimento “Ser homem” e A o acontecimento “Almoçar
no restaurante”. A resposta é P (H|A) = 2/11).

H H H H
A 4 18 22 A 1/9 1/2 11/18
(a) (b)
A 8 6 14 A 2/9 1/6 7/18
12 24 36 1/3 2/3 1

Erro ou sugestão? Contacta-me: mailto:ricardoferreira.contactar@gmail.com192


Vê no YouTube: https://www.youtube.com/channel/UCkRcdeyQ50TWFmk7vyuzf_g

Você também pode gostar